Текст
                    Г. Ф.Меледин
ФИЗИКА В ЗАДАЧАХ: ЭКЗАМЕНАЦИОННЫЕ ЗАДАЧИ С РЕШЕНИЯМИ 2-е изд., перераб. и доп.— М.: Наука. Гл. ред. физ.-мат. лит., 1990.— 272 с.
Содержит свыше пятисот задач письменного экзамена по физике, предлагавшихся поступающим на физический факультет Новосибирского государственного университета. В большинстве своем задачи оригинальны. Особый интерес вызывают задачи-оценки и задачи-демонстрации. К наиболее трудным задачам даны подробные решения. В новом издании (1-е изд.—1985 г.) добавлено свыше ста интересных задач конкурсных экзаменов последних лет.
Для слушателей и преподавателей подготовительных отделений и курсов, выпускников и старшеклассников общеобразовательных школ, учащихся физико-математических школ, членов вузовских приемных комиссий, а также лиц, занимающихся самообразованием.
ОГЛАВЛЕНИЕ
Предисловие ко второму изданию	5
Предисловие к первому изданию	7
Основные обозначения	10
ЗАДАЧИ
Глава I. Механика	11
§ 1. Кинематика. Относительность движения (И). § 2. Динамика (15). § 3. Вращательное движение. Колебания (18). § 4. Законы сохранения энергии и импульса. Работа. Мощность (22). § 5.
Статика. Момент сил (31). § 6. Механика жидкости и газа (39).
Глава II. Тепловые явления	48
§ 1. Газовые законы. Тепловое расширение (48). § 2. Пар (57). § 3. Графики для тепловых процессов. Закон сохранения энергии в тепловых процессах (59).
Глава III. Электричество и магнетизм	64
§ 1. Напряженность, потенциал и энергия электростатического поля (64). § 2. Электрическая емкость. Конденсаторы (72). § 3. Цепи постоянного и переменного тока (79). § 4. Электромагнитная индукция. Взаимодействие тока с магнитным полем (87).
Глава IV. Оптика	92
§1. Отражение. Зеркала (92). §2. Преломление (94). § 3. Линзы (96). § 4. Фотометрия (102).
Глава V. Задачи-оценки	104
Глава VI. Задачи-демонстрации	108
ОТВЕТЫ И РЕШЕНИЯ
Г лава Г Механика ,	116
Глава II. Тепловые явления	167
Глава III. Электричество и магнетизм	186
Глава IV. Оптика	220
Глава V. Задачи-оценки	235
Глава VI. Задачи-демонстрации	248
Список рекомендуемой литературы	259
Приложение. Единицы и размерности физических величин в СИ	260
ОГЛАВЛЕНИЕ
Предисловие ко второму	изданию......................... 5
Предисловие к первому	изданию......................... 7
Основные обозначения................................... 10
ЗАДАЧИ
Глава I. Механика...................................... 11
§ 1. Кинематика. Относительность движения (11). § 2. Динамика (15). § 3. Вращательное движение. Колебания (18). § 4. Законы сохранения энергии и импульса. Работа. Мощность (22). § 5. Статика. Момент сил (31). § 6. Механика жидкости и газа (39).
Глава II. Тепловые явления ............................ 48
§ 1. Газовые законы. Тепловое расширение (48). § 2. Пар (57). § 3. Графики для тепловых процессов. Закон сохранения энергии в тепловых процессах (59).
Глава III. Электричество и магнетизм................... 64
§ 1. Напряженность, потенциал и энергия электростатического поля (64). § 2. Электрическая емкость. Конденсаторы (72). § 3. Цепи постоянного и переменного тока (79).
§ 4. Электромагнитная индукция. Взаимодействие тока с магнитным полем (87).
Глава IV. Оптика....................................... 92
§ 1. Отражение. Зеркала (92). § 2. Преломление (94).
§3. Линзы (96). §4. Фотометрия (102).
Глава	V.	Задачи-оценки .	. . .................... 104
Глава	VI.	Задачи-демонстрации..................... 108
1*	3
ОТВЕТЫ И РЕШЕНИЯ
Глава I.
Глава II.
Глава III.
Глава IV.
Глава V.
Глава VI.
Механика................................
Тепловые явления ....................... Электричество и магнетизм .............. Оптика ................................. Задачи-оценки .......................... Задачи-демонстрации ....................
Список рекомендуемой литературы .
Приложение. Единицы и размерности физических величин в Си.........................
116
167
186
220
235
248
259
260
ПРЕДИСЛОВИЕ КО ВТОРОМУ ИЗДАНИЮ
Новое издание учебного пособия, как и прежнее, вызвавшее большой интерес, представляет собой сборник задач, предлагавшихся на вступительных экзаменах по физике в Новосибирском государственном университете. Добавлено свыше ста новых оригинальных задач, накопившихся за годы, прошедшие после первого издания книги. Дополнены практически все разделы задачника. Некоторая часть задач, как и раньше, дополнена числовыми данными. Текст задач и решений из предыдущего издания включен практически без изменений, исправления незначительны. Подход к подбору задач, стиль и характер решений сохранен и в новом издании.
В условиях, когда роль самостоятельности и активности в обучении и развитии мышления усиливается (это касается и отбора в вузы и подготовки к вступительным экзаменам), значимость целого ряда учебных пособий соответствующей направленности заметно возрастает. Хотелось бы надеяться, что этот задачник окажется полезным и с этой точки зрения. Действительно, например, задачи-оценки (число которых увеличено здесь в полтора раза) требуют, чтобы решающий их снач"та сам сформулировал простую физическую модель рассматриваемого явления, выбирая самые главные физические параметры, определяющие в основном физику задачи, и получил ответ, опять-таки задавая самостоятельно числовые значения этих параметров.
Задачи-демонстрации «работают» в сходном направлении. При их решении надо в демонстрируемом (или описываемом в задачнике) явлении среди целого ряда действующих физических факторов выделить главный и объяснить, опираясь на это, физическое явление.
Краткость приводимых в задачнике решений простых и не слишком сложных задач, в свою очередь, оставляет место для размышлений, поиска своего подхода к решению
5
данной задачи, попыткам самостоятельно разобраться в тонкостях решения задачи. Что касается сложных задач, то можно надеяться, что знакомство с их решением и удовлетворение в случае самостоятельного их решения должны поднять интерес к физике и способствовать развитию физического мышления.
Автор полагает, что пособие окажет существенную помощь и тем, кто сам готовится к поступлению в вуз, и тем, кто готовит к этому других.
Составлением задач занимался многочисленный отряд физиков Сибирского отделения АН СССР и Новосибирского государственного университета. К именам, названным в предисловии к первому изданию, теперь следует добавить Г. В. Федотовича и А. П. Ершова.
В заключение автор выражает свою искренную признательность профессору Ленинградского государственного университета Е. И. Бутикову за большую работу с рукописью и конструктивную критику, глубокую благодарность всем участникам создания этой книги.
ПРЕДИСЛОВИЕ К ПЕРВОМУ ИЗДАНИЮ
Решение задач — эффективное средство усвоения физики, надежный инструмент для контроля за степенью понимания физических законов.
Данный сборник содержит большое количество самых разных задач по физике, которые можно решить, не выходя за рамки программы вступительных экзаменов в §узы. В него вошло свыше четырехсот оригинальных задач, предлагавшихся на письменном вступительном экзамене по физике, проводимом в Новосибирском государственном университете начиная с 1966 г. Эти задачи неизменно из года в год вызывают большой интерес школьников, учителей и преподавателей вузов. Они широко используются в физических олимпиадах, разнообразных школах (зимних и летних, заочных и очных, воскресных и вечерних), на подготовительных отделениях и курсах. Некоторые задачи опубликованы в журнале «Квант» и вошли в ряд учебных пособий.
Сборник имеет два основных раздела: «Задачи» и «Ответы и решения». Материал разделов разбит на главы, которые посвящены механике, тепловым явлениям, электричеству и магнетизму и оптике. Здесь немало задач с реальным содержанием, требующих ясного понимания масштабов явле-. ний. Широко также представлены и задачи идеализированные, только моделирующие реальную ситуацию, физический процесс, явление. Такие задачи обязательно используются в практике обучения физике, контроля за ее усвоением. Иногда они полезны для овладения типичными приемами решения задач.
В задачнике увеличено по сравнению с экзаменами количество задач с числовыми данными. Уместно подчеркнуть, что задачи по физике должны учить умению доводить результат до числа, абитуриенту необходимо знать систему единиц, представлять себе размерности и характерные масштабы физических величин.
Среди задач сборника есть совсем простые, стандартные (они отмечены кружком). Немало и сравнительно сложных (есть очень трудные), требующих смекалки, глубоких зна-
7
ний, умения разобраться в непривычной или усложненной ситуации (такие задачи отмечены звездочкой).
В связи с тем, что в задачнике собраны экзаменационные задачи, предполагающие подготовленность по всему школьному курсу физики и математики, использование данного пособия должно быть особенно эффективным и полезным на завершающей стадии подготовки к вступительным экзаменам по физике, на этапе повторения материала. Однако большинство задач пригодно и для начального этапа изучения физики в школе.
Задачи-оценки и задачи-демонстрации, новые в практике письменных вступительных экзаменов и пока еще недостаточно популярные в школе, собраны в отдельные главы: пятую и шестую. В начале каждой из этих глав дается необходимое в данном случае краткое введение — пояснение к таким задачам.
В задачах-оценках (глава пятая) надо самому выбрать необходимые параметры, построить более или менее грубую модель явления, получить числовой результат, прикинуть его реальность. Отметим, что с подобными задачами, отобранными, конечно, в соответствии со школьной программой, уверенно справляется большинство абитуриентов. Учитывая, что оценки физических величин широко используются в практике физика-исследователя (как теоретика, так и экспериментатора), можно предполагать, что они, видимо, постепенно будут завоевывать свои позиции в процессе обучения физике как в школе, так и в вузе.
В задачах-демонстрациях (глава шестая) надо объяснить физическое явление, демонстрируемое на экзамене, среди различных факторов выделить главный.
Задачи-оценки и задачи-демонстрации — самые сложные для абитуриента в отношении подготовки к экзамену задачи: они требуют значительного методического опыта и знания основ физики, поэтому браться за их решение стоит, когда пройдены и повторены все разделы физики. Именно поэтому в первую очередь эти задачи собраны в отдельные главы, отделены от обычных задач соответствующего раздела физики.
Можно надеяться, что сравнительно небольшое количество задач, единых по подходу к решению, позволяющих научиться быстро получать числовой ответ в достаточно сложных физических ситуациях, если уж заинтересует читателя, то будет решено полностью.
В отношении решений наш задачник занимает промежуточное положение среди двух интересных пособий, вышед
8
ших в издательстве «Наука» под одним и тем же названием «Задачи по физике». Один задачник составлен И. И. Воробьевым, П. И. Зубковым, Г. А. Кутузовой и др. под редакцией О. Я. Савченко, авторами другого являются И. Ш. Слобо-децкий и Л. Г. Асламазов.
В отличие от задачника И. И. Воробьева, П. И. Зубкова, Г. А. Кутузовой и др. наш сборник содержит не только ответы, но и решения большинства задач, не выходящих за рамки школьной программы. Разбор задач, особенно сложных, должен существенно помочь читателю в усвоении физики и приемов решения задач и в подготовке к вступительным экзаменам в вузы. Краткость многих решений позволяет не обесценивать задачи: многое еще остается для самостоятельной работы. Приводимые в задачнике ответы и решения можно использовать в качестве указаний и для самоконтроля.
В книге И. Ш. Слободецкого и Л. Г. Асламазова решения сложных задач превратились в глубокие, очень интересные обсуждения вопросов современной физики и техники, стали научно-популярными статьями, доступными школьникам. Кстати, их можно использовать в качестве полезного дополнительного материала при разборе некоторых трудных задач нашего сборника, а также ряда задач-оценок.
Завершает наше пособие список рекомендуемой литературы.
В составлении задач, использованных в сборнике, принял участие широкий круг физиков Сибирского отделения АН СССР и преподавателей Новосибирского государственного университета. Около половины задач предложены Г. Л. Коткиным и Г. В. Мелединым. В составление остальных задач наибольший вклад внесли В. П. Бородин, П. И. Зубков и О. Я. Савченко.
В заключение автор считает своим приятным долгом поблагодарить рецензентов за доброжелательную критику и целый ряд полезных советов, способствовавших улучшению задачника. Важными и существенными для автора были беседы с Г. Л. Коткиным и И. Ф. Гинзбургом. Автор искренне благодарен всем вложившим свой труд в создание Этой книги.
ОСНОВНЫЕ ОБОЗНАЧЕНИЯ
с, и, V — скорость
t, т — время, интервал времени
Д, I, d — расстояние
Ц, h — высота
Й, со — угловая скорость, круговая частота
<р, а, р, у — угол
g — ускорение свободного падения
R, г — радиус
D, d — диаметр, толщина, рае-мер
F — сила
М, т — масса
k — коэффициент трения
а — ускорение
S — площадь
И — жесткость
Т — период
р — ПЛОТНОСТЬ
Ро — плотность ВОДЫ
Р — МОЩНОСТЬ
ч- кпд
Q — количество теплоты
р — давление
Ро — атмосферное давление
а — температурный коэффициент линейного расширения Т, t — температура р — молярная масса с — удельная теплоемкость X — удельная теплота парообразования
U — внутренняя энергия
Ё — напряженность электрического поля
Q, q, е — заряд
О — поверхностная плотность заряда
Ц7 — кинетическая энергия
U — напряжение, разность потенциалов
R — сопротивление
С — емкость
I — ток (сила тока)
п — показатель преломления
В — магнитная индукцйя
Ф *- магнитный поток
Ё — индуктивность
<£ -ЭДС
R — коэффициент отражения г — фокусное расстояние
; 10
ЗАДАЧИ
Глава I. МЕХАНИКА
§ 1.	Кинематика. Относительность движения
1.1	°. При скорости ветра цх=10 м/с капля дождя падает под углом 30° к вертикали. При какой скорости ветра и2 капля будет падать под углом 45°?
1.2	°. Кольцо сварено из двух полуколец радиуса Д, скорости звука в которых равны сх и с2. Через какое время встретятся звуковые волны, возбужденные ударом по точке сварки?
» 1.3°. В реку, скорость течения которой zz=0,5 м/с, из некоторой точки О на берегу у самой воды бросают камень перпендикулярно берегу. Скорость поверхностных волн в воде с=1 м/с. Через какое время после падения камня волна от него придет в точку О, если камень упал в воду на расстоянии 1= 10 м от берега?
1.4	°. Самолет летит по прямой из города М в город N и обратно. Найти отношение полных времен полета в случаях, когда от М к N дует ветер со скоростью и и когда ветер с той же скоростью дует перпендикулярно прямой MN. Скорость самолета относительно воздуха в том и другом случаях равна и.
« 1.5. Гидролокатор подводной лодки, погружающейся вертикально, излучает короткие звуковые сигналы длительности т0 в направлении дна. Длительность отраженных сигналов, измеряемых гидроакустиком на лодке, равна т. Какова скорость погружения лодки? Скорость звука в воде равна с. Дно горизонтально.
1.6	°. При взрыве покоящейся цилиндрической бомбы радиуса R осколки, разлетающиеся в радиальном направлении, за время t удаляются от оси цилиндра на расстояние li. На какое расстояние /2 от оси цилиндра удаляются
11
осколки за то же время t, если в момент взрыва бомба- будет вращаться вокруг своей оси с угловой скоростью со? Влиянием силы тяжести пренебречь.
1.7	*. Самолет пролетает над наблюдателем на постоянной высоте h с постоянной скоростью v, большей скорости звука с. Какой угол с вертикалью составляет направление на самолет, определяемое по с _о	иг звуку в тот момент, когда
истинное (видимое) направление от наблюдателя на самолет составляет с вертикалью угол ср?
Рис. 1.10	о 1.8°. Пассажир первого ваго-
на поезда длины I прогуливался по перрону. Когда он был рядом с последним вагоном, поезд начал двигаться с ускорением а. Пассажир сразу же побежал со скоростью и. Через какое время он догонит свой вагон?
о 1.9°. В момент, когда опоздавший пассажир вбежал на платформу, мимо него прошел — за время t± — предпоследний вагон. Последний вагон прошел мимо пассажира за время /2. На сколько опоздал пассажир к отходу поезда? Поезд движется равноускоренно. Длина вагонов одинакова.
1.10	°. Из одной точки вылетают одновременно две частицы с горизонтальными противоположно направленными скоростями щ=2 м/с и и2=5 м/с (рис. 1.10). Через какой интервал времени угол между направлениями скоростей этих частиц станет равным 90°? Считать, что ускорение свободного падения £=10 м/с2.
1.11	°. Два тела, двигаясь с противоположно направленными скоростями, после соударения на горизонтальной плоскости одновременно остановились через время /=1 с на расстоянии /=1 м друг от друга. Найти коэффициент трения скольжения между плоскостью и каждым телом. Считать, что ускорение свободного падения £=10 м/с2. • 1.12. Из точки А вертикально вверх брошен камень со скоростью v=10 м/с. Через какое время следует бросить с той же по модулю скоростью второй камень из точки В под углом а=45° к горизонту, чтобы он попал в первый камень? Точки А и В расположены на одной горизонтали. Расстояние между ними /=4 м. Считать, что ускорение свободного падения §=10 м/с2.
• 1.13. Утка летела по горизонтальной прямой с постоянной скоростью и (рис. 1.13). В нее бросил камень неопытный охотник, причем бросок был сделан без упреждения, 12
т. е. в момент броска скорость камня v была направлена как раз на утку под углом а к горизонту. На какой высоте
и
-р---S
V 'Да
3
''
а
Рис.
h
летела утка, если камень все же попал в нее? Сопротивлением воздуха, размерами уткн и ростом охотника пренебречь.
1.14.	Какое расстояние по горизонтали пролетит мяч, брошенный со скоростью и=10 м/с под углом а=60° к горизонту, если он ударится о потолок (рис. 1.14)? Высота потолка й=3 м, удар упругий. Сопротивлением воздуха пренебречь.
1.15.	В конической лунке с вертикальной осью симметрии и углом раствора 2а=90° движется шарик (рис. 1.15),


Рис. 1.15
3

»
\
Рис. 1.14
ударяясь через одно н то же время т=1 с о противоположные точки А и В, расположенные на одной горизонтали. Найти максимальную и минимальную скорости шарика. Считать, что ускорение свободного падения g=10 м/с2.
1.16.	На поверхности земли взорвался маленький шар так, что осколки разлетелись во веб стороны равномерно с одинаковой по модулю скоростью и. Какова масса осколков, выпавших на поверхность земли вне круга радиуса £ (с центром в точке взрыва), если полная мйсса осколков равна Л4? Сопротивлением воздуха пренебречь.
1.17*	. Под каким наименьшим углом к горизонту следует бросать мяч, чтобы он пролетел сквозь бйскетббльное
13
кольцо сверху, не ударившись о него? Радиус мяча равен г, радиус кольца R=2r, высота его над полом Н~3 м. Баскетболист бросает мяч с высоты /г=2 м, находясь на расстоянии /=5 м от кольца, считая по горизонтали. Изменением скорости мяча за время пролета через кольцо пренебречь.
1.18°	. Бесконечный конус с углом раствора 90° движется с постоянной скоростью справа налево к центру покоящегося шара. Направление скорости совпадает с осью конуса. Шар разрывается на множество осколков, которые летят во все стороны равномерно с той же по модулю скоростью V. Какая часть осколков попадает на конус? Влиянием силы тяжести пренебречь.
1.19°	. Параллельный пучок одинаковых ядер движется со скоростью V. Ядра в пучке начинают самопроизвольно делиться на две части одинаковой массы. Максимальная скорость осколков, движущихся в направлении пучка, равна и (в неподвижной системе отсчета). Найти скорость осколков, движущихся в направлении, перпендикулярном пучку.
1.20.	Круглое ядро радиуса R, движущееся со скоростью V, пролетает сквозь рой мух, движущихся со скоростью и перпендикулярно направлению полета ядра. Толщина роя равна d, в единице его объема в среднем находится п мух. Сколько мух убьет ядро? Влиянием силы тяжести пренебречь.
1.21°	. Шарик движется между двумя массивными вертикальными стенками, соударяясь с ними. Одна из стенок закреплена, другая удаляется от нее с постоянной скоростью и—50 см/с. Считая движение шарика все время горизонтальным, а удары о стенки — абсолютно упругими, найти его окончательную скорость, если начальная скорость v0 равна: а) 1967 см/с; б) 1917 см/с.
1.22.	Елочное украшение (тонкостенный стеклянный шарик) разбивается при падении на каменный пол с минимальной высоты h (рис. 1.22а). С какой минимальной скоростью urain этот шарик должен налететь на такой же покоившийся до соударения шарик, чтобы оба шарика разбились (рис. 1.226)?
1.23°	. Тяжелая горизонтальная плита движется вниз з постоянной скоростью ©о- С высоты h отпускают (без начальной скорости) шарик, который начинает подпрыгивать на плите. Найти зависимость скорости шарика от времени и построить график этой зависимости. Ускорение свободного падения равно g. Считать удары шарика о плиту упругими.
14
1.24.	На гладкую наклонную плоскость с углом наклона к горизонту а=45° поставили цилиндрический с плоским дном стакан высоты /г=0,1 м (рис. 1.24). В момент начала
а	б
Рис. 1.22
движения стакана от верхнего его края внутрь роняют маленький шарик. Какой путь по наклонной плоскости пройдет стакан к моменту n-го (п=5) удара шарика о дно? Считать удары шарика о дно стакана упругими.
§ 2. Динамика
1.25	°. На тело массы т, вначале покоившееся на горизонтальной плоскости, в течение времени t действует горизонтальная сила F. Коэффициент трения тела о плоскость равен k. Какое расстояние пройдет тело за время движения?
1.26	°. С каким максимальным ускорением может двигаться вверх по наклонной дороге автомобиль, если угол наклона дороги к горизонту равен а, а коэффициент трения между колесами автомобиля и дорогой равен /г? Ускорение свободного падения равно £•*).
1.27	°. Угол наклона ленты подъемника к горизонту а=5°. Коэффициент трения между грузом и лентой й=0,2. При каком максимальном ускорении ленты поднимаемый ящик не будет скользить по ленте подъемника? Лента подъемника не прогибается. Считать g=10 м/с2.
1.28	. Тонкостенный цилиндр катится по горизонтальной плоскости с ускорением а. Брусок А, размеры которого малы по сравнению с радиусом цилиндра, скользит по внутренней поверхности цилиндра так, что угол между радиусом ОА и вертикалью остается постоянным. Найти
*>	Здесь и далее в этой главе в задачах с участием силы тяжести ускорение свободного падения считается известным и обозначается буквой g.
15
этот угол, если коэффициент трения бруска о поверхность цилиндра равен k.
1.29°	. С наклонной плоскости, угол наклона которой равен а, соскальзывает без трения клин. Верхняя грань клина горизонтальна. На клине покоится тело массы т. Найти силу трения, действующую на тело.
□ 1.30. На плоскости, образующей угол а с горизонтом, лежит шайба массы т (рис. 1.30). Какую минимальную
Рис. 1.30
силу надо приложить к шайбе в горизонтальном направлении вдоль плоскости, чтобы она сдвинулась? Коэффициент трения равен k.
it. 1.31°. С наклонной плоскости, угол наклона которой равен а, соскальзывают два груза массы тг и тг, связанные невесомой нерастяжимой нитью (рис. 1.31). Коэффициенты трения между грузами и плоскостью равны соответственно /21 и /г2, причем k2>kL. Найти силу натяжения нити.
1.32°	. Цепочка из п тел одинаковой массы т, соединенных пружинами, движется вдоль горизонтальной плоскости под действием силы F . Найти силу натяжения каждой пружины, если коэффициент трения тел о плоскость равен k. х 1.33°. На тело массы т, расположенное на горизонтальной плоскости, действует сила F, направленная вниз под углом а к горизонту. Сила приложена к центру масс, коэффициент трения равен k. Найти ускорение тела.
1.34.	Найти ускорение тела, если в условии предыдущей задачи сила F направлена вверх под углом а к горизонту, у 1.35. Система грузов, изображенная на рис. 1.35, находится в лифте, который движется вверх с ускорением а. Найти силу натяжения нити, если коэффициент трения между грузом массы и опорой равен k.
1.36.	Найти ускорение цилиндра, скользящего по желобу, угол раствора которого равен 2а. Желоб имеет вид двугранного угла, ребро которого наклонено к горизонту под углом р, а плоскости образуют с горизонтом одинаковые углы. Коэффициент трения между цилиндром и поверхностью желоба равен k.
16
1.37.	Груз массы mt находится на столе, который движется горизонтально с ускорением а (рис. 1.37). К грузу присоединена нить, перекинутая через блок. К другому концу нити подвешен второй груз массы т2. Найти силу
натяжения нити, если коэффициент трения груза массы гщ о стол равен k.
1.38.	Грузик подвешен в точке D на трех одинаковых пружинах, закрепленных на горизонтальной линии в точках А, В, С, причем расстояние АВ равно ВС и равно длине недеформированной пружины (рис. 1.38). В положении равновесия 2/ADB==^/BDC=3(f. Внезапно пружина AD разорвалась. Найти модуль и направление ускорения грузика сразу после разрыва. Массой пружины пренебречь.
Рис.
1.38
Рис, 1.39
1.39.	Через неподвижный блок перекинута нерастяжимая нить, к одному концу которой прикреплен груз массы т0, а к другому в первый раз присоединили пружину с подвешенным к ней грузом массы т, (рис. 1.39а), во второй раз — пружину, имеющую другую жесткость, с подвешен
17
ным к ней грузом массы гп2 (рис. 1.396), а в третий раз последовательно к первой пружине с подвешенным грузом массы m-i присоединили вторую пружину с подвешенным грузом массы т2 (рис. 1.39в). В первый раз удлинение пружины равно во второй раз — равно х2. Каково суммарное удлинение пружин в третий раз? Рассматривать установившееся движение грузов (т. е. в отсутствие колебаний). Массой блока, нити и пружины пренебречь.
1.40*	. Однородный тяжелый канат, подвешенный за один конец, не рвется, если длина каната не превышает значения /0. Пусть тот же канат выскальзывает под действием силы тяжести из горизонтально расположенной трубки с загнутым вниз под прямым углом конпом. При какой максимальной длине канат выскользнет, не порвавшись? Трение отсутствует. Радиусом изгиба трубки пренебречь.
1.41*	. На трехгранную призму, левый угол которой а<45°, а верхний — прямой, падают однородным потоком с одной и той же высоты шарики (рис. 1.41). В какую сторону будет двигаться призма? Учитывать только один удар каждого шарика. Считать удары упругими. Трением пренебречь.
1.42.	1) Цилиндрический сосуд закрыт сверху поршнем массы М, площадь которого равна S. На поршне без потери энергии подпрыгивают шарики массы т каждый	Среднее
число ударов о поршень в единицу времени равно nt, высота подскока каждого шарика равна h, атмосферное давление равно р0. Найти давление газа под поршнем.
2) Решить эту же задачу при условии,- что задано не среднее
число ударов в единицу времени nt, а полное число шариков N.
§ 3. Вращательное движение. Колебания
1.43	*. Через сколько лет повторяется великое противостояние Марса *), если один оборот вокруг Солнца Марс делает за 687 суток?
Рис. 1.41
*) Противостоянием Марса называется момент, когда Солнце, Земля и Марс располагаются на одной прямой по одну сторону от Солнца. Так как орбита Марса довольно сильно вытянута по сравнению с орбитой Земли, то расстояние сближения этих планет в мо-18
”Йг 1.44. С какой угловой скоростью должен вращаться вокруг горизонтальной оси симметрии цилиндр, чтобы мелкие частицы, находящиеся внутри цилиндра, не соскальзывали с его поверхности-1 Коэффициент трения между поверхностью цилиндра и- частицами k=l. Внутренний радиус цилиндра равен г.
1.45	°. Система из двух маятников, в которой точкой подвеса второго маятника служит массивное тело первого маятника (двойной маятник), вращается вокруг вертикальной оси так, что обе нити лежат в одной плоскости и составляют с вертикалью постоянные углы а и р. Массы грузов маятников равны. Длины нитей одинаковы и равны I. Найти угловую скорость вращения двойного маятника.
1.4Д	. Дорога имеет вираж с углом наклона к горизонту а и радиусом закругления г. Какую скорость должен иметь на вираже велосипедист, чтобы не упасть? Трением колес на вираже пренебречь.
1.47	. Спортивный молот —.ядро на тросике, которое бросают, раскрутив вокруг себя с достаточной скоростью. Найти максимальное расстояние, которое может пролететь молот, если удерживающее усилие спортсмена перед моментом броска в п раз превышало силу тяжести, действующую на молот. Расстояние вдоль троса от оси вращения до ядра равно I. Сопротивлением воздуха и начальной высотой траектории ядра пренебречь.
1.48	°. Груз массы т прикреплен двумя одинаковыми пружинами жесткости и к рамке и может двигаться без трения вдоль горизонтального стержня АВ, соединенного с рамкой. Рамку привели во вращение с угловой скоростью со вокруг вертикальной оси 00', лежащей в плоскости рамки на расстоянии I от ее центра (рис. 1.48). Находясь на каком расстоянии от центра рамки, груз может быть неподвижен относительно вращающейся рамки?
1.49	°. Два тела массы и т2 привязаны к нити, закрепленной в некоторой точке, и находятся на гладком столе. Расстояния от них до закрепленной точки равны 1г и /2 соответственно. Система вращается в горизонтальной плоскости вокруг оси, проходящей через закрепленную точку, с угловой скоростью со. Найти силы натяжения, действующие на нить.
1.50	. Мальчик вращает на тросе длины I модель самолета. Рука, держащая трос, описывает окружность радиуса менты противостояний различно. Великим противостоянием Марса называется момент наибольшего его сближения с Землей, в то время как обе эти планеты находятся на одной прямой с Солнцем.
19
г, (рис. 1.50). Самолет движется по окружности радиуса г2, причем Г1+/>Г2, продольная ось самолета расположена по касательной к этой окружности. Найти силу натяжения троса. Масса самолета равна т, угловая скорость равна со.
Траектории движения руки и самолета лежат в горизонтальной плоскости. Подъемная сила направлена вверх.
1.51	. По гладкому столу движутся два тела массы rth и т2, соединенные невесомой нерастяжимой нитью длины I. В некоторый момент времени скорость тела массы mt ока
зывается равной нулю, а скорость тела массы т2 — равной и и направленной перпендикулярно нити. Найти силу натяжения нити.
1.52	. Два шарика массой М каждый скреплены невесомой пружиной жесткости х (рис. 1.52а). К ним через невесомые нити прикреплены еще два шарика массы tn так, что вся система образует квадрат со стороной I, покоящийся на горизонтальном гладком столе. Затем эта связка постепенно раскручивается на столе вокруг ее центра масс до тех пор, пока квадрат не превратится В ромб с углом 60° при вершинах, в которых находятся ща-рики массы М (рис. 1.526). Определить угловую скорость вращения связки.
20
1.53	*. По тонкой проволочной спирали, образующей винтовую линию и стоящей вертикально, скользит нанизанная на проволоку бусинка (рис. 1.53). Радиус спирали равен /?, угол наклона проволоки к горизонту равен а. Найти установившуюся скорость бусинки, если коэффициент трения ее о проволоку равен k.
1.54	*. Найти силу, действующую на вертикальную стенку со стороны опрокидывающейся гантели в Ломент,
когда ось гантели составляет угол а с горизонтом (рис. 1.54). Гантель начинает свое движение из вертикального положения без начальной скорости. Масса каждого шара гантели равна т, расстояние между шарами много больше радиуса шара. Трением пренебречь.
1.55	. Две одинаковые звезды Л и В движутся по окружностям под действием взаимного притяжения на неизменном расстоянии г друг от друга (рис. 1.55). На некотором неизвестном расстоянии от звезд в плоскости их орбиты движется легкая планета С, причем АС=ВС=х, а треугольник АВС сохраняет при движении звезд и планеты свои размеры. Найти расстояние х. Масса планеты С пренебрежимо мала по сравнению с массами звезд А и В.
1.56	*. Две звезды обращаются друг относительно друга С ПОСТОЯННЫМИ ПО модулю скоростями И ©2 и с одним и тем же периодом Т. Найти массы звезд и расстояние между ними. Гравитационная постоянная равна G.
1.57	*. Известно, что спутник, находящийся на орбите, высота которой над поверхностью Земли /г=3,6-104 км, совершает оборот вокруг Земли за одни сутки и может «висеть» над одной и той же точкой экватора. Допустим, что для проведения спортивных передач с Московской олимпиады на такую же высоту запустили спутник, который «завис» над Москвой. Какую силу тяги должен развивать двигатель спутника, чтобы удерживать его на заданной
21
Рис. 1.59
в движение в
орбите? Масса спутника tn~ 1 т, широта Москвы — около 60°, радиус Земли r3=6,4-103 км.
1.58	°. От груза, висящего на пружине жесткости и, отваливается его часть массы т. На какую максимальную высоту поднимется после этого оставшаяся часть груза?
1.59	. Определить период колебаний груза С, шарнирно прикрепленного двумя легкими стержнями длины I к стержню АВ, укрепленному под углом а к горизонту (рис. 1.59). /_ВАС=Трением пренебречь.
1.60	*. Точку подвеса маятника длины I мгновенно приводят изонтальном направлении с посто-
янной скоростью V, затем после перемещения на расстояние х мгновенно останавливают. При какой скорости точки подвеса колебания маятника, возникшие с началом движения, прекращаются сразу же после остановки? Перед началом движения маятник покоился. Угол отклонения маятника от вертикали считать малым.
§ 4. Законы сохранения энергии и импульса.
Работа. Мощность
1.61.	Нить длины I с привязанным к ней шариком массы т отклонили на 90° от вертикали и отпустили. На каком наименьшем расстоянии под точкой подвеса нужно поставить гвоздь, чтобы нить, зацепившись за него, порвалась, если она выдерживает силу натяжения Т?
1.62.	Легкий пластмассовый шарик для игры в настольный теннис роняют с высоты h. В нижней точке его траектории по нему ударяют ракеткой снизу вверх, после чего шарик подпрыгивает на высоту, в п раз большую первоначальной. Определить скорость ракетки в момент удара. Считать удар упругим. Сопротивлением воздуха пренебречь. Масса ракетки много больше массы шарика.
1.63.	По цилиндрической гладкой и жесткой спирали, образующей винтовую линию/' ось которой вертикальна, скользит бусинка массы т. Радиус витка спирали равен г, шаг спирали (расстояние по вертикали между двумя соседними витками) равен h0. С какой силой действует на спираль бусинка в момент, когда она опустилась на рас-22
стояние h (по вертикали)? Бусинка была отпущена без начальной скорости.
1.64.	Два груза массы mi и т2 (тх>т2) соединены нитью, переброшенной через неподвижный блок, и расположены над столом на высоте h. В начальный момент грузы покоятся, затем их отпускают. Какое количество теплоты выделится при ударе груза о стол? Удар абсолютно неупругий, т. е. груз прилипает к столу.
1.65°	. Какое количество теплоты выделится при перевертывании наполовину погруженного в воду свободно
плавающего бруска квадратного сечения из неустойчивого вертикального положения в более устойчивое горизонтальное (рис. 1.65)? Масса бруска т=10 г, длина /=20 см, сечение dxd=lxl см2. Считать, что ускорение свободного падения g=10 м/с2.
1.66.	В цилиндр радиуса R, частично заполненный водой, падает цилиндрическая пробка радиуса г и высоты h (рис. 1.66). Начальная высота нижней поверхности пробки над уровнем воды равна Н, начальная скорость равна нулю. Какое количество теплоты выделится после того, как движение пробки и воды прекратится? Плотность пробки равна р, плотность воды — р0.
1.67.	На глубине Н моря, на дне, стоит бак кубической формы, имеющий высоту h и заполненный водой. Какую минимальную работу надо совершить, чтобы выкачать воду из бака? Плотность воды равна р0.
1.68.	Космический корабль движется к Луне под влиянием ее притяжения. На „большом расстоянии от Луны скорость корабля относительно нее была нулевой. На какой высоте от поверхности Луны должен быть включен тормозной двигатель для осуществления мягкой посадки, если считать, что двигатель создает пятикратную перегрузку (5g)? Ускорение свободного падения на поверхности
23
Луны в п=6 раз меньше, чем на Земле (gn=g/6). Радиус Луны гл«1,7-103 км. Изменением массы корабля при торможении пренебречь. Считать ускорение свободного падения вблизи поверхности Луны постоянным.
1.69.	Тело запускают на полюсе Земли строго по вертикали с первой космической скоростью v1=VgR3 = 8 км/с. На какое максимальное расстояние h от поверхности Земли удалится это тело?
1.70.	Тело запустили вдоль экватора в направлении с востока на запад с такой скоростью, что очень далеко от Земли его скорость стала нулевой. Такое же тело с той же скоростью запустили тоже вдоль экватора, но в направлении с запада на восток. С какой скоростью тело будет двигаться на очень большом расстоянии от Земли? Сопротивлением воздуха пренебречь. Длина экватора 7=4-10 км, период обращения Земли Т=\ сут, радиус Земли г3= =6,4-103 км. Считать, что ускорение свободного падения на Земле g= 10 м/с2.
1.71.	В горизонтальной гладкой трубе имеется кольцевая петля радиуса г (рис. 1.71), расположенная в вертикальной плоскости. С какой минимальной скоростью должен двигаться на горизонтальном участке трубы тонкий
Рис. 1.72
гибкий канат длины />2лг, чтобы пройти через петлю? Считать радиус петли г много большим радиусов трубы и каната.
1.72.	Узкая трубка постоянного сечения образует квадрат со стороной I, закрепленный в вертикальной плоскости (рис. 1.72). Трубка заполнена равными объемами двух не проникающих друг в друга жидкостей с плотностями pi и p2<Pi. Вначале более плотная жидкость заполняла верхнюю часть трубки. В некоторый момент жидкости пришли в движение. Найти их максимальную скорость. Трением пренебречь.
24
1.73°	. На концах невесомой пружины жесткости х имеются маленькие бусинки массы т каждая. Бусинки надеты на жестко закрепленные стержни так, как показано на рис. 1.73. Расстояние между свободными концами стержней /о совпадает с длиной пружины в нерастянутом состоянии. С какой скоростью будет двигаться пружина в направлении оси х после того, как она соскочит со стержней? В начальный момент пружина покоится, а ее длина равна I. Трением и силой тяжести пренебречь.
Рис. 1.74
1.74.	К концам двух нерастянутых пружин, жесткости которых равны Xj и х2 (рис. 1.74), прикреплено тело массы т так, что оно может двигаться вдоль прямой АВ. Конец левой пружины закреплен. Удерживая тело на месте, конец В правой пружины отводят на расстояние I и закрепляют в точке В', после чего тело отпускают. Найти наибольшую скорость тела. Трением и силой тяжести пренебречь.
1.75*	. На груз массы М, висящий на пружине, кладут перегрузок массы т, удерживая груз в первоначальном
ММ
тх/л
Рис. 1.75
nt


Рис. 1.76
положении, а затем его отпускают (рис. 1.75). Найти максимальное значение силы, действующей на перегрузок со стороны груза.
1.76*	. Детский пружинный пистолет, который можно представить себе в виде пружины конечной массы, прикрепленной к неподвижной стенке, выстреливает шариком со
23
скоростью v (рис. 1.76). Если выстрелить шариком вдвое большей массы, то скорость его станет равной V 2/Згл Какова будет скорость, если выстрелить шариком втрое большей массы?
1.77е	. Два одинаковых шарика соединены невесомым стержнем длины 10. Система расположена на горизонтальной плоскости и приведена во вращение так, что ее центр покоится. Сколько оборотов сделает система? Начальная скорость каждого из шариков равна г<0, коэффициент трения о плоскость равен k.
1.78е	. По наклонной плоскости с углом наклона к горизонту Uj с высоты Н без начальной скорости соскальзывает тело. Достигнув точки А, оно начинает подниматься вверх по наклонной плоскости с углом наклона и2 (рис. 1.78). Полагая коэффициенты трения тела о плоскости равными kx и k2, найти высоту подъема тела. Переход с плоскости на плоскость — плавный и гладкий.
Рис. 1.78
Рис. 1.79

1.79*	. Два одинаковых тела массы т каждое, соединенные пружиной жесткости и, лежат на горизонтальной плоскости (рис. Г.79). Левое тело касается вертикальной стенки. Какую минимальную скорость, направленную к стенке, надо сообщить правому телу, чтобы при обратном движении от стенки оно сдвинуло левое тело? Коэффициент трения каждого тела о плоскость равен k. Пружина в начальный момент не деформирована. •
1.80*	. На горизонтальной плоскости лежат два бруска массы nix и т2, соединенные ненапряженной пружиной. Какую наименьшую постоянную силу, направленную горизонтально, нужно приложить к первому бруску, чтобы сдвинулся и второй? Коэффициент трения брусков о плоскость равен k.
1.81.	Под каким углом к горизонту надо направить струю воды из брандспойта, чтобы она падала на расстоянии I от него? Плотность воды равна р0, площадь отверстия— S, мощность мотора — Р, а его КПД — jp Высоту отверстия над землей принять равной нулю.
26
1.82*	. Автомобиль массы т с обеими ведущими осями трогается с места. Двигатель работает с постоянной мощностью Р. Коэффициент трения скольжения колес о дорогу равен k. Найти зависимость скорости автомобиля от времени и начертить качественный график этой зависимости. Сопротивлением воздуха и трением в механизмах пренебречь.
1.83.	Пуля массы т, летевшая с начальной скоростью v, пробивает один подвешенный груз массы т и застревает во втором подвешенном грузе той же массы. Пренебрегая временем взаимодействия пули с грузом, найти количество теплоты Qj, выделившееся в первом грузе, если во втором выделилось количество теплоты Q2-
1.84.	Артиллеристы стреляют так, чтобы ядро попало в неприятельский лагерь. В момент вылета ядра из пушки на него садится верхом барон Мюнхаузен, и потому ядро падает, не долетев до цели. Какую часть пути Мюнхаузену придется пройти пешком, чтобы добраться до вражеского лагеря? Принять, что Мюнхаузен впятеро тяжелее ядра. Посадку барона на ядро считать абсолютно неупругим ударом.
1.85°	. На краю стола высоты h лежит маленький шарик массы тх. В него попадает пуля массы т2, движущаяся горизонтально со скоростью V, направленной в центр шарика. Пуля застревает в шарике. На каком расстоянии от стола по горизонтали шарик упадет на землю?
1.86.	Небольшое тело, двигаясь с горизонтальной скоростью на высоте Н, распалось на две части одинаковой массы. Одна часть упала на землю через время после распада. Через какое время после распада окажется на земле вторая часть, упавшая позднее? Сопротивлением воздуха пренебречь.
1.87.	Тележка массы тг вместе с человеком массы т2 движется со скоростью и. Человек начинает идти с постоянной скоростью по тележке в том же направлении. При какой скорости человека относительно тележки она остановится? Трением колес тележки о землю пренебречь.
1.88.	На покоящееся тело массы mY налетает со скоростью v тело массы тг. Сила, возникающая при взаимодействии тел, линейно растет за время т от нуля до значения Fa, а затем линейно убывает до нуля за то же время т. Определить скорости тел после взаимодействия и количество выделившейся теплоты.
1.89.	В центре незакрепленной сферы радиуса г и массы Л4 разрывается на мелкие осколки снаряд массы т. Осколки
27
разлетаются во все стороны равномерно с одинаковом по модулю скоростью v (рис. 1.89). В сфере имеются два круглых отверстия с центрами, лежащими на одном диаметре. Осколки, не вылетевшие через отверстие, прилипают к поверхности сферы. Найти конечную скорость
О-w2
Рис. 1.90
Рис. 1.91
сферы. Углы а, и а2 считать близкими, но не одинаковыми. Сопротивлением воздуха и силон тяжести пренебречь.
1.90*	. На горизонтальной гладкой плоскости в начальный момент покоится прямоугольная рамка массы mt, длина большей стороны которой равна I (рис. 1.90). Внутри рамки по плоскости со скоростью ч), параллельной длинной стороне, начинает двигаться шарик массы т2, который продолжает движение, ударяясь о середины коротких сторон рамки. Найти время между ударами шарика об одну и ту же короткую сторону. Размерами шарика пренебречь. Считать удары упругими.
1.91*	. На горизонтальную шероховатую ленту ширины I (рис. 1.91), движущуюся со скоростью п, въезжает шайба со
Рис. 1.93
скоростью V, направленной перпендикулярно краю ленты. Шайба съезжает с ленты со скоростью, направленной под углом 45° к краю. Найти коэффициент трения шайбы о ленту.
1.92°	. Два шарика массы и т2 одновременно начинают соскальзывать навстречу друг другу без трения и вращения с двух горок одинаковой формы и высоты Н. При столкновении шарики слипаются. На какую высоту поднимутся слипшиеся шарики?
28
1.93.	Два одинаковых шара массы т покоятся, касаясь друг друга. Третий шар налетает на них, двигаясь по прямой, касающейся обоих шаров (рис. 1.93). Найти массу налетающего шара, если после удара он останавливается Радиусы всех шаров одинаковы. Считать удар упругим.
1.94.	Шар, движущийся со скоростью и, упруго сталкивается с двумя покоящимися шарами, как в предыдущей задаче. Найти скорости шаров после столкновения. Шары имеют одинаковые массы и радиусы.
1.95*	. Два упругих гладких шара одновременно вылетают из вершин А и В равностороннего треугольника в направлении третьей его вершины С с одинаковыми по модулю скоростями. Масса шара А втрое больше массы шара В, а радиусы шаров одинаковы. Каким будет угол между скоростями шаров после удара?
1.96*	. Два одинаковых гладких упругих шарика А и В (рис. 1.96) движутся во встречных направлениях со скоростями v и 2v, причем прямые, проходящие через центры каждого из шариков в направлении их движения, касаются другого шарика. Найти, под каким углом к первоначальному направлению будет двигаться шарик А после соударения.
1.97.	На гладком горизонтальном столе вдоль одной прямой лежат, не соприкасаясь, «==1969 шаров, радиусы которых одинаковы, а массы равны т, пг/2, т/4, . . ., т]2п~1. На первый шар налетает со скоростью, параллельной той же прямой, шар массы 2m. Найти скорость, которую приобретает последний шар. Считать удары упругими и лобовыми.
1.98.	Два одинаковых шара массы т каждый связаны прочной нитью (рис. 1.98). Доска массы М~2т налетает
Рис. 1.98	Рис. 1.99
со скоростью и~\ м/с на эту систему и ударяет по середине нити. Найти скорости шаров при ударе о доску.
1.99*	. Стержень АВС (рис. 1.99), на котором закреплены два одинаковых грузика В и С, подвешен за точку А к
29
очень длинной нити ОА (ОА^-АС). Стержень удерживали в горизонтальном положении, при этом нить О А была вертикальной, а затем отпустили. Какой будет скорость точки А в момент, когда стержень будет проходить низшее положение? Массой стержня и нити пренебречь. АВ='1, ВС—21.
1.100*	. Пара одинаковых грузиков А и В, связанных нитью длины I (рис. 1.100), начинает соскальзывать с гладкого стола высоты I, причем в этот момент грузик В находится на высоте ВС—21/3. Достигнув пола, грузик В прилипает к нему, сразу после чего грузик А слетает со стола. На какой высоте у над уровнем пола окажется грузик А, когда нить вновь станет натянутой?
А

Рис. 1.100	Рис. 1.101
1.101*	. Прямоугольный брусок массы Л4 с полусферической выемкой радиуса г=0,2 м стоит вплотную к вертикальной стенке на горизонтальной поверхности (рис. 1.101). С какой максимальной высоты над ближайшей к стенке верхней точкой А края выемки надо отпустить маленький шарик массы т—М/5, чтобы он не поднялся над противоположной точкой В выемки? Трением пренебречь.
1.102*	. Изогнутая под углом 2а узкая трубка АКВ неподвижно закреплена на тележке так, что каждое колено ее составляет угол а с вертикалью (рис. 1.102). Половина трубки заполнена водой, удерживаемой заслонкой К. Тележка может двигаться по горизонтальной плоскости. В некоторый момент заслонку К открывают. Найти скорость тележки в тот момент, когда середина столба воды проходит самое нижнее положение. Начальные скорости равны нулю. Масса тележки с пустой трубкой равна М, масса воды равна т, АК=ВК=1. Трением пренебречь.
1.103*	. Внутри U-образной трубки массы М, лежащей на столе, находится нерастяжимая нить массы т (рис. 1.103). В начальный момент в каждом колене трубки находится по половине нити, а сама трубка движется. Нить в трубке
30
движется так, что скорость конца А нити равна а скорость конца В — нулю. С какой скоростью будет двигаться трубка, когда нить вылетит из нее? Трением пренебречь. Радиус изгиба трубки считать очень малым.
1.104*	. В вертикальном цилиндре под поршнем массы М прыгают, упруго ударяясь о поршень и дно цилиндра, п шариков (п>1) массы т каждый. Система находится в равновесии. Высота поршня над дном равна Л. На какую
Рис. 1.102	Рис. 1.103
высоту будут подскакивать шарики, если поршень быстро убрать? Трением поршня о стенки цилиндра и атмосферным давлением пренебречь.
1.105*	. Гладкий клин массы М может скользить по горизонтальной плоскости. На его грань, образующую угол а с горизонтом, положен гладкий брусок массы т. Найти ускорение клина. Трением пренебречь.
§ 5. Статика. Момент сил
1.106	°. Шар лежит в щели АВС, образованной двумя плоскими стенками, причем ребро щели горизонтально (рис. 1.106). Найти угол между плоскостями, если сила давления шара па вертикальную стенку ВС вдвое превышает силу тяжести, действующую на шар. Трением пренебречь.
1.107	°. Шар радиуса г и массы т подвешен на нити длины I, закрепленной на вертикальной стенке. Найти силу, с которой шар действует на стенку. Трением пренебречь.
1.108	°. Шарик радиуса г и массы т удерживается на неподвижном шаре радиуса R невесомой нерастяжимой нитью длины /, закрепленной в верхней точке С шара (рис. 1.108). Других точек соприкосновения между нитью
31
и шаром нет. Найти силу натяжения нити. Трением пренебречь.
1.109	. 1) Два шарика масс пц и т2, соединенные невесомым жестким стержнем длины /, покоятся в сферической полости радиуса R. Под каким углом а к горизонту расположится стержень? Трением пренебречь.
2)	В гладкой закрепленной полусфере свободно лежит стержень массы т, так что его угол с горизонтом равен а, а один конец выходит за край полусферы. Найти силы, с которыми действует стержень на полусферу в точках соприкосновения с ней.
1.110	°. Динамометр D поставили между двумя отрезками троса АВС и ЛЕС так, что он находится на диагонали BE ромба (рис. 1.110). Точка С закреплена. Найти
силу, которая приложена к точке А, если динамометр показывает силу Fo. ^ЕАВ—^ВСЕ = а.
1.111	°. Цилиндр радиуса г, лежащий на подставке, разрезан пополам по вертикальной плоскости, проходящей через его ось. Масса каждой половины цилиндра равна т, а их центры тяжести находятся на расстоянии I от оси цилиндра. Чтобы цилиндр не распался, через него переки-
32
нули невесомую нерастяжимую нить с одинаковыми грузами на концах (рис. 1.111). Найти минимальную массу грузов, не допускающих распада цилиндра. Трением, пренебречь.
1.112	. Однородная нить массы т свободно висит так, что оба ее конца закреплены и находятся на одной высоте. Сила натяжения нити в нижней точке равна То. Найти силу натяжения нити вблизи точек подвеса.
1.113	. Концы нерастяжимой невесомой нити длины L, на которую надета тяжелая бусинка С, закреплены в точках А и В, находящихся на разных уровнях (рис. 1.113).
Пренебрегая размерами бусинки и трением, найти расстояние от точки А до вертикали, проходящей через бусинку. Параметры I и h, указанные на рисунке, считать известными.
1.114	*. Три нерастяжимых нити одинаковой длины,, прикрепленные к кольцу 1 (рис. 1.114) на одинаковых расстояниях друг от друга и к кольцу 3 аналогичным образом, пропущены внутрь кольца 2, как показано на рисунке. Радиусы колец 1 и 2 одинаковы и равны г, радиус кольца 3 в два раза больше. Все кольца сделаны из одной и той же проволоки. Кольцо 1 удерживают в горизонтальной плоскости, система находится в равновесии. Найти расстояние между центрами колец 2 и 3. Трением пренебречь.
1.115	. Ракета с конической носовой частью движется в пылевом облаке с постоянной скоростью с, направленной вдоль ее оси. Плотность облака равна р. Площадь поперечного сечения ракеты равна 5, угол раствора конической части 2а. Найти силу тяги, развиваемую двигателем ракеты. Столкновения пылинок с корпусом ракеты считать упругими.
2 Г. В. Меледив
83
1.116.	Ракета равномерно движется сквозь разреженное облако пыли. Во сколько раз нужно увеличить силу тяги,.чтобы скорость ракеты стала вдвое больше?
1.117*	. Два шара, сделанные из одного материала, падают на землю, замедляясь в разреженном облаке пыли. Во сколько раз установившаяся скорость одного шара больше скорости другого, если радиус его вдвое больше? Учитывать только сопротивление, оказываемое пылинками (а не воздухом).
1.118.	Тяжелый прут согнули в середине под углом 90° и подвесили свободно за один из концов. Какой угол с вертикалью образует прикрепленный конец?
1.119.	Однородный стержень массы т одним концом опирается на горизонтальную, другим — на наклонную плоскость. Угол между плоскостями равен а. Какую силу, направленную вдоль наклонной плоскости, надо приложить к одному из концов стержня, чтобы он находился в равновесии? Трением пренебречь.
1.120°	. В вертикальную стенку вбиты два гвоздя А и В, на которые сверху опирается стоящий у стены гладкий круглый обруч массы т с центром в точке О (рис. 1.120). Найти силы, действующие на каждый из гвоздей, если радиусы ОА и ОВ составляют с вертикалью ОС углы а и р соответственно. Трением пренебречь.
Рис. 1.120
Рис. 1.121
1.121°	. Гладкий круглый тонкий обруч массы т удерживается у стенки в вертикальном положении с помощью двух гвоздей. Первый из них расположен внутри обруча (точка Л) и касается его так, что радиус, проведенный в точку касания А, составляет с вертикалью угол АОС=а (рис. 1.121). Второй гвоздь находится вне обруча (точка В). Соответствующий угол ВОС—&. Найти, с какой силой обруч действует на каждый гвоздь. Трением пренебречь.
1.122.	Два гладких шара масс ащ и т2 подвешены на невесомой нерастяжимой нити к одной и той же точке О, как показано на рис. 1.122. Радиус большего шара равен г, 34
Длина нити, его удерживающей, равна I. Какой угол эта нйть образует с вертикалью?
1.123°	. Грузы неизвестных масс mt и т2 соединены Короткой невесомой и нерастяжимой нитью длины I и раз-ьЙЙдены на гладкой цилиндрической поверхности радиуса г
Рис. 1.122
(рис. 1.123). При равновесии угол между вертикалью и радиусом, проведенным в точку, где находится груз массы mi, равен а. Найти отношение масс грузов.
1.124.	Однородная балка массы М и длины L подвешена за концы на двух пружинах. Обе пружины в ненагружен-ном состоянии имеют одинаковую длину, но при действии одинаковой нагрузки удлинение правой пружины в п раз больше, чем удлинение левой. На каком расстоянии I от левого конца балки надо положить груз массы т, чтобы балка приняла горизонтальное положение?
1.125.	Чтобы сдвинуть контейнер влево, к центру его правой стороны, перпендикулярно ей, необходимо приложить силу Fi—102 Н, а чтобы сдвинуть его вправо, нужно
9
Рис. 1.125
приложить к центру левой стороны, перпендикулярно ей, силу F2 —1,5-102 Н (рис. 1.125). Найти массу контейнера. Левые опоры, в отличие от правых, сделаны на роликах, обеспечивающих пренебрежимо малое трение. Размеры опор малы. Контейнер считать однородным кубом.
1.126*	. Колесо снабжено тормозом ВС (рис. 1.126), который шарнирно закреплен в точке В на той же жесткой 2*
35
раме, что и ось колеса А. Растянутая пружина AG прижи» мает тормозную колодку С к обдду колеса так, что угол АСВ равен а. При вращении колеса по часовой стрелке сила трения, создаваемая колодкой тормоза, равна F±. Коэффициент трения колодки об обод равен k. Найти силу трения при вращении колеса против часовой стрелки.
1.127.	На горизонтальной поверхности стоит куб (рис. 1.127). С какой минимальной силой и под каким углом к горизонту а нужно тянуть куб за верхнее ребро, чтобы он опрокинулся без проскальзывания, если коэффициент трения равен k, а масса куба равна т?
V/7777777777777777)
Рис. 1.127
1.128.	Пластина, изготовленная из двух квадратных пластин одинакового размера и массы, находится на горизонтальном столе. К точке А пластины (рис. 1.128) прикреплена нить, за которую тянут в горизонтальном направлении с постоянной скоростью. Найти угол а между линией соединения квадратов АВ и нитью, если коэффициенты трения квадратов о стол равны /г1=0,6 и /г2—0,4. Пластина движется поступательно.
1.129*	. Опорами однородного бруска массы т являются два полуцилиндра, левый из которых имеет коэффициент
Рис. 1.129
трения о стол, равный kj, а правый — k2 (рис. 1.129). Какую горизонтальную силу F надо приложить к бруску, чтобы он двигался равномерно? Размерами опор прене-
36
бречь. Параметры I и d, указанные на рисунке считать Известными.	’
1.130*	. Небольшой груз массы т закреплен посередине невесомой тележки высоты h. Расстояния от него до обеих осей тележки равны I. Тележка катится по наклонной плоскости с углом при основании а (рис. 1.130). В некоторый момент с помощью тормозных колодок мгновенно
Рис. 1.131
останавливают вращение колес тележки. Коэффициент трения скольжения передних колес о плоскость равен klt задних — k2- При каком угле а тележка начнет двигаться равномерно?
1.131.	На ось радиуса г с жестко закрепленными на ней колесами радиуса R давит пресс с силой F (рис. 1.131). Коэффициент трения между осью и прессом, а также между колесами и горизонтальной плоскостью равен k. Ось тянут за трос ОА. Какую работу нужно совершить, чтобы сдвинуть ось на расстояние 1<R? Силой тяжести пренебречь.
1.132.	На катушку, лежащую на горизонтальном столе, давит клин, который может смещаться только вдоль вертикальной плоскости (рис. 1.132). Коэффициент трения катушки о клин и стол равен k. При каком значении коэффициента k катушка сдвинется влево, вращаясь против часовой стрелки? Силой тяжести, действующей на катушку, пренебречь.
1.133.	У стенки, прижимаясь к ней, лежит катушка массы т радиуса 2R, на внутренний цилиндр которой намотана нить (рис. 1.133). За нить тянут вертикально вниз. При каком значении силы натяжения нити F катушка начнет вращаться? Коэффициенты трения о пол и стенку одинаковы и равны k, радиус внутреннего цилиндра равен R.
1.134.	Между двумя одинаковыми досками массы М каждая, шарнирно закрепленными в точке А, удержи-
37
вается шар массы т (рис. 1.134). Точка касания доски и шара находится посередине доски. .Угол между досками равен 2а. При каком минимальном значении коэффициента трения это возможно?
1.135*	. На горизонтальном столе находится лист бумаги, прижатый однородным стержнем массы т, верхний
Рис. 1.136
конец которого шарнирно закреплен. Какую минимальную горизонтальную силу необходимо приложить к листу, чтобы вытащить его? Угол между стержнем и листом равен а, коэффициент трения между ними равен k. Трением между столом и бумагой пренебречь.
1.136*	. Жесткий стержень длины I может свободно поворачиваться вокруг оси О, закрепленной на расстоянии I от гладкой вертикальной стенки (рис/ 1.136). Между стержнем и стенкой зажат брусок толщины h. При какой толщине бруска его невозможно протянуть вниз, если коэффициент трения между стержнем и бруском равен /г?
1.137*	. Два столба высоты Н каждый вкопаны в землю на небольшую глубину так, что вытащить столб можно, приложив вертикальную силу Fo. Чтобы повысить устойчивость, к столбам на высоте h шарнирно прикреплены жесткие подпорки, образующие угол а с вертикалью (рис. 1.137). С какой максимальной силой можно натянуть трос между верхушками столбов? Изменением вертикальной силы F<ZF0, действующей на столбы при натяжении троса, пренебречь.
1.138.	Брусок длины I и массы т подвешен на двух параллельных невесомых жестких стержнях, соединенных шарниром с перекладиной длины L, стоящей на опорах (рис. 1.138). Правый стержень находится на расстоянии d 38
от правого конца перекладины. Брусок начинает движение из наивысшего положения без начальной скорости. Найти максимальную разность сид, действующих на правую и левую опоры перекладины. Прогибом перекладины и трением пренебречь.
1.139.	На краю стола лежит однородный брусок массы М. и длины L, выступающий за край на расстояние d. Брусок закреплен на шарнире и может вращаться вокруг края стола. К концу бруска подвешен на нити длины I груз массы т. Груз отклонили на некоторый угол а и затем отпустили. При каком угле а брусок начнет поворачиваться? 0° < а < 90°.
1.140*	. На верхнюю точку закрепленного шара поставлена игрушка «ванька-встанька». Нижняя поверхность игрушки — полушар такого же радиуса, а центр тяжести ее — точка С, расположен на половине вертикального радиуса полушара. Упадет ли «ванька-встанька» с шара? Проскальзывания нет.
§	6. Механика жидкости и газа
1.141	°. Из шланга, наклоненного под углом а к горизонту, бьет со скоростью Ч) вода. Найти массу воды, находящейся в данный момент в воздухе, если площадь сечения отверстия шланга равна 5, высота его над землей равна h, плотность воды равна р0.
1.142	. Половник, наполненный тяжелой жидкостью, аккуратно подвесили за конец ручки (рис. 1.142). Часть жидкости при этом вылилась. Найти объем жидкости, оставшейся в половнике, если черпалка половника имеет
39
форму полусферы радиуса г, а ручка, касательная к полу, сфере, имеет длину I = К8г. Массой половника пренебречь.
1.143	*. На тележке стоит сосуд с высокими стенками и квадратным дном, имеющим сторону I. Нижнее ребро сосуда шарнирно закреплено. В сосуд налита жидкость
Рис. 1.142
Рис. 1.144
до уровня /г> 1/2. Тележку тянут с ускорением а, придерживая сосуд. Когда поверхность жидкости успокаивается, сосуд отпускают. При какой минимальной высоте уровня жидкости сосуд опрокинется? Массой сосуда пренебречь.
1.144	°. В цилиндр сечения St налита несжимаемая жидкость, поверх которой помещен поршень (рис. 1.144). Внутри этого поршня имеется цилиндрическая вставка сечения S2. Сила трения между поршнем и вставкой может достигать значения F; между поршнем и стенками цилиндра трения нет. С какой минимальной силой нужно надавить сверху на вставку, чтобы выдавить ее из поршня? Силой тяжести пренебречь.
1.145	°. Поршень вытесняет воду из вертикального цилиндрического сосуда через малое отверстие, находящееся у дна сосуда и имеющее площадь So. Высота сосуда равна h, площадь основания — S. Какую работу совершает поршень, если он движется с постоянной скоростью с? Учесть действие силы тяжести.
1.146	°. Тонкостенный стакан массы т, расположенный вертикально вниз дном, плавает на границе раздела двух жидкостей с плотностями pt и р2 (рис. 1.146). Найти глубину погружения стакана в нижнюю жидкость, если дно стакана имеет толщину h и площадь S. Массой стенок стакана пренебречь.
1.147	°. Цилиндрический сосуд заполнен двумя непере-мешивающимися жидкостями с плотностями pi и р2. В жид-40
кость погружается куб с длиной ребра I. Найти глубину погружения куба в жидкость с плотностью р2, если плотность вещества куба равна р (p2>p>Pi).
1.148	. На дне сосуда, наполненного жидкостью, плотность которой равна р, стоит Г-образное тело (размеры указаны на рис. 1.148). Жидкость под нижнюю грань тела
не подтекает. Плотность тела равна 2р. При какой высоте уровня жидкости в сосуде равновесие тела нарушается?
1.149	. В стенке цилиндрического сосуда радиуса /?, наполненного водой до высоты h, возле дна имеется отверстие, закрытое пробкой. Какую работу нужно совершить, чтобы вдвинуть пробку в сосуд на длину /? Пробка имеет форму цилиндра радиуса г и длины, большей I. Трение не учитывать. Плотность воды равна р0. Сосуд достаточно высок, так что вода из него не выливается.
1.150	. В прямоугольный высокий сосуд налита жидкость плотности р. В одной из стенок у дна сосуда имеется прямоугольное отверстие высоты h, в которое вдвинута на расстояние I невесомая пробка того же сечения. Между пробкой и дном сосуда жидкость не проникает. При какой высоте уровня жидкости над пробкой жидкость не сможет ее вытолкнуть? Коэффициент трения пробки о дно сосуда равен k. Атмосферное давление равно р0. Трением пробки о стенки сосуда пренебречь.
1.151	. В сосуде, наполненном водой плотности р0, на дне, представляющем собой наклонную плоскость, стоит металлический кубик плотности р. Найти силу давления кубика на дно. Расстояние от верхнего ребра кубика до поверхности воды равно Л, угол наклона дна к горизонту равен а. Между дном и нижней гранью кубика вода не проникает. Ребро, кубика равно I.
41
1.152	°. На дне сосуда на одной из своих боковых граций лежит треугольная призма. В сосуд налили жидкость плотности ро, причем ее уровень сравнялся с верхним реб-, ром призмы. Какова плотность материала призмы, если сила давления призмы на дно сосуда увеличилась в три раза? Жидкость под призму не подтекает.
1.153	. Одна из стенок прямоугольного сосуда с водой образована бруском. Брусок представляет собой призму, в плоскостях боковых сторон сосуда имеющую сечение в виде равнобедренного прямоугольного треугольника, и может перемещаться по дну сосуда (рис. 1.153). Считая,
что трение между бруском и боковыми стенками отсутствует, найти минимальный коэффициент трения между основанием сосуда и бруском, при котором брусок придет в движение. Длина бруска 7=20 см, его масса т=90 г, угол при вершине призмы а=45°, высота столба воды h—i см, плотность воды ро=1'1О3 кг/м3.
1.154	. Круглое отверстие в дне сосуда закрыто конической пробкой с сечением основания S (рис. 1.154). При какой наибольшей плотности материала пробки р можно, доливая воду, добиться всплытия пробки? Площадь отверстия равна So, плотность воды равна р0. Поверхностным натяжением пренебречь. Объем конуса, имеющего площадь основания S и высоту h, равен AS/3.
1.155	*. 1) Трубка радиуса г, закрытая снизу алюминиевой пластинкой, имеющей форму цилиндра радиуса R и высоты h, погружена в воду на глубину Н (рис. 1.155). Расстояние между_ осями трубки и пластинки равно d. Давление воды прижимает пластинку к трубке. До какой высоты следует налит* воду в трубку, чтобы пластинка отделилась от трубки? Плотность воды равна р0, алюминия — р.
2)	Алюминиевая пластинка заменена деревянной. До какой высоты следует налить в трубку воду, чтобы пластинка всплыла? Плотность дерева равна р.
42
1.156	*. 1) Трубка радиуса г, закрытая снизу алюминиевой клиновидной пластинкой, в сечении образующей прямоугольный треугольник с катетами а и Ь, погружена в воду на глубину Н (рис. 1.156). Верхняя грань клина представляет собой квадрат со стороной а, причем ось трубки проходит через середину квадрата. Давление воды
прижимает клин к трубке. До какой высоты следует налить воду в трубку, чтобы клин отделился от нее? Плотность воды равна р0, плотность алюминия — р.
2)	Алюминиевый клин заменен деревянным. До какой высоты следует налить воду в трубку, чтобы клин всплыл? Плотность дерева равна р.
1.157	°. В U-образную трубку, имеющую сечение S, налита жидкость плотности р. На сколько поднимется уровень жидкости в правом колене трубки по отношению к первоначальному уровню, если в левое колено опустили тело массы т и плотности, меньшей плотности жидкости, которое может свободно в нем плавать? Капиллярными явлениями пренебречь.
1.158	*. В U-образной трубке с соприкасающимися
внутренними стенками в равновесии находятся жидкости
с плотностями Pi и р2 (р»<р2), так что граница раздела между ними проходит через центр дна трубки (рис. 1.158). На высоте /г0 от нижней точки трубки во внутренних стенках появляется небольшое отверстие и начинается перетекание жидкости. На сколько изменится
после перетекания уровень в колене Рис } }58 с жидкостью, имеющей плотность рг?
Считать трубку тонкой и перемешивание жидкостей невозможным (возможен только разрыв столба жидкости в месте появления отверстия).
43
1.159	. Ртуть в барометре заменили на сжимаемую жидкость, плотность которой в результате действия силы тяжести зависит от глубины h по закону p=Po(l+a/i). Какова будет высота столба этой жидкости при атмосферном давлении р?
1.160	. В цилиндрический сосуд, площадь основания которого равна S, налита сжимаемая жидкость массы М, плотность р которой в результате действия силы тяжести зависит от глубины h по закону р=р0(14-а/г). На сколько изменится высота столба жидкости, если в нее опустить кубик массы т, который при этом не ложится на дно (т. е. плавает в погруженном состоянии)?
1.161	°. Определить силу натяжения вертикального троса, медленно вытягивающего конец бревна массы т= =240 кг из воды, если бревно при вытягивании остается затопленным наполовину (т. е. происходит лишь разворот бревна вокруг его центра).
1.162	°. Шар, до половины погруженный в воду, лежит на дне сосуда и давит на него с силой, равной трети действующей на него силы тяжести. Найти плотность шара.
1.163	°. К коромыслу равноплечих весов подвешены два груза одинаковой массы. Если каждый из грузов поместить в жидкости с плотностями pi и р2, то равновесие сохранится (рис. 1.163). Найти отношение плотностей грузов.
О
Рис. 1.163	Рис. 1.165
1.164	°. Два шара одинакового объема, но разной плотности закреплены на концах стержня, шарнирно подвешенного в центре. Шарнир находится на поверхности воды, пр» этом один шар погружается в воду на три четверти своего объема^ а другой — на одну четверть. Найти плотность более тяжелого шара р2, если плотность легкого шара равна рх. Плотность воды равна р0.
1.165	°. На горизонтальном дне бассейна под водой лежит невесомый шар радиуса г с тонкой тяжелой ручкой длины I, опирающейся о дно (рис. 1.165). Найти наименьшую массу ручки, при которой шар еще лежит на дне. Плотность жидкости равна р0.
44
L166°. Шар массы т и радиуса г полностью погружен в жидкость и прикреплен к гладаЙй вертикальной стене о помощью нити длины /=г. Если шар отпустить, он будет плавать, наполовину погрузившись в жидкость. С какой силой прикрепленный шар давит на вертикальную стенку?
1.167	. С какой силой Давит на етенку цилиндрического стакана палочка массы т, наполовину погруженная $ Воду и не достающая до дна? Угол наклона палочки к горизонту равен а. Трением пренебречь.
1.168	°. Треугольная призма, объем которой равен V, а плотность материала pi, погружена в жидкость плотности p2>pt (рис. 1.168). Призма всплывает с постоянной скоростью, скользя по тонкому слою жидкости вдоль стенки, угол наклона к горизонту которой равен углу а призмы при вершине. Найти силу сопротивления движению. Угол при основании призмы равен 90°.
Рис. 1.169
1.169	. Два шарика одинакового радиуса /?=1 см, один из алюминия, другой деревянный, соединенные длинной нитью, медленно тонут в воде, двигаясь с постоянной скоростью (рис. 1.169). Найти силу сопротивления воды, действующую на каждый из шариков. Плотность алюминия р1=2,7-108 кг/м3, дерева р2=0,5-103 кг/м8, воды ро=Ь1О8 кг/м?. Считать, что ускорение свободного падения 10 м/с2.
1.170	°. Кубик плотности pi удерживается в равновесии невесомой пружиной под наклонной стенкой, угол наклона которой равен а, в жидкости плотности р2>Р1. Между стенкой и кубиком — тонкий слой жидкости. Найти длину пружины, если в ненагруженном состоянии ее длина равна /0, а в нагруженном состоянии, когда кубик подвешен к пружине в отсутствие жидкости, ее длина равна I (рис. 1.170).
43
1.171	*. Заполненный водой цилиндрический сосуд радиуса R вращается вокруг своей оси с угловой скоростью и. На дне его лежит шар радиуса r<R/2 и плотности р. С какой силой шар давит на боковую стенку цилиндра? Ось цилиндра вертикальна, плотность воды равна р0.
1.172	*. Закрытый цилиндрический сосуд, наполненный на три четверти объема водой, вращается вокруг своей оси так, что в его центре образуется цилиндрическая полость. В воде плавает, погрузившись на две трети своей длины I, тонкий стержень АВ (рис. 1.172). Найти плотность стержня. Радиус цилиндра равен R, длина стержня K3R/4, плотность воды равна р0. Силой тяжести пренебречь.
1.173	. На поверхности воды плавает деревянный брусок квадратного сечения, имеющий плотность р=0,5- 103 кг/м8. Какое из двух положений равновесия, показанных на рис. 1.173, будет устойчивым и почему?
Рис. 1.172
1.174	*. Цилиндрический стакан до высоты h заполнен кусочками льда. Поры между кусочками льда сквозные и в исходном состоянии заполнены воздухом. Льдинки занимают долю объема а=60 %. Лед начинает таять, причем соотношение объемов льдинок и пор между ними остается неизменным. Найти высоту уровня воды в стакане в момент, когда растаяла доля льда (1=70 %. Плотность сплошного льда р=0,9Л03 кг/м8, плотность воды р0== ==!• 103 кг/м3.
1.175	. Трубка, диаметр которой много меньше длины, свернута в кольцо радиуса г и заполнена водой вся, за исключением небольшого участка в нижней точке трубки 46
(точка Л), заполненного маслом (рис. 1.175). Плоскость кольца вертикальна. В начальный момент масляная пробка начинает всплывать в направлении к точке В. Найти ее скорость в момент, когда она проходит мимо точки В. ^/АОВ= 135°. Плотность масла равна р, плотность воды
Рис. 1.173
Рис. 1.175
р0. Длина масляной пробки Трением о стенки трубки пренебречь. Просачивание воды сквозь пробку отсутствует.
1.176	. В U-образной трубке постоянного сечения колеблется жидкость плотности р. Жидкость занимает участок трубки длины I. Найти давление на глубине Н в вертикальном участке правого колена в момент, когда уровень жидкости в правом колене выше, чем в левом, на величину h.
1.177	". Струя воды в фонтане поднимается на высоту Н над уровнем выходной трубы нас трубе подсоединяют вертикальную трубу такого же диаметра, имеющую высоту h<H (рис. 1.177). Во сколько раз следует изменить после подсоединения дополнительной трубы мощность насоса, чтобы суммарная высота подсоединенной трубы и вылетающей из нее струи воды осталась равной Я? Потерями энергии воды на трение о стенки труб пренебречь.
1.178	. Брусок массы т удерживается в воздухе п струями воды, бьющими вертикально вверх из отверстий, имеющих сечение S. Скорость воды на выходе из отверстий равна v. Достигая бруска, вода разлетается от него в горизонтальной плоскости. На какой высоте над отверстиями удерживается брусок? Плотность воды равна р0.
1.179	*. В вертикальной трубе радиуса R, заполненной жидкостью плотности р1( вдоль оси трубы всплывает круглый стержень радиуса г и длины /, причем /^>7? (рис. 1.179). Плотность стержня p2<Pi. Пренебрегая концевыми эффек-
К этой выходной
Рис. 1.177
47
тами и трением, найти скорость и ускорение стержня в зависимости от пройденного расстояния h, если в начальный момент времени он покоился.
1.180	*. В короткой трубке переменного сечения поддерживается неизменное во времени течение вязкой несжимаемой жидкости плотности р. В сечениях / и ~;+—1 II, площади которых равны S, и 32 соответ-fljly ственно, скорость жидкости можно, считать по-Н j	ZF	стоянной по сечению. Найти	силу, с которой
£ I	£	жидкость действует на участок трубы между
£ [	£	сечениями I и II, и количество	теплоты, которое
£ !	£	выделяется в единицу времени	в объеме трубы
£|£ между этими сечениями. Давление и скорость £ ]£- жидкости в сечении I равны рг и щ; в сечении Нд-£ II давление равно р2.
1.181	*. Труба радиуса г заполнена порис-Рис. 1.179 той средой плотности рх. Невесомый поршень, на который действует сила F, вдвигаясь в трубу, уплотняет среду до плотности р2. С какой скоростью движется поршень, если уплотнение происходит скачком, т. е. по среде с некоторой скоростью перемещается поверхность 5, по одну сторону от которой плотность среды равна ръ а по другую — равна р2? В начальный момент поверхность 5 совпадает с поршнем.
1.182	. В U-образную трубку налиты две жидкости различных плотностей. Линия раздела жидкости проходит через нижнюю точку трубки. Высота столба жидкости в правом колене равна hi. Найти максимальную скорость жидкости, если в начальный момент уровень жидкости, находящейся в правом колене, был опущен относительно равновесного на расстояние h0. Трением пренебречь.
Глава II. ТЕПЛОВЫЕ ЯВЛЕНИЯ
§ 1. Газовые законы. Тепловое расширение
2.1°	. Основаниями цилиндрического сосуда, из которого откачан воздух, являются две крышки площади 5, каждую из которых тянут 16 лощадей. Каково ускорение лошадей в момент отрыва крышки? Атмосферное давление равно р0. Масса каждой лошади равна т.
2.2.	Цилиндрический сосуд с тонкими двойными стенками наполнили до краев жидкостью плотности р (рис. 2.2) 48
Высота сосуда4 равна Н, площадь дна равна 5, площадь сечения внутреннего цилиндра равна 5/2. Между внутренним цилиндром и дном имеется узкая щель. Найти значение атмосферного давления, если масса жидкости в сосуде равна т. Стенки сосуда хорошо проводят тепло. Давление насыщенных паров жидкости мало по сравнению с атмосферным.
2.3.	Тяжелый поршень массы т вставляют в открытый сверху стоящий вертикально цилиндрический сосуд, площадь сечения которого 5 равна площади поршня, и отпускают. Найти давление в сосуде в момент, когда скорость
Рис. 2.2
Рис. 2.6
поршня максимальна. Атмосферное давление равно р0. Трением пренебречь.
2.4.	Две полусферы соединены в сферу, из которой частично откачан воздух, и подвешены в точке полюса. Полусферы оторвутся друг от друга, если к точке противоположного полюса подвесить груз массы М. До какой температуры нужно нагреть воздух внутри сферы, чтобы она развалилась без дополнительного груза? Масса каждой полусферы равна т, наружный радиус равен г, атмосферное давление равно р0, начальная температура воздуха и сферы равна То. Тепловым расширением сферы пренебречь.
2.5°	. В цилиндрический открытый сосуд налита неис-паряющаяся жидкость. При температуре /0=О °C давление на дно сосуда равно р0. Найти давление на дно сосуда при температуре t, если температурный коэффициент линейного расширения материала сосуда равен а. Уровень жидкости остается все время ниже края сосуда.
2.6.	Центральный стержень С и внешняя оболочка А эталона длины (рис. 2.6) имеют температурный коэффициент линейного расширения щ. Их длины при 20 °C одинаковы и равны I. Температурный коэффициент линейного расширения внутренней трубы В равен а2. Какой должна
49
быть длина L внутренней трубы В при 20 °C, чтобы тепловое расширение не меняло полной длины эталона ED7
2.7*	. На крыше, наклоненной к горизонту под углом <р=30°, лежит свинцовый лист массы т. Коэффициент трения свинца о крышу &=0,7 (fc>tg <р). Длина листа при температуре ^=10 °C /=1 м. Считая, что температура листа в течение суток повышается, достигая наивысшего значения 72=20 °C, а потом понижается до значения определить положение точек листа, неподвижных в течение суток при нагревании и при остывании. Найти расстояние. на которое сползет лист за п=30 сут устойчивой погоды. Температурный коэффициент линейного расширения свинца а=3- ИГ5 К-1.
2.8.	В запаянной с одного конца горизонтальной трубке сечения 5 на расстоянии I от запаянного конца находится поршень массы т. Другой конец трубки открыт, по обе стороны поршня — воздух, давление которого равно р0. Трубку начинают вращать с угловой скоростью со вокруг вертикальной оси, проходящей через запаянный конец трубки. На каком расстоянии от запаянного конца трубки будет находиться поршень? Температуру воздуха считать постоянной, трением пренебречь.
2.9.	В запаянную сверху трубку барометра, которая была целиком заполнена ртутью (с высотой столба Н), впускают снизу порцию воздуха объема V при атмосферном давлении р0 (рис. 2.9). На какое расстояние h от верхнего торца трубки опустится столб ртути? Внутреннее сечение трубки равно S, плотность ртути р.
*7
Рис. 2.10
2.10°	. В открытом цилиндрическом сосуде между поршнями А и В (рис. 2.10) находится идеальный газ. Над поршнем В находится жидкость плотности р, заполняющая сосуд до верха. Высота столба жидкости равна На какое расстояние надо сместить вверх поршень А, чтобы 50
над поршнем В остался столб жидкости высоты Н2, если температура газа при этом остается постоянной? Первоначальное расстояние между поршнями равно h. Атмосферное давление равно р0. Массой поршня В и трением поршней о стенки сосуда пренебречь.
2.11.	Сосуд высоты 2h, разделенный посередине горизонтальной перегородкой, содержит в верхней части воду, а в нижней — воздух при атмосферном давлении р0. В перегородке открывается небольшое отверстие, такое, что вода начинает протекать в нижнюю часть сосуда. Какой толщины будет слой воды в нижней части сосуда в момент, когда воздух начнет проходить из отверстия вверх? Плотность воды равна р0. Температуру воздуха считать постоянной.
2.12.	В герметически закрытый цилиндрический сосуд высоты h через крышку вертикально вставлена немного не доходящая до дна сосуда тонкостенная трубка длины /. В первоначально сухой сосуд через трубку наливают воду. Найти высоту уровня воды в сосуде, при котором трубка заполнится водой. Атмосферное давление равно рй, плотность воды равна р0. Соединение трубки с крышкой сосуда герметично. Поверхностным натяжением пренебречь.
2.13*	. Вертикально стоящие закрытые цилиндры А и В имеют одинаковую высоту Н и соответственно. Они закреплены на дне широкого открытого цилиндра С с площадью дна 35 (рис. 2.13). В верхней части цилиндры А и В соединены трубкой 1, а в нижней через небольшие отверстия в дне все три цилиндра связаны друг с другом общим узким каналом 2. Первоначально цилиндр А доверху наполнен водой, а отверстие в дне этого цилиндра закрыто. Найти высоту уровня воды, устано-
вившуюся в цилиндре С после открывания отверстия в цилиндре А. Атмосферное давление равно р0, плотность воды равна р0. Температура постоянна. Объемом трубки, канала и стенок сосудов пренебречь.
2.14. В вакууме находится тонкостенный эластичный пузырь радиуса Ri с газом, внутри которого находится такой же пузырь радиуса R2 и с таким же газом. Внутренний пузырь лопается. Найти радиус внешнего пузыря, если температура газа поддерживается постоянной
площади дна S и S/2
Рис. 2.13
51
Считать, что эластичная оболочка создает давление внутри пузыря, обратно пропорциональное его радиусу.
2.15°	. В длинной узкой пробирке с воздухом, расположенной горизонтально, капеЛька ртути находится на расстоянии h от дна. Если пробирку повернуть вверх отверстием, то капелька окажется на расстоянии /2 от дна. На каком расстоянии от дна окажется капелька, если перевернуть пробирку вверх дном? Температуру воздуха считать постоянной.
2.16.	Участки АВ и CD вертикальной запаянной с концов узкой стеклянной трубки ABCDEF заполнены воздухом, участки ВС и DE — ртутью, в участке EF — вакуум (рис. 2.16). Длины все участков одинаковы. Давление в нижней точке А равно р. Трубку осторожно пере
Рис. 2.16
ворачивают так, что точка F оказывается внизу. Каким станет давление в точке F? Температуру воздуха считать постоянной.
2.17.	В U-образной трубке с воздухом на одинаковой высоте h удерживают два поршня'массы т каждыифис. 2.17). Площадь сечения левого колена трубки равна 2S, площадь сечения правого колена и нижней части трубки равна S. Длина нижней части равна 3/г. Давление воздуха в трубке р0 равно атмосферному. Поршни отпускают. Найти установившиеся высоты поршней. Поршни могут перемещаться только по вертикальным участкам трубки. Температуру воздуха считать постоянной.
2.18.	Прямоугольный сосуд с непроницаемыми стенками разделен вертикальным подвижным легким поршнем на две части: слева находится ртуть, справа — воздух. В начальный момент поршень находится в равновесии и делит сосуд на две равные части длины I. На сколько сместится поршень вправо после увеличения температуры (по шкале 52
Кельвина) в три раза? Тепловым расширением ртути и сосуда, а также трением пренебречь.
2.19°	. Между поршнем и дном закрытого цилиндрического сосуда, заполненного воздухом, закреплена перегородка с отверстием, закрытым пробкой. Давление воздуха между перегородкой и дном сосуда равно pt. Массы воздуха по обе стороны от перегородки одинаковы. При движении поршня пробка вылетает, когда разность давлений по обе стороны от перегородки становится равной Др. В этот момент поршень останавливают. Найти установившееся в сосуде давление р2. Температуру воздуха считать постоянной.
2.20.	В вертикально стоящем цилиндрическом сосуде, заполненном воздухом, находятся в равновесии два тонких одинаковых тяжелых поршня. Расстояние между поршнями и расстояние от нижнего поршня до дна сосуда одинаковы и равны 1= 10 см, давление между поршнями равно удвоенному нормальному атмосферному давлению р0. На верхний поршень давят таким образом, что он перемещается на место нижнего. На каком расстоянии от дна будет находиться нижний поршень? Температуру воздуха считать постоянной. Трением пренебречь.
2.21.	Два расположенных горизонтально цилиндрических сосуда, соединенных герметически, перекрыты поршнями, соединенными несжимаемым стержнем. Между поршнями и вне их находится воздух при атмосферном давлении р0. Площади поршней равны Si и S2. Первоначальный объем воздуха между поршнями равен Ко (рис. 2.21). На сколько сместятся поршни, если давление в камере А повысить до значения р? Температуру воздуха считать постоянной. Трением пренебречь. Давление в камере В остается равным атмосферному.
Рис. 2.21
Рис. 2.22
2.22.	Два закрепленных цилиндра (рис. 2.22), площади сечения которых равны Si и S2, соединены трубкой. Цилиндры перекрыты поршнями, жестко соединенными между собой. Объем газа, ограниченный поршнями, вначале был
63
равен V, а давление внутри системы было равно атмосферному давлению снаружи р0. Затем внешнее атмосферное давление изменяется до значения р. На какое расстояние сдвинутся поршни? Температуру газа считать постоянной. Трением пренебречь.
2.23.	В двух соединенных герметически цилиндрических сосудах различного поперечного сечения, закрытых поршнями, находятся идеальные газы при одной и той же температуре. Поршни жестко связаны между собой стержнем и находятся в равновесии. Между поршнями — вакуум. Расстояния от дна сосудов до поршней одинаковы (рис. 2.23). Температуру газа в каждом цилиндре изменили, после чего поршни сместились вправо на половину расстояния между поршнем и дном. Найти отношение температур газов (по шкале Кельвина).
Рис. 2.23	Рис. 2.24
2.24.	В шероховатую трубу, площадь сечения которой равна S, вставляют пробку и проталкивают ее внутрь стержнем на всю его длину. Затем вставляют и проталкивают точно такую же вторую пробку, а затем третью. Найти силу трения между стенками трубы и пробкой, если в результате первая пробка оказалась на расстоянии двойной длины стержня от левого конца трубы (рис. 2-24). Атмосферное давление равно р0. Температуру воздуха считать постоянной. Толщиной пробок пренебречь.
2.25.	В горизонтально закрепленной, открытой с торцов трубе сечения S находятся два поршня. В исходном состоянии левый поршень соединен недеформированной пружиной жесткости к со стенкой, давление газа р0 между поршнями равно атмосферному, расстояние I от правого поршня до края трубы равно расстоянию между поршнями (рис. 2.25). Правый поршень медленно вытянули до края трубы. Какую силу надо приложить к поршню, чтобы удерживать его в этом положении? Температуру газа считать постоянной. Трением пренебречь.
2.26*	. Цилиндрическая камера длины 21 с поршнем, площадь сечения которого равна S, может двигаться по 54
горизонтальной плоскости с коэффициентом трения k (рис. 2.26). Слева от поршня, расположенного в центре камеры, находится газ при температуре То и давлении /;0 Между неподвижной стенкой и поршнем помещена пружина жесткости к. Во сколько раз нужно увеличить температуру
Рис. 2.25
Рис. 2.26
газа слева от поршня, чтобы объем этого газа удвоился, если трением между камерой и поршнем можно пренебречь? Масса камеры и поршня равна т. Наружное давление равно ро-
2.27°	. В вертикальном открытом сверху цилиндрическом сосуде, имеющем площадь поперечного сечения S, на высоте h от дна находится поршень массы т, поддерживаемый сжатым газом с молярной массой ц. Температура газа равна 7, атмосферное давление равно р0. Определить массу газа в сосуде. Трением пренебречь.
2.28°	. Найти подъемную силу аэростата, наполненного гелием. Масса гелия равна т. Давления и температуры внутри и вне оболочки аэростата одинаковы. Молярная мае- .	------
са гелия ц, воздуха — цв. v<
Ускорение свободного падения 51---------------ПГТ/? т
равно §. Массой оболочки пре- °1 ’ ’ \~у	2
небречь.	___—
2.29.	Через трубку пере-	рис. 2.29
менного сечения продувают
воздух. Входное отверстие имеет площадь Si, выходное — S2 (рис. 2.29). На входе скорость воздуха постоянна и равна vlt температура 7\, давление На выходе температура Т2, давление р2. Найти скорость воздуха в выходном сечении.
2.30.	Закрытый цилиндрический сосуд, вместимость которого равна V, заполнен газом с молярной массой р. при температуре Т. При свободном падении сосуда давление газа на дно равно р. Найти разность сил давления на дно и крышку в случае, когда сосуд покоится. Молярный
55
объем газа при нормальных условиях, т. е. при давлении р0 и температуре То> равен 1/0-
2.31°	. В вертикальном закрытом цилиндрическом сосуде, площадь основания которого равна S, разделенном поршнем массы m на два отсека, находится газ. Масса газа под поршнем в п раз больше, чем над поршнем. В по-
ложении равновесия поршень находится посередине сосуда, а температура газа в обоих отсеках одинакова. Найти
давление газа в каждом отсеке. Трением и силой тяжести, действующей на газ, пренебречь.
2.32. Посередине закрытой с торцов трубы длины 2L и сечения S находится поршень (рис. 2.32). Слева и справа от поршня находятся разные газы при одинаковом давлении р. На какое расстоя
ние сместится поршень, если он становится проницаемым для одного из газов? Сила трения поршня о трубу равна F. Температуру газа считать постоянной.
2.33. В вертикальном закрытом цилиндрическом сосуде, высота которого равна 2/г, а площадь основания S, находится тяжелый поршень массы т. Первоначально поршень, делящий объем сосуда пополам, уравновешен. Над поршнем находится гелий при давлении рНе, под поршнем — кислород. Поршень проницаем для гелия и непроницаем для кислорода. Через некоторое время пор-
шень занимает новое равновесное положение, смещаясь вверх. Найти смещение поршня. Температуру газов считать постоянной. Трением пренебречь.
2.34. В блюдце налито т=30 г воды, а сверху на воду поставлен перевернутый вверх дном разогретый цилиндрический стакан с тонкими стенками. До какой наименьшей температуры должен быть нагрет стакан, чтобы после остывания его до температуры окружающего воздуха Го= —300 К в него оказалась бы втянута вся вода? Атмосферное давление ра—106 Па, площадь сечения стакана 5= =20 см2, высота /7=10 см, плотность воды р0=1-103 кг/м3. Считать, что ускорение свободного падения g=10 м/с2. Явлением испарения, поверхностным натяжением и расширением самого стакана пренебречь.
56
§ 2.	Пар
2.35°	. В воду при /01=90 °C бросают раскаленные платиновые опилки. Найти начальную температуру опилок /02, если известно, что после прекращения кипения уровень воды остался первоначальным. Удельная теплоемкость воды Ci=4,19• 103 Дж/(кг- К), удельная теплота парообразования воды Xi=2,26-106 Дж/кг при температуре кипения /кип=100°С, плотность платины р2=21,4-103 кг/м3, удельная теплоемкость платины с2= 128 Дж/(кг-К). Изменением плотности воды при нагреве пренебречь.
2.36.	Теплообменник представляет собой трубу, внутри которой находится змеевик. В змеевик в 1 с поступает 1 кг
Рис. 2.36
Рис. 2.39
водяного пара, т. е. расход пара 73=1 кг/с, при температуре /х= 100 °C. Навстречу пару движется вода, расход которой 72=10 кг/с (рис. 2.36). Определить температуру воды на выходе из теплообменника, если на входе, температура воды /2=20 °C. Удельная теплота парообразования воды Х=2,26-106 Дж/кг, удельная теплоемкость воды с=4,19-103 Дж/(кг-К).
2.37.	В герметично закрытом сосуде, вместимость которого 1/0=1,1 л, находится Л4=100 г кипящей воды и пара при температуре 100 °C. Найти массу пара. Плотность воды равна р0. Считать, что воздуха в сосуде нет.
2.38.	Паровой котел частично заполнен водой, а частично — смесью воздуха и насыщенного пара при температуре /1=100 °C. Начальное давление в котле р=Зр0= =3-10® Па. Найти давление в котле в случае, когда температуру в нем понизили до значения /2=10°С. Давлением насыщенного водяного пара при 10 °C пренебречь.
2.39*	. Цилиндр прикреплен вверх дном к стенке открытого сосуда с водой (рис. 2.39). Верхняя часть цилиндра заполнена воздухом, давление которого равно атмосфер-
57
ному давлению. Высота дна цилиндра над уровнем воды h—1 см. Вода имеет температуру /о=0 °C. На сколько сместится уровень воды в цилиндре, если воду и воздух нагреть до 100 °C (но не доводить воду до кипения)? Тепловым расширением воды и цилиндра, а также давлением водяного пара при /о=0°С пренебречь. Сосуд широкий.
2.40.	В сосуде над поверхностью воды находится воздух, сжатый поршнем до давления рх=3-106 Па. Расстояние от поршня до поверхности воды и толщина слоя воды одинаковы: /г=2 см. Температура воздуха и воды ^=6 °C. На каком расстоянии от поверхности воды окажется поршень, если трубку с водой нагреть до температуры С = Ю0°С? Давлением водяного пара при температуре и трением пренебречь.
2.41.	В цилиндрическом сосуде под поршнем вначале находится v=l моль водяного пара при температуре Т и давлении р. Давление насыщенного пара воды при этой температуре равно 2р. Поршень вдвигают так, что первоначальный объем под поршнем уменьшается в четыре раза. Найти массу сконденсировавшейся воды, если температура остается неизменной. Молярная масса воды ц= = 0,018 кг/моль.
2.42.	Посередине горизонтальной трубки, закрытой с торцов, находится поршень. Слева и справа от него при давлении р имеется пар, конденсирующийся при давлении 2р. Трубку ставят вертикально. При этом объем под поршнем уменьшается в четыре раза. Найти вес поршня, если площадь поршня равна S. Трение пренебрежимо мало. Температура в обоих отсеках одинакова и постоянна.
Рис. 2.43
2.43.	В цилиндрической трубе на расстояниях L и 2L от закрытого торца находятся два поршня, которые могут перемещаться без трения (рис. 2.43). В левом отсеке на-58
ходится пары воды при давлении р, а в правом — воздух при том же давлении. Давление насыщенных паров воды равно 2р. Правый поршень медленно вдвинули на расстояние /. На какое расстояние сдвинется левый поршень? Температуру паров воды и воздуха считать постоянной.
2.44.	Пробирка длины I, содержащая воздух и насыщенный водяной пар, касается открытым концом поверхности воды (рис. 2.44). Пробирку погружают в воду наполовину. При этом поверхность воды в пробирке оказывается на глубине h. Найти давление насыщенного водяного пара. Температуру паров воды и воздуха считать постоянной. Атмосферное давление рп, плотность воды р0.
2.45°	. На какую высоту нужно поднять поршень, закрывающий сосуд с водой, чтобы вся вода испарилась? Толщина слоя воды равна h, плотность воды равна р„, ее молярная масса равна р, давление насыщенного водяного пара равно р. Температура воды и пара Т поддерживается постоянной. Воздуха в сосуде нет.
2.46.	В цилиндрическом сосуде над поршнем при температуре Т находится насыщенный пар. Определить массу сконденсировавшегося при изотермическом вдвигании поршня пара, если при этом совершена работа А. Молярная масса пара равна р.
2.47.	В цилиндрическом сосуде под поршнем при температуре Т находится насыщенный пар. При изотермическом вдвигании в сосуд поршня выделилось количество теплоты Q. Найти совершенную при этом работу. Молярная масса пара равна р, удельная теплота парообразования воды равна л.
§ 3.	Графики для тепловых процессов. Закон сохранения энергии в тепловых процессах
2.48.	Идеальный газ участвует в некотором процессе, изображенном в р,V-координатах, проходя последовательно состояния 1, 2, 3, 4, 1 (рис. 2.48). Изобразить этот же процесс в р, Т-координатах.
2.49°	. Диаграмма циклического процесса для одного моля газа в осях р, Т образует прямоугольник ABCD (рис. 2.49), стороны ВС и AD которого соответствуют давлениям р2 и pi, а АВ и CD — температурам 1\ и Т2. Найти максимальный и минимальный объемы газа. Газовая постоянная равна R.
2.50.	Один моль идеального газа участвует в некотором процессе, изображенном в р, Т-координатах, проходя по
59
следовательно состояния 1, 2, 3, 4, 1 (рис. 2.50). При этом 7\==Г3=2Го, Тt=To, 712“=4710, рх—р^—^ро, p^pi^pjQ. Найти работу, совершенную газом за этот цикл.
Г	\
м	”
* Г
Z>|
Рис. 2.49
Рис. 2.48
2.51.	Один моль идеального газа участвует в некоторое процессе, изображенном в р,У-координатах (рис. 2.51), Продолжения отрезков прямых 1—2 и 3—4 проходят через
начало координат, а кривые 1—4 и 2—3 являются изотермами. Изобразить этот процесс в Т, V -координатах и найти объем Vs, если известны объемы 1/х и И2=И4.
।
---------,-----1—
о	V2 Г
Рис. 2.52
2.52.	Один моль газа, внутренняя энергия которого U—cT, участвует в некотором процессе, изображенном на рис. 2.52. Газ проходит последовательно состояния 1,2,3.
60
Найти поглощенное газом в этом процессе количество теплоты, если известны объемы Va и давления plt р2.
2.53.	На р, V-диаграмме изображен процесс расширения газа, при котором он переходит из состояния 1 с давлением р и объемом V в состояние 2 с давлением р/2 и объемом 2V (рис. 2.53). Найти количество теплоты Q, которое сообщили этому газу. Линия 1—2 — отрезок прямой.
2.54*	. Один моль идеального газа, внутренняя энергия которого U—(3/2)RT, сначала нагревают, затем охлаждают так, что замкнутый цикл 1—2—3—1 на р, К-диаграмме состоит из отрезков прямых 1—2 и 3—1, параллельных осям р и V соответственно, и изотермы 2—3. Найти количество теплоты, отданное газом в процессе охлаждения. Давление и объем газа в состоянии 1 равны рг и Vlt давление газа в состоянии 2 равно р2.
2.55.	Закрытый с торцов теплоизолированный цилиндрический сосуд перегорожен подвижным поршнем массы М. С обеих сторон от поршня находится по одному молю идеального газа, внутренняя энергия которого U=cT. Масса сосуда с газом равна т. Коротким ударом сосуду сообщают скорость и, направленную вдоль его оси. На сколько изменится температура газа после затухания колебаний поршня? Трением между поршнем и стенками сосуда, а также теплоемкостью поршня пренебречь.
2.56.	В длинной закрытой трубке между двумя поршнями массы т каждый находится идеальный газ, масса которого много меньше массы поршней. В остальном пространстве трубки — вакуум. В начальный момент правый поршень имеет скорость и, а левый — 3v (рис. 2.56). Найти максимальную температуру газа, если стенки трубки и поршни
Рис. 2.56
теплонепроницаемы. Температура газа в начальный момент равна То. Внутренняя энергия одного моля газа U=cT Трением пренебречь.
2.57.	В гладком вертикальном цилиндрическом сосуде под невесомым поршнем площади S находится воздух при атмосферном давлении /?0 и температуре То. Внутри сосуд
61-
разделен на два одинаковых объема неподвижной горизонтальной перегородкой с маленьким отверстием. На поршень кладут груз массы т, под действием которого поршень очень медленно опускается до перегородки. Найти температуру воздуха внутри сосуда, если его стенки и поршень не проводят тепло. Внутренняя энергия одного моля газа U=cT.
2.58*	. В двух одинаковых колбах, соединенных трубкой, перекрытой краном, находится воздух при одинаковой температуре Tr—Ti и разных давлениях. После того как кран открыли, часть воздуха перешла из одной колбы в другую. Через некоторое время давления в колбах сравнялись, движение газа прекратилось и температура в одной из колб стала равной Т[. Какой будет температура в другой колбе в этот момент? Внутренняя энергия одного моля воздуха U=cT. Объемом соединительной трубки пренебречь. Теплообмен со стенками не учитывать.
2.59*	. В вертикальном цилиндрическом сосуде, площадь сечения которого равна S, под поршнем массы ш находится газ, разделенный перегородкой на два одинаковых объема. Давление газа в нижней части сосуда равно р, внешнее давление равно ро, температура газа в обеих частях сосуда равна Т. На сколько сместится поршень, если убрать перегородку? Внутренняя энергия одного моля газа U=cT. Высота каждой части сосуда равна h. Стенки сосуда и поршень не проводят тепло. Трением пренебречь.
2.60.	Поршень удерживается в середине неподвижного теплоизолированного закрытого цилиндрического сосуда длины 21, площади сечения S. Правую половину сосуда занимает газ, температура и давление которого равны Л и plt в левой половине — вакуум. Поршень соединен с левым торцом сосуда пружиной жесткости к (рис. 2.60). Найти установившуюся температуру газа Тг после того, как поршень отпустили. Длина недефор-мированной пружины равна 21,
внутренняя энергия одного моля газа U—cT. Трением, теплоемкостью цилиндра и поршня пренебречь.
2.61*	. В теплоизолированном длинном цилиндрическом сосуде, стоящем вертикально, на высоте h от дна висит на нити поршень массы т. Под поршнем находится один моль газа, давление которого в начальный момент равно внешнему давлению р0, а температура равна То. Какое ко-62
личество теплоты нужно подвести к газу, чтобы поршень поднялся до высоты 2/г? Внутренняя энергия одного моля газа U=cT. Трением пренебречь.
2.62.	В теплоизолированном закрытом поршнем цилиндрическом сосуде, расположенном горизонтально, содержится один моль газа при давлении в два раза меньшем внешнего и температуре Т. Поршень может свободно передвигаться, увеличивая вместимость сосуда, и удерживается стопором от движения в противоположную сторону (рис. 2.62).
Р/2
. 3
Ж
Рис. 2.62
Рис. 2.63
Какое количество теплоты следует подвести к газу, чтобы его объем увеличился в два раза? Внутренняя энергия одного моля газа U=cT.
2.63*	. Два цилиндра одинаковой длины I и с площадью сечений: левого, равной 5, и правого, равной aS (рис. 2.63), соединены между собой. Посредине каждого цилиндра находятся поршни, соединенные жестким стержнем. Во всех трех отсеках системы находится идеальный газ. Давление в отсеке / равно р, а в отсеке 3 — равно ftp. Трение пренебрежимо мало, поршни находятся в равновесии. К системе подвели количество теплоты Q так, что температура возросла, оставаясь во всех отсеках одинаковой. Определить изменение давления в отсеке 1. Внутренняя энергия одного моля газа равна сТ. Теплоемкостью цилиндров и поршней пренебречь.
2.64*	. В горизонтальном неподвижном цилиндрическом сосуде, закрытом поршнем массы М, находится газ. Газ нагревают. Поршень, двигаясь равноускоренно, приобретает скорость v. Найти количество теплоты, сообщенной газу. Внутренняя энергия моля газа U=cT. Теплоемкостью сосуда и поршня, а также внешним давлением пренебречь.
2.65*	. В горизонтальном неподвижном цилиндрическом сосуде, закрытом поршнем, площадь сечения которого равна S, находится один моль газа при температуре Та и давле-
нии р0. Внешнее давление постоянно и равно р0 (рис. 2.65). Газ нагревают внешним источником теплоты. Поршень на-
Рис. 2.65
чинает двигаться, причем сила трения скольжения равна f. Найти зависимость температуры газа Т от получаемого им от внешнего источника количества теплоты Q, если в газ поступает еще и половина количества теплоты, выде
ляющегося при трении поршня о стенки сосуда. Построить график этой зависимости. Внутренняя энергия одного моля газа U=cT. Теплоемкостью сосуда и поршня пренебречь.
Глава III. ЭЛЕКТРИЧЕСТВО И МАГНЕТИЗМ
§ 1. Напряженность, потенциал и энергия электростатического поля
3.1	°. Незаряженный металлический цилиндр вращается вокруг своей оси с постоянной угловой скоростью (0. Найти напряженность Е электрического поля в цилиндре на расстоянии г от его оси. Заряд электрона равен е, масса пг.
3.2	°. Напряженность электрического поля Е у поверхности Земли в среднем равна 120 В/м и направлена по вертикали. Найти электрический заряд Земли, учитывая, что ее радиус Р3=6,4-103 км, а е0=8,85-10"12 Ф/м.
3.3	°. В вершинах ромба, сторонами которого служат нерастяжимые нити, находятся в равновесии заряды qlt q2, qlt q«. Найти угол при вершине с зарядом qt.
3.4	°. По кольцу радиуса R, расположенному вертикально в поле тяжести Земли, могут скользить одинаковые шарики массы т.
1)	Какие заряды следует сообщить двум подвижным шарикам, чтобы они расположились на концах горизонтальной хорды, стягивающей дугу 120°? Рассмотреть два случая: хорда проходит выше или ниже центра кольца.
2)	Какой заряд нужно сообщить неподвижно закрепленному шарику, чтобы он и два подвижных шарика, имеющие одинаковый заряд q, расположились в вершинах правильного треугольника? Рассмотреть два случая: закрепленный шарик находится на верхнем или нижнем конце вертикального диаметра.
64
3)	Два шарика закреплены на концах вертикального диаметра и имеют заряды qx и q2. Какие заряды нужно сообщить двум подвижным шарикам, чтобы все четыре шарика расположились в вершинах квадрата?
4)	Два подвижных шарика имеют одинаковый заряд. Какой заряд нужно сообщить двум закрепленным шарикам, расположенным на концах горизонтальной хорды, стягивающей дугу 90°, чтобы все четыре шарика расположились в вершинах квадрата? Рассмотреть два случая: хорда проходит выше или ниже центра кольца.
3.5°	. К бесконечной плоскости, расположенной вертикально в поле тяжести Земли и имеющей поверхностную плотность заряда о, прикреплен на нити одноименно заряженный шарик массы тис зарядом q. Найти силу натяжения нити и угол а отклонения нити от вертикали. Напряженность поля заряженной плоскости £=0/280.
3.6.	Две бесконечные параллельные проводящие плиты заряжены так, что суммарная поверхностная плотность заряда обеих поверхностей первой плиты равна Oj, а второй — о2. Найти плотность заряда каждой поверхности обеих плит.
3.7*	. Кольцо из проволоки разрывается, если его зарядить зарядом q. Диаметр кольца и диаметр проволоки увеличили в три раза. Какой заряд следует сообщить новому кольцу, чтобы оно разорвалось?
3.8°	. Равномерно заряженный стержень АВ создает в точке О электрическое поле напряженности £0, потенциал которого равен <р0 (рис. 3.8). Какими станут напряженность поля и потенциал в точке О, если в плоскости АОВ поместить еще один такой же и так же заряженный стержень А'В', причем АО=ВО=А'О=В'О и A'B’_LAB?

I I
I
Рис. 3.8
Рис. 3.9
3.9°	. Две стороны правильного треугольника образованы одинаковыми равномерно заряженными палочками. При этом в центре 0 треугольника (рис. 3.9) потенциал равен
3 Г. В. Меледин
Фо, а напряженность электрического поля равна Ео. Найти потенциал ф, а также модуль и направление вектора напряженности Е, которые будут в точке О, если убрать одну из палочек.
3.10.	Три одинаковых проводящих шара расположены в вершинах равнобедренного прямоугольного треугольника, катеты которого велики по сравнению с радиусами шаров, l^r (рис. 3.10). Вначале заряд имеется лишь на шаре 1. Затем шары соединяют проводником, после чего проводник убирают. Потом такую же процедуру совершают с шарами 2 и 3, а затем с шарами 3 и 1. Какой заряд после этого окажется на каждом из шаров?
Рис. 3.10
Рис. 3.11
3.11.	Разноименные заряды q и —q находятся на расстояниях /j и 12 от заземленной сферы малого радиуса г (рис. 3.11). Ра< тояние от зарядов до поверхности земли и других заземле./ ых предметов много больше It и /2. Найти силу, с которой заряды действуют на сферу. Угол с вершиной в центре сферы, образованный прямыми, проведенными через заряды, равен 90°.
3.12.	Два металлических шара, заряд каждого из которых равен q, расположены на расстоянии I друг от друга. Первый шар заземляют и затем удаляют заземляющий проводник. Затем такую же процедуру совершают со вторым шаром. После этого снова заземляют первый шар и т. д. Каково отношение зарядов на шарах после 2п (п — целое) заземлений и изоляций? Оба шара находятся на очень большом расстоянии от земли. Радиусы шаров г много меньше I.
3.13*	. Внутри сферы радиуса /?, заряд которой равен q, находится заземленная проводящая сфера радиуса г. Центры сфер совпадают. Найти напряженность электрического поля вне большой сферы на расстоянии I от ее центра.
3.14°	. Вокруг неподвижного протона, заряд которого равен е, по окружности радиуса г движутся, образуя квад-66
рат, четыре электрона, заряд которых равен —е, а масса т. Определить скорость электронов.
3.15.	Электрон и позитрон движутся по окружности вокруг своего неподвижного центра масс, образуя атом позитрония. Найти отношение потенциальной и кинетической энергий частиц. Электрон и позитрон отличаются только знаками заряда.
3.16.	Две частицы масс тг и т2, разноименно заряженные, движутся под влиянием взаимного электрического притяжения по окружности вокруг неподвижного центра масс. Скорость частицы массы мгновенно увеличивают в п раз, не изменяя ее направления. При каком минимальном 11 частицы разлетятся бесконечно далеко друг от друга? Заряды частиц равны по модулю.
3.17.	Какую работу надо совершить, чтобы ионизовать атом водорода, т. е. удалить электрон, заряд которого е=—1,6- 10~1а Кл, от протона на очень большое расстояние? Диаметр атома водорода d принять равным 10-8 см. Выразить работу по ионизации атома в электронвольтах (1 эВ= = 1,6-10~19 Дж=1,6-10-1? эрг; электрическая постоянная щ=8,85-10~12 Ф/м).
3.18,	Скорости двух электронов©! и ©2 лежат в одной плоскости и при расстоянии I между электронами образуют
Рис. 3.18
углы а с прямой, соединяющей электроны (рис. 3.18). На какое минимальное расстояние сблизятся электроны, если скорости ©! и ©2 равны по модулю и? Заряд электрона равен е, масса равна т.
3.19.	Маятник ОА представляет собой невесомую тонкую изолирующую спицу длины I, на конце которой находится шарик массы т, имеющий заряд q (рис. 3.19). Второй шарик, заряд которого равен —q, закреплен в точке С, причем ОВ=1 — вертикаль, ВС=1 — горизонталь. Найти силу, действующую на ось маятника в момент прохождения им точки В. В начальный момент времени маятник имел скорость, равную нулю, и был отклонен от вертикали на угол а=45°. Ускорение свободного падения равно g.
3.20.	Заряженный шарик подвешен на нерастяжимой изолирующей нити длины I (рис. 3.20). Масса шарика равна з*	67
tn, его заряд равен q. На одной высоте с точкой подвеса на расстоянии 21 от нее закреплен заряд —q. Найти минимальную скорость, которую должен иметь шарик в нижней точке, чтобы, двигаясь по окружности, он достиг верхней точки. Размерами шарика пренебречь.
Рис. 3.19
3.21.	Два шарика с зарядами q и —q одинаковой массы т, соединенных невесомым стержнем длины /, движутся по окружности в однородном электрическом поле напряженности Е (рис. 3.21). В тот момент, когда стержень направлен вдоль вектора Е, заряды имеют скорость v0. Найти силу натяжения стержня в момент, когда он повернулся на 90°. Силой тяжести пренебречь.
Рис. 3.21
3.22.	Жесткое тонкое непроводящее кольцо’массы М, равномерно заряженное зарядом q, имеет около точки А (рис. 3.22) небольшой зазор размера I, много меньшего, чем радиус кольца. Кольцо расположено в горизонтальной плоскости так, что может только свободно вращаться относительно вертикальной оси, проходящей через центр кольца О. Покоившееся вначале кольцо пришло во вращение после того, как было включено постоянное горизонтальное однородное электрическое поле напряженности Е, перпендикулярное ОА. Найти максимальную скорость точек кольца.
68
3.23.	Два одинаковых шарика массы т, имеющие одинаковый заряд q, начинают скользить по двум одинаковым неподвижным и непроводящим спицам. Спицы расположены в вертикальной плоскости, причем каждая спица наклонена к горизонту под углом а (рис. 3.23). На какую мак-
.|V
777777X7777777.	’
Рис. 3.23
симальную высоту над первоначальным уровнем поднимутся шарики? В начальный момент шарики покоились и находились на расстоянии L друг от друга и на расстоянии I от концов спиц. Трением пренебречь.
3.24.	Три маленьких одинаковых шарика массы т, имеющие одинаковый заряд q, могут скользить по очень длинному стержню. Какую скорость будут иметь шарики на очень большом расстоянии друг от друга, если в начальный момент они покоились и расстояние между ними было равно /?
3.25.	Два одинаковых шарика, имеющие одинаковый заряд <7, соединены пружиной. Шарики колеблются так, что расстояние между ними меняется от I до 4/. Найти жесткость пружины, если ее длина в свободном состоянии равна 2/.
3.26.	Два небольших тела, связанные нитью длины I, лежат на горизонтальной плоскости. Заряд каждого тела равен q, масса равна т. Нить пережигают, и тела начинают скользить по плоскости. Какую максимальную скорость разовьют тела, если коэффициент трения равен /г?
3.27.	На горизонтальной шероховатой поверхности закреплен заряд qi. Тело массы пг, имеющее заряд q2, может перемещаться по поверхности. На каком расстоянии от заряда qi тело остановится, если в начальный момент оно покоилось и находилось на расстоянии /0 от заряда qj Заряды qr и qz — одного знака. Коэффициент трения равен k.
3.28*	. Три одинаковых одноименно заряженных тела, заряд каждого из которых равен q, а масса т, соединены невесомыми, нерастяжимыми и непроводящими нитями длины I так, что нити образуют равносторонний треугольник. Одну из нитей пережигают. Найти максимальные скорости тел.
69
3.29*	. В точках А и В (рис. 3.29) на расстоянии |АВ\ — 1 закреплены заряды +9q и —q. Вдоль прямой АВ к ним движется частица массы т, имеющая заряд +<7. Какую наименьшую скорость должна иметь эта частица на очень большом расстоянии, чтобы достигнуть точки В?
жесткости
S
1ц
3.30.	Расстояние между закрепленными разноименными зарядами q± и q2 равно I. Частица массы т, имеющая заряд q3 одного знака с q2, летит по прямой, соединяющей закрепленные заряды, со стороны заряда <7^ Какую наименьшую скорость должна иметь частица на большом расстоянии от зарядов и q2, чтобы достичь заряда qj
3.31.	Маленький шарик массы т подвешен на пружине и несет заряд q. В начальный момент шарик удерживают так, что пружина недеформиро-вана. Под шариком на расстоянии h лежит такой же шарик с зарядом —q (рис. 3.31). Верхний шарик отпускают. При каком минимальном значении q нижний шарик под- . прыгнет?
3.32.	Шарик массы т, имеющий заряд д, находится внизу под закрепленным зарядом —q на расстоянии / от него. Какую минимальную скорость, направленную вниз, надо сообщить шарику, чтобы он упал на землю? Расстояние до земли велико, движение происходит в поле тяготения Земли.
3.33.	Сфера радиуса г расположена внутри сферы радиуса R. Центры сфер совпадают. Внутренняя сфера равномерно заряжена. Ее заряд равен <71. Между сферами строго вдоль радиусов движется множество одинаковых шариков, каждый из которых имеет заряд <72- Шарики попеременно ударяются о поверхности сфер. В момент удара о внутреннюю сферу каждый шарик имеет кинетическую энергию IFK. Найти среднее давление на внешнюю сферу, если среднее давление на внутреннюю сферу, оказываемое шариками при ударах о нее, равно р. Удары упругие. Заряды qt и <72 одного знака. Взаимодействием шариков друг с другом пренебречь.
h

Рис. 3.31
70
3.34°	. Четыре непроводящих шарика радиуса г=10-3 и, в центре каждого из которых находится заряд </—10-7 Кл, расположены вдоль прямой, касаясь друг друга. Какую работу нужно совершить, чтобы сложить из этих шариков пирамидку (правильный тетраэдр)? Влиянием силы тяжести пренебречь.
3.35°	. В закрепленной металлической сфере радиуса г=10-2 м, имеющей заряд q-,=—10-8 Кл, проделано очень маленькое отверстие. Точечный заряд д2=10~6 Кл массы т~ 10’° кг летит по прямой, проходящей через центр сферы и отверстие, имея на очень большом расстоянии от сферы скорость у0=1 м/с. Какова скорость v точечного заряда в центре сферы?
3.36.	Частица массы /щ, имеющая заряд qlt движется со скоростью Vo по оси заряженного кольца, приближаясь к нему. Какую наименьшую скорость должна иметь частица на очень большом расстоянии от кольца, чтобы пролететь сквозь него? Масса кольца равна т2, его радиус равен г, а заряд равен q2. Кольцо не закреплено.
3.37*	. В тонкостенной непроводящей равномерно заряженной сфере массы т± и радиуса г имеются два небольших диаметрально противоположных отверстия. Заряд сферы равен <71. В начальный момент сфера покоится. По прямой, соединяющей отверстия, из бесконечности движется со скоростью v частица массы тг с зарядом q2, одноименным с <7j. Найти время, в течение которого заряд q2 будет находиться внутри сферы.
3.38.	Через п последовательно расположенных вдоль одной оси металлических цилиндрических трубок ускорителя, радиусы которых много меньше длины, проходит пучок заряженных частиц, испускаемых некоторым источником. Трубки поочередно подключены к разным полюсам генератора постоянной частоты и амплитуды напряжения. Длина первой трубки равна I. Длины следующих пролетных трубок выбираются так, чтобы за время пролета заряженной частицы от одного ускоряющего промежутка (пространство между концами пролетных трубок) до другого электрическое поле меняло свой знак и становилось снова ускоряющим. Найти длины пролетных трубок, пренебрегая всюду расстояниями между их концами. В первый ускоряющий промежуток (между источником и первой трубкой) частица попадает с нулевой скоростью. Считать, что скорость частиц далека от скорости света.
3.39*	. Центры двух неметаллических закрепленных сфер радиуса г расположены на расстоянии 4г друг от дру
71
га. По поверхности каждой из них равномерно распределен заряд <71- В правой сфере на оси, соединяющей центры сфер, имеются два маленьких отверстия. Какова должна быть минимальная скорость v расположенной посередине между сферами частицы массы т, имеющей заряд q2, чтобы она смогла пролететь через отверстия правой сферы? Заряды и у2 одного знака.
3.40*	. Центры двух неметаллических неподвижных сфер радиуса г, по поверхности которых равномерно распределен одинаковый положительный заряд, расположены на расстоянии 2,4 г друг от друга. По линии центров в сферах сделаны небольшие отверстия. Вдоль этой линии движется отрицательно заряженная частица, имеющая в средней точке между сферами близкую к нулю скорость. На какое максимальное расстояние она удалится от этой точки?
3.41*	. Центры двух непроводящих шаров скреплены жесткой невесомой спицей, вдоль которой может скользить третий шар. Все шары одинаковы и имеют массу т. На каждом шаре равномерно по поверхности распределен заряд q. Найти максимальную скорость среднего шара, если в начальный момент его центр был удален от центра левого шара на расстояние I, а от центра правого — на расстояние 21. В начальный момент все шары покоятся. Вся система не закреплена. Трением пренебречь.
Рис. 3.42
3.42*	. Центры двух шаров массы т и радиуса г разнесены на расстояние Юг (рис. 3.42). На одном из них равномерно распределен заряд q, на втором —q. Правый шар привязан к удаленной стене нитью, которая выдерживает на разрыв силу натяжения Т. Левый шар отпускают. Найти скорость шаров после соударения, если удар абсолютно неупругий. Заряды не перераспределяются.
§ 2. Электрическая емкость. Конденсаторы
3.43.	Плоский конденсатор находится во внешнем однородном электрическом поле напряженности £’=103 В/м, перпендикулярном пластинам. Площадь пластин конденсатора 5=10-2мг. Какие заряды окажутся на каждой из
72
пластин, если конденсатор замкнуть проводником накоротко? Пластины конденсатора до замыкания не заряжены. Влиянием силы тяжести пренебречь.
3.44.	Плоский конденсатор находится во внешнем однородном электрическом поле напряженности Е, направление которого совпадает с направлением поля в конденсаторе. По пластинам, площадь которых равна S, равномерно распределены заряды q и —q. Какую работу нужно совершить, чтобы перевернуть конденсатор, поменяв пластины местами? Расстояние между пластинами равно d. Влиянием силы тяжести пренебречь.
3.45.	Большая тонкая проводящая пластина, площадь которой равна S, а толщина равна d, помещена в однородное электрическое поле напряженности Е, перпендикулярное пластине. Какое количество теплоты выделится в проводнике, если поле мгновенно выключить?
3.46е	. Два одинаковых плоских конденсатора емкости С соединены параллельно и заряжены до разности потенциалов U. Пластины одного из конденсаторов разводят на большое расстояние. Найти разность потенциалов на втором конденсаторе и его энергию.
3.47.	Электрон влетает в плоский заряженный конденсатор, двигаясь в начальный момент по его средней линии со скоростью Vo (рис. 3.47). Через какое время нужно изменить направление электрического поля в конденсаторе на противоположное, чтобы на вылете из конденсатора электрон пересек плоскость Л В? Длина конденсатора равна I.
Vo
Рис. 3.47
Рис. 3.48
3.48.	В центре плоского конденсатора, расстояние между пластинами которого равно 2Z, находится заряженная сетка (рис. 3.48). Разность потенциалов между сеткой и положительно заряженной пластиной В вдвое больше разности потенциалов между сеткой и отрицательно заряженной пластиной А. Из пластины А под углом а к ее плоскости вылетает положительно заряженная частица и достигает точки,
73
расположенной на расстоянии Z/2 от пластины В. Найти расстояние от точки вылета частицы до точки ее возврата на пластину А. Силой тяжести пренебречь.
3.49.	В четырехэлектродной лампе с плоскими Электродами, площади которых одинаковы и равны S, расстояния между катодом К и анодом А, сетками G, и G3 равны /, dt и d2 (рис. 3.49). Напряжения Ut и U2 известны. Найти заряд на сетке Gf, если известно, что ток через лампу не идет.
Рис. 3.50
3.50°	. Три любых конденсатора в схеме, изображенной на рис. 3.50, имеют одинаковую емкость, а четвертый — в два раза большую. Найти разность потенциалов между точками А и В, если ЭДС батареи £—6 В.
3.51.	Конденсаторы емкости Cj=5 мкФ и С2=10 мкФ включены в схему, как показано на рис. 3.51. К точкам А и В приложена разность потенциалов	В. Найти
3.52э	. Конденсатор сделан из двух наборов пластин, как показано на рис. 3.52. Найти емкость конденсатора, пренебрегая краевыми эффектами. Площадь каждой пластины равна S. Расстояние между пластинами одинаково и равно d (d много меньше поперечных размеров пластин). Полное число пластин равно 2п.
3.53°	. Одну пластину незаряженного конденсатора емкости С заземляют, а другую присоединяют длинным тонким 74
проводом к удаленному от окружающих предметов проводящему шару радиуса г, имеющему заряд q0. Какой заряд останется на шаре?
3.54.	Два проводящих шара радиусов г и R расположены далеко друг от друга и соединены с обкладками конденсатора емкости С (рис. 3.54). Шару радиуса г, отсоедин; в
Рис. 3.54
его от обкладки, сообщили заряд Q, а после зарядки снова присоединили. Какой заряд оказался на другом шаре? Емкостью проводов пренебречь.
3.55.	Конденсаторы емкости С1( С2, С3 включены в схему с резистором, как показано на рис. 3.55. Найти установившиеся заряды конденсаторов. Напряжение U и сопротивление резистора R известны.
Рис. 3.56
3.56.	Конденсаторы емкости Ci и С2 включены в схему, как показано на рис. 3.56. В начальный момент ключ находится в среднем положении (0), а конденсаторы не заряжены. Ключ переводят в положение 1 и через некоторое время — в положение 2. Какая разность потенциалов установится на конденсаторе емкости CR
3.57.	К конденсатору 7 емкости С, заряженному до разности потенциалов U, подсоединяется батарея из конденсаторов такой же емкости, как показано на рис. 3.57. Найти заряд на каждом из шести конденсаторов.
3.58*	. Ключ в цепи, изображенной на рис. 3.58, переводят поочередно в положения 1 и 2 на очень малые одинаковые промежутки времени (рис. 3.58). Какой заряд окажется на
75
конденсаторе после большого числа переключений, если изменение заряда конденсатора за время одного переключения незначительно? Величины Ki, Кг, с?,, S2 и С известны.
Рис. 3.57
Рис. 3.58
3.59.	В участок цепи, изображенный на рис. 3.59 а, подаются повторяющиеся с периодом Т прямоугольные импульсы, длительность которых равна т (рис. 3.59 б). Найти
Рис. 3.59
установившееся через большое время напряжение на конденсаторе, если в течение одного периода напряжение на
конденсаторе изменяется незначительно. Величины U, Rv и известны.
3.60*. Трем одинаковым изолированным конденсаторам, емкости С каждый, были сообщены заряды ql; q2 и q3 (рис. 3.60). Конденсаторы соединили. Найти новые заряды на конденсаторах.
3.61*. Конденсаторы 1, 2 и 3 соединяют, как показано на рис. 3.61. Конденсатор 1 при этом не заряжен, конденса-
Рис. 3.60
торы 2 и 3 заряжены, причем в случае соединения всех трех конденсаторов конденсаторы 2 и 3 подключены друг к другу разноименными обкладками. Найти заряд, появляющийся на конденсаторе 1 при соединении его с конденсаторами 2 и 3, если при соединении с конденсатором 2 на нем возни-
76
кает заряд qi2, а при соединении с конденсатором 3 — заряд q13. Емкости конденсаторов равны соответственно Ci, С2, С3. Конденсаторы 2 и 3 имеют в случае соединения трех конденсаторов те же первоначальные заряды, что и в случаях раздельного их соединения конденсатором 1.
Рис. 3.61
3.62°	. Два электропроводящих поршня площади S образуют в непроводящей трубке плоский конденсатор, заполненный воздухом при атмосферном давлении р0. Во сколько раз изменится расстояние между поршнями, если их зарядить разноименными зарядами q и —<?? Температуру воздуха считать постоянной. Трением пренебречь.
3.63*	. Три одинаковые проводящие пластины А, В и С (рис. 3.63) расположены параллельно друг другу на расстояниях dr и d2. Вначале на пластине А находится заряд q, а пластины В и Сне заряжены. Затем к пластинам В и С присоединяется батарея с ЭДС, равной U, а пластины А и С соединяются проводником. Найти установившиеся заряды на пластинах. Площадь каждой пластины S.
Рис. 3.63
3.64*	. Четыре одинаковые металлические пластины 1—4 площади S каждая с зарядами Qj, •—Q1( Q2, —Q2 соответственно (рис. 3.64), установлены на равных расстояниях друг от друга. Внешние пластины 1 и 4 соединяют проводником. Найти разность потенциалов между внутренними пластинами 2 и 3. Расстояние d мало по сравнению с размерами пластин.
3.65.	Два плоских конденсатора емкости С каждый, соединенные параллельно и заряженные до напряжения V,
77
отсоединяют от источника. Пластины одного из конденсаторов могут двигаться свободно навстречу друг другу. Найти их скорость в момент, когда зазор между пластинами конденсатора уменьшится в два раза. Масса каждой пластины равна М. Силой тяжести пренебречь.
3.66*	. Плоский конденсатор подключен к источнику напряжения U (рис. 3.66). Пластины конденсатора имеют площадь S каждая, расстояние между пластинами равно dY.

~------------г и
xzzzzzzzzzzzza.X
Рис. 3.66
К нижней пластине прижата металлическая пластинка той же площади S, толщины d2, имеющая массу т. Металлическую пластинку отпускают. С какой скоростью она ударится о верхнюю пластину конденсатора? Силой тяжести пренебречь.
3.67*	. В схеме, изображенной на рис. 3.67, в начальный момент ключ К замкнут, а заряженные до напряжения U= =3 В конденсаторы имеют одинаковую емкость С=2 мкФ. Разомкнув ключ, емкость переменного конденсатора уменьшают до значения С/2, после чего ключ К замыкают. Затем, опять разомкнув ключ, емкость переменного конденсатора доводят до прежней величины С и ключ К замыкают. Какое количество теплоты выделится на сопротивлении?
3.68.	В схеме, изображенной на рис. 3.68, напряжение на конденсаторе С2 равно U, а конденсаторы С± и С3 не
Рис. 3.68
Рис. 3.69
заряжены. Ключ К попеременно переключают в положения А и В. Какое количество теплоты выделится после очень большого числа переключений?
78
3.69*	. Три одинаковые плоские металлические пластины А, В и С (рис. 3.69) площади S каждая расположены параллельно друг другу на расстояниях dx и d2. Пластины изолированы, и на пластинах В и С находятся заряды q и —q, а пластина А не заряжена. Пластины Л и С соединили через сопротивление, замкнув ключ /С Найти количество теплоты, выделившееся на сопротивлении.
§ 3.	Цепи постоянного и переменного тока
3.70°	. Из проводящей трубки с внешним радиусом b и толщиной стенок d изготовили проволоку квадратного сечения dxd. Определить отношение сопротивления проволоки к сопротивлению, которое имела трубка при подключении электродов к ее торцам.
3.71°	. Сетка состоит из одинаковых звеньев (рис. 3.71). Сопротивление каждого звена Д==1 Ом. Найти сопротивление между точками Л и В.
3.72°	. Найти сопротивление между точками Л и В цепи, изображенной на рис. 3.72, при разомкнутом и замкнутом ключе. Сопротивление каждой стороны и диагонали квадрата Ом.
3.73.	Найти сопротивление между соседними вершинами проволочного куба. Сопротивление каждого из ребер куба Д=1 Ом.
3.74.	Найти сопротивление между вершинами куба, расположенными на диагонали одной из его граней. Сопротав-ление каждого из ребер куба Д=1 Ом.
3.75.	В двухпроводной линии длины I на некотором расстоянии х от ее начала А'А пробило изоляцию, что привело к появлению некоторого конечного сопротивления между проводами в этом месте (рис. 3.75). Для поиска места пробоя провели три измерения: сопротивление между точками
79
А и А' равно R\ при разомкнутых концах В и В' и равно R2 при коротко замкнутых концах В и В', и сопротивление между точками В и В' при разомкнутых концах А и А' равно /?3- Найти расстояние ,v.
3.76'	. В ускорителе пучок частиц движется по круговой орбите радиуса г со скоростью и. Средний ток, создаваемый пучком, равен I. Найти заряд пучка.
Рис. 3.75
Рис. 3.77
3.77*	. На однородный стержень, оба конца которого заземлены, падает пучок электронов, причем в единицу времени на единицу длины стержня попадает постоянное число электронов (рис. 3.77). Найти разность потенциалов между серединой стержня (точка Л) и его концом (точка В). Сопротивление стержня равно R. Ток на участке заземле ния DF равен 7.
3.78°	. Провод АС В изогнут так, что точки А, С и В находятся в вершинах правильного треугольника (рис. 3.78). К серединам сторон АС и ВС подключена перемычка EF
А
Рис. 3,78	Рис. 3.79
из провода с вдвое меньшей площадью сечения. К точкам А и В подано напряжение 7/=3 В. Найти падение напряжения на перемычке.
3.79°	. Найти напряжение U между точками Л и В в схеме, изображенной на рис. 3.79. Сопротивление резисторов R и ток 7 известны.
80
3.80°	. Найти напряжение U между точками А и В в схеме, изображенной на рис. 3.80, если на вход схемы подано напряжение 160 В.
3.81.	Резисторы и вольтметр включены в цепь переменного тока, как показано на рис. 3.81. Напряжение между
точками А и А' втрое меньше напряжения между точками В и В'. Найти сопротивление Rx, если сопротивление R известно.
3.82.	Резистор, сопротивление которого равно Rlt и элемент, переменное сопротивление которого R2=R<>2— —cU, где U—падение напряжения на элементе, а с — некоторая константа, включены последовательно в электрическую цепь. Найти ток в цепи, если на вход ее подведено напряжение Uo, а падение напряжения U на элементе неизвестно.
3.83.	Электрический прибор подключен к источнику питания двумя длинными проводами сечения S0=l мм2 каждый. При включении прибора выяснилось, что напряжение на приборе меньше напряжения на выходе источника питания на 10 %. Какой должна быть площадь сечения подводящих проводов той же длины, для того чтобы напряжение уменьшилось только на 1 %?
3.84.	Линия из N ламп, соединенных между собой одинаковыми отрезками проводов, имеющими сопротивление R
Рис. 3.84
(рис. 3.84), подсоединена к источнику тока, напряжение которого равно UQ. Считая, что в результате нагрева нити ток 70> потребляемый лампой, не зависит от напряжения
81
на ней, найти сопротивление провода /?, при котором напряжение на последней' лампе будет составлять 0,9 Ua.
3.85.	В цепь, составленную из источника ЭДС с внутренним сопротивлением г—1 Ом и сопротивления 7?=103 Ом, включается вольтметр, первый раз — параллельно сопротивлению (рис. 3.85 а), второй — последовательно с ним
(рис. 3.856). Показания вольтметра оказались одинаковыми. Найти сопротивление вольтметра.
3.86°	. Три одинаковые батареи подсоединяют к сопротивлению /?=10 Ом, соединив их первый раз параллельно, а второй — последовательно. При этом мощность, выделяемая на сопротивлении, во втором случае в 4 раза превышает мощность, выделяемую в первом случае. Определить внутреннее сопротивление одной батареи.
3.87.	Найти напряжение, которое покажет вольтметр, включенный в схему так, как показано на рис. 3.87. ЭДС источников тока и их внутренние сопротивления равны и <§г, г, и г2. Сопротивление нагрузки равно R.
3.88.	Найти напряжение, которое покажет высокоомный вольтметр, включенный в схему так, как показано на рис. 3.88. Сопротивления резисторов Ri, R2, Rs и и напряжение U0 известны.
3.89.	Металлическая звезда, все ребра которой обладают сопротивлением R, включена в электрическую цепь, как показано на рис. 3.89. Найти отношение количеств теплоты, выделившейся на ребрах BD, ВС, CD, АВ, BE, за одно 82
и то же время. Как изменятся эти отношения, если сопротивление ребра BD сделать равным нулю, а сопротивление ребра CD равным 27??
3.90.	Металлическая звезда включена в сеть, как показано на рис. 3.90. Сопротивление ребра' АС равно нулю,
сопротивление ребра ВС равно 37?, сопротивления остальных ребер равны 7?. Найти ток через ребро АС и мощность, выделяющуюся во всей звезде.
3.91.	Плоский конденсатор, заполненный веществом с диэлектрической проницаемостью е и удельным сопротивлением р, подключен к источнику тока, имеющему ЭДС S и внутреннее сопротивление г. Найти заряд, накопившийся на конденсаторе, если сопротивление утечки конденсатора равно 7?. Сопротивлением проводов пренебречь.
3.92.	В плоский конденсатор с квадратными пластинами вдвигается с постоянной скоростью v металлическая пластина. Конденсатор включен последовательно с резистором,
имеющим сопротивление 7?, и с источником тока, ЭДС которого равна £ (рис. 3.92). Найти установившуюся мощность, выделяющуюся на резисторе. Расстояние между пластина
83
ми конденсатора равно d0. Площадь вдвигаемой пластины равна площади пластин конденсатора 1X1, а ее толщина равна d.
3.93.	Конденсатор емкости С и резисторы, сопротивления которых равны Rx и R2, включены в электрическую цепь, как показано на рис. 3.93. Найти заряд на конденсаторе, если по цепи течет постоянный ток. Напряжение UQ известно.
3.94.	Конденсаторы емкости С\ и С2 и резисторы, сопротивления которых равны Ri, R2, R3, включены в электрическую цепь, как показано на рис. 3.94. Найти установившиеся заряды на конденсаторах. Напряжение t/0 известно.
Рис. 3.94
3.95.	Конденсаторы емкости С1( С2, С3 и резисторы, сопротивления которых равны R, и R2, включены в электрическую цепь, как показано на рис. 3.95. Найти установившиеся заряды на конденсаторах. Напряжение Uo известно.
3.96.	Конденсатор емкости С и резисторы, сопротивления которых равны R, включены в электрическую цепь, как показано на рис. 3.96. Найти установившийся заряд на конденсаторе. Напряжение t/0 известно.
Рис. 3.97
3.97.	Конденсаторы, емкости которых равны С, и резисторы, имеющие сопротивления R, включены в цепь, как показано на рис. 3.97. Найти заряд на заземленной обкладке конденсатора. Напряжение Uo известно.
84
3.98.	1) Заряженный конденсатор емкости С замыкают на резистор, имеющий переменное сопротивление. Найти зависимость сопротивления резистора от времени, если ток через него остается постоянным до полной разрядки конденсатора. Начальное сопротивление резистора равно Ro.
2)	Заряженный конденсатор переменной емкости замыкают на резистор, имеющий сопротивление R. Найти зависимость емкости от времени, если ток в цепи остается постоянным до полной разрядки конденсатора. Начальная емкость равна Со.
3.99°	. Резисторы 1,2,3 включены в электрическую цепь с диодом, как показано на рис. 3.99. Определить мощность, выделяющуюся на резисторе 3. Напряжение источника пе-
Рис. 3.99
ременного тока равно U, сопротивление всех резисторов равно R.
3.100°	. Два сопротивления R, и R2 и два диода подключены к источнику переменного тока с напряжением U так, как показано на рис. 3.100. Найти среднюю мощность, выделяющуюся в цепи.
3.101*	. Найти максимальное падение напряжения на резисторе, имеющем сопротивление R, и долю периода, в
R=100m
Рис. 3.101
течение которой ток в цепи отличен от нуля (рис. 3.101). Построить график зависимости от времени падения напряжения на сопротивлении R. Амплитудное значение напряжения источника переменного тока равно 220 В, а частота равна 50 Гц. Внутренним сопротивлением батереи постоян
85
ной ЭДС можно пренебречь. Решить задачу для двух случаев, когда зависимость тока через диод от приложенного к нему напряжения имеет вид, представленный на рис. 3.101.
3.102.	Через нагревательную спираль, сопротивление которой постоянно, пропускают постоянный ток. На сколько процентов изменится среднее количество теплоты, выделяющейся в спирали в единицу времени, если через спираль пропускать одновременно с постоянным током переменный (синусоидальный) ток, амплитудное значение которого составляет 10 % от силы постоянного тока?
3.103.	Металлический шар, находящийся в вакууме и удаленный от окружающих предметов, заземлен через резистор, имеющий сопротивление /?. На шар налетает пучок электронов, скорость которых вдали от шара равна и, так что на шар попадает nt электронов в единицу времени. Какое количество теплоты выделяется в шаре в единицу времени? Масса и заряд электрона равны т и е.
3.104.	Плоский конденсатор массы М с горизонтально расположенными пластинами подвешен на пружине. При этом удлинение пружины равно Al. Найти дополнительное удлинение пружины, если в конденсатор параллельно его пластинам будет входить пучок электронов, отклоняющихся на выходе из конденсатора на угол а от горизонтали. Сила тока пучка электронов равна I, масса, заряд и начальная скорость электронов равны т. е, v0.
3.105.	Пучок электронов влетает со скоростью в плоский конденсатор параллельно его пластинам (рис. 3.105).
Сколько электронов в единицу времени попадет на положительно заряженную пластину? Пучок имеет прямоугольное сечение, высота которого равна расстоянию между пластинами d, а ширина равна х. Число электронов в единице объема пучка равно nv. Масса и заряд электронов равны т и е. На конденсатор подано напряжение U, длина пластин в направлении движения пучка равна I.
86
Рис. 3.107
3.106*	. Пучок электронов влетает со скоростью v0 в плоский конденсатор, целиком заполняя на входе зазор между пластинами (рис. 3.106). Конденсатор последовательно с резистором, имеющим сопротивление R, подключен к источнику тока, ЭДС которого равна <§. Найти ток через резистор. Расстояние между пластинами равно d, длина пластин в направлении движения пучка равна I, ширина —х. Число электронов в единице объема равно nv, масса и заряд электронов — т и е. Отрицательно заряженная пластина конденсатора заземлена.
3.107.	В вакуумном диоде, анод и катод которого — параллельные пластины, I=cUil2, где с — некоторая постоянная (рис. 3.107). Во сколько раз увеличится сила давления на анод, возникающая из-за ударов электронов о его поверхность, если напряжение на диоде увеличить в два раза? Начальной скоростью электронов, вылетающйх с катода, пренебречь.
§	4. Электромагнитная индукция. Взаимодействие тока с магнитным полем
3.108	°. Четыре одинаковые проволоки длины I каждая, связанные на концах шарнирами, образуют квадрат. Квадрат помещен в однородное магнитное поле индукции В, перпендикулярное его плоскости. Противоположные вершины проволочного квадрата растягивают до тех пор, пока он не превращается в прямой проводник. Ка- _ кой заряд протечет при этом через гальвано-метр, соединенный последовательно с одной /	\
из проволок, если сопротивление каждой i	1
проволоки равно /?/4?	'k J
3.109	. Кольцевой виток с током находится
в переменном магнитном поле, индукция ко- (	\
торого B=Bosin®^ перпендикулярна плос- 4	/
кости витка. Виток превратили в восьмерку, х. J составленную из двух равных колец, не вы-
водя при этом провод витка из плоскости Рис. 3.109 (рис. 3.109). Во сколько раз при этом изме-
нится амплитуда тока в витке? Индуктивностью витка пренебречь.
3.110	°. Кольцевой виток радиуса г, сделанный из проволоки с сопротивлением единицы длины р, находится в по
87
стоянном однородном магнитном поле, индукция которого В перпендикулярна плоскости витка. Виток превратили в восьмерку, составленную из двух равных колец, не выводя при этом виток из его плоскости (см. рис. 3.109 к предыдущей задаче). Какой заряд при этом пройдет по проволоке?
3.11	Г. Проволочное кольцо диаметра d, имеющее сопротивление /?, помещено в переменное однородное магнитное поле, перпендикулярное его плоскости. Магнитная индукция нарастает линейно за время от нуля до значения В и затем линейно уменьшается до нуля за время 12. Какое количество теплоты выделится в кольце?
3.112	*. В ускорителе по окружности радиуса г в магнит
ном поле, перпендикулярном плоскости траектории, двш жется тонкий пучок протонов. Найти силу тока пучка после того, как частицы сделали один оборот. Сила тока пучка в начальный момент равна 70, полное число протонов в камере равно п. Поток индукции магнитного поля через орбиту протонного пучка изменяется с постоянной скоростью (ДФ/А^=^) так, что протоны ускоряются. Масса и заряд протонов равны т и е, их скорость остается далекой от скорости света.
3.113	. Непроводящее кольцо массы т и радиуса г, имеющее равномерно распределенный небольшой заряд q, может свободно вращаться вокруг своей оси. Кольцо поме-
щено в перпендикулярное плоскости кольца магнитное поле,
индукция которого в центральной области кольца радиуса
Кг равна 2В, а
Рис. 3.114
в остальном пространстве внутри кольца равна В. Магнитное поле начинает равномерно уменьшаться до нуля. Какую скорость приобретает кольцо после исчезновения магнитного поля, если в начальный момент оно покоилось?
3.114	*. Проволочное кольцо радиуса R имеет проводящую перемычку,
расположенную вдоль диаметра (рис.
3.114). В левую и правую полуокружности включены конденсаторы емкостями Ci и С2. Кольцо помещено в нарастаю-
щее линейно со временем магнитное поле с индукцией B(t)=Bf)tlT, перпендикулярное его плоскости. В некоторый момент времени перемычку убирают и затем прекращают изменять магнитное поле. Найти установившиеся заряды
на конденсаторах.
3.115	*. Два параллельных идеально проводящих редь-са расположены на расстоянии 7=0,1 м друг от друга в плоскости, перпендикулярной однородному магнитному полю
88
индукции В—1 Тл. Рельсы соединены неподвижным проводником, сопротивление которого Ro=3 Ом, и двумя проводниками, сопротивления которых R0=l Ом и R2~2 Ом, движущимися в разные стороны от неподвижного проводника с ПОСТОЯННЫМИ скоростями U1=O,3 м/с и v2=0,2 м/с. Найти ток, проходящий через неподвижный проводник.
3.116	*. Проводник EF движется с постоянной скоростью V, замыкая два проводника АС и AD, образующих между собой угол а (рис. 3.116). Перпендикулярно плоскости системы проводников приложено постоянное однородное магнитное поле индукции В. Найти полное количество теплоты, выделившееся в цепи за время движения проводника EF от точки А до точки С. Сопротивление единицы длины
проводника EF равно Rt. Сопротивлением проводников АС и AD пренебречь. AC—l0, EF_\_AC, vJ_EF.
3.117	. Проводник OD может скользить по дуге ADC радиуса I (рис. 3.117). Перпендикулярно плоскости дуги приложено постоянное однородное магнитное поле индукции В. Какую силу надо приложить в точке D перпендикулярно проводнику OD, чтобы вращать его с постоянной угловой скоростью со? Сопротивление участка ОС равно R. Сопротивлением остальных проводников пренебречь.
3.118	. Проволочная прямоугольная рамка со сторонами длины It и /2, массы которых равны т* и т2, может свободно вращаться вокруг горизонтальной оси так, что одна из сторон длины li остается неподвижной. Рамка находится В вертикальном однородном магнитном поле индукции В. Найти такую силу тока в рамке, при которой она будет неподвижна в поле тяготения Земли и наклонена к горизонту под углом а.
3.119	. П-образная проволочная рамка подвешена на горизонтальной оси и находится в однородном вертикально направленном магнитном поле с индукцией В (рис. 3.119). Рамка состоит из стержня длины I и массы т и двух невесомых жестких стержней длины h. Через рамку пропускают
89
кратковременный импульс тока силой /0 длительности т. Определить максимальный угол отклонения рамки от первоначального положения. Смещение рамки за время т очень мало.
3.120	. В вертикальном направлении создано однородное постоянное магнитное поле индукции В. Шарик массы т с
Рис. 3.120
Рис. 3.122
зарядом q, подвешенный на нити длины I, движется по окружности так, что нить составляет угол а с вертикалью (рис. 3.120). Найти угловую скорость движения шарика.
3.121	. Горизонтальный проводник массы т и длины I может скользить по двум вертикальным проводящим стержням без нарушения электрического контакта. Стержни разнесены на расстояние I друг от друга и соединены внизу конденсатором емкости С. Перпендикулярно плоскости движения приложено постоянное однородное магнитное поле индукции В. Найти ускорение стержня. Сопротивлением образовавшейся электрической цепи, а также трением пренебречь. Система находится в поле тяготения Земли.
3.122	*. Горизонтальный проводник массы т может скользить без нарушения электрического контакта по двум вертикальным проводящим стержням. Стержни разнесены на расстояние I друг от друга и соединены внизу источником тока, ЭДС которого равна S (рис. 3.122). Перпендикулярно плоскости движения приложено постоянное однородное магнитное поле индукции В. Найти установившуюся скорость, с которой будет подниматься стержень. Сопротивление проводника равно R. Сопротивлением стержней и источника тока, а также трением пренебречь. Система находится в поле тяготения Земли.
3.123	*. В ванну, наполненную ртутью, параллельно противоположным ее торцам опущены электроды, размеры 90
которых совпадают с размерами торцевых плоскостей. Расстояние между электродами I много меньше размеров электродов и равно длине ванны. Перпендикулярно дну ванны приложено постоянное однородное магнитное поле индукции В. Под каким углом к горизонту расположится поверхность ртути, если к электродам приложить напряжение /7? Удельное сопротивление ртути равно у, а ее плотность — р.
3.124	*. МГД-насос представляет собой канал прямоугольного сечения высоты Л=0,1 м, две противоположные
Рис. 3.124
Рис. 3.125
стенки которого проводящие (рис. 3.124). Расстояние между ними /=0,05 м. К проводящим стенкам подводится разность потенциалов U—1 В. Перпендикулярно двум непроводящим стенкам приложено однородное магнитное поле индукции В=0,1 Тл. Нижняя часть канала касается поверхности ртути, верхняя соединена с непроводящей вертикальной трубой. На какую высоту поднимется ртуть? Удельное сопротивление ртути у=10-в Ом-м, плотность р=14Х Х103 кг/м3. Считать, что ускорение свободного падения g=10 м/с2.
3.125	. Небольшое заряженное тело массы т, прикрепленное к нити длины /.может двигаться по окружности в вертикальной плоскости. Однородное магнитное поле индукции В перпендикулярно этой плоскости и направлено, как показано на рис. 3.125. При какой наименьшей скорости тела в нижней точке оно сможет совершить полный оборот? Заряд тела положителен и равен q.
3.126	*. Одна обмотка понижающего трансформатора имеет N витков, другая — один виток. Трансформатор подключен к источнику переменного тока с ЭДС <§. К выходному витку подсоединен гальванометр с внутренним сопротивлением г так, что подсоединения 1 и 2 делят виток на участки с сопротивлением 7? 1 и R2, как показано на рис. 3.126. Какой ток покажет гальванометр? Рассеянием магнитного потока пренебречь.
91
3.127	*. Заряженный конденсатор емкости С замкнут на катушку индуктивности L. Найти такую зависимость от времени емкости конденсатора, при котором ток в цепи нарастает прямо пропорционально времени.
3.128	. Конденсатор емкости С и катушки индуктивности Z-! и L, включены в электрическую цепь, как показано на
Рис. 3.126
Рис. 3.128
рис. 3.128. Найти наибольшую силу тока в цепи, если начальная разность потенциалов на катушках индуктивности равна UQ. Энергия магнитного поля в катушке индуктивности L равна Ыг[2.
Глава IV. ОПТИКА
§ 1.	Отражение. Зеркала
4.1	. В .комнате длины L и высоты Н висит на стене плоское зеркало. Человек смотрит в него, находясь на расстоянии I от той стены, на которой оно висит. Какова должна быть наименьшая высота зеркала, чтобы человек мог видеть стену, находящуюся за его спиной, во всю высоту?
4.2	. Два человека стоят на расстояниях 1± и Z2 от вертикальной стены и на расстоянии Zo друг от друга. Первый громко произносит короткое слово. За какое время он должен произнести это слово, чтобы второй человек услышал конец слова совпадающим с началом эха? Скорость звука в воздухе равна с.
4.3	*. Автомобиль движется со скоростью и мимо длинной стены, удаляясь от нее под углом а. В момент, когда рад-стояние от автомобиля до стены равно Z, шофер подает короткий звуковой сигнал. Какое расстояние пройдет автомобиль до момента, когда шофер услышит эхо? Скорость звука в воздухе равна с.
4.4	°. Вогнутое сферическое металлическое зеркало, направленное на Солнце, сфокусировало свет в точку, распо
92
ложенную на оси зеркала на расстоянии If от его центра. Температура зеркала была при этом равна 4. На каком расстоянии от центра зеркала будет находиться изображение после того, как зеркало нагреется до температуры /2? Полюс зеркала закреплен. Температурный коэффициент линейного расширения металла равен а.
4.5	. Полуцилиндрическое зеркало, поперечное сечение которого представляет собой полуокружность, поместили в широкий пучок света, параллельный оптической оси зеркала. Найти наибольший угол между лучами в отраженном от зеркала пучке света (угол расхождения).
4.6	. Сферу радиуса R посеребрили изнутри и отрезали от нее меньшую часть — сферическое зеркало — так, что плоскость разреза прошла на расстоянии R/2 от центра сферы. В центр окружности, образованной линией разреза, поместили точечный источник света. Найти максимальный угол между световыми лучами, отраженными от сферического зеркала.
4.7	°. На главной оптической оси вогнутого зеркала радиуса кривизны г помещен точечный источник света S. Расстояние от него до зеркала d=(3/4)r. На каком расстоянии х от вогнутого зеркала надо поставить перпендикулярное главной оптической оси плоское зеркало, чтобы лучи от источника S, отраженные сначала одним, а потом другим зеркалом, снова сфокусировались в точке S?
4.8	°. 1) В центре сферического зеркала радиуса г поместили точечный источник света S и, разрезав зеркало пополам по горизонтали, отодвинули верхнюю его половину на расстояние вдоль главной оптической оси (рис. 4.8). Найти расстоя- S'
ние между изображениями источ- з	S
ника света.	----------—А. —
2) В центре сферического зерка- к  г ла расположен точечный источник	%,
света S. Зеркало разрезали попо-
лам. Обе половины симметрично	Рис. 4.8
отодвинули на расстояние h от
главной оптической оси целого зеркала. Найти расстояние между изображениями источника света в зеркалах.
4.9°. При прохождении потока нейтронов через пластинку кадмия толщины 1 мм число частиц уменьшилось на 15 %, а их скорость не изменилась. Какая доля потока нейтронов пройдет через пластинку кадмия толщины 8 мм?
4.10. Пучок света интенсивности 10 падает на плоскопараллельную пластину по нормали к ее поверхности. Най-
93
тп интенсивность пучка, прошедшего сквозь пластину. Коэффициент отражения света на каждой поверхности пластины в любую сторону равен Поглощением света в пластине пренебречь.
§	2. Преломление
4.11°	. Луч света падает под углом а на стопку плоских прозрачных пластин одинаковой толщины, показатель преломления каждой из которых в k раз меньше, чем у вышележащей. При каком наименьшем угле падения луч не пройдет сквозь стопку? Показатель преломления верхней пластины равен п, число пластин равно N.
4.12.	Свет от источника, висящего над аквариумом, падает конусом с углом раствора 2а. Аквариум высоты ha заполнен водой, на нем, касаясь воды, лежит плоское стекло толщины h. На сколько изменится радиус светового пятна на дне аквариума, если убрать стекло и слить воду?3 Показатели преломления стекла и воды равны п и п0 (п0<2 <Zn).
4.13.	Трубу радиуса г0, по которой распространяется свет, замыкает шар того же радиуса. Внутренняя сферическая область шара радиуса г<г0 заполнена веществом, поглощающим свет. Оболочка поглощающей области сделана из стекла с показателем преломления п. Какая доля энергии света проходит сквозь шар? Центры шара и его поглощающей свет области совпадают. Отражением света от оболочки пренебречь.
4.14.	Каким должен быть внешний радиус изгиба световода, сделанного из прозрачного вещества с показателем
преломления п=4/3, чтобы при диаметре световода, равном d=l мм, свет, вошедший в световод перпендикулярно плоскости сечения (рис. 4.14), распространялся, не выходя наружу через боковую поверхность?
4.15.	В веществе с показателем преломления и==1,3 распространяется узкий параллельный световой пучок. Сечение пучка — круг (рис. 4.15). Этот пучок встречается
94
со сферической полостью (пустотой), диаметр которой совпадает с его осью и существенно превышает диаметр пучка. Во сколько раз пучок будет шире на выходе из полости?
4.16*	. В боковой стенке сосуда, наполненного жидкостью с показателем преломления п, проделано небольшое отверстие радиуса г, через которое вытекает струя. По оси отверстия горизонтально направляется тонкий луч света. При какой высоте h уровня жидкости над отверстием луч света сможет выйти из струи, ни разу не испытав полного внутреннего отражения? Считать показатель преломления п достаточно большим. Изменением поперечного сечения струи пренебречь.
4.17*	. На плоскости зачернен круг радиуса го=О,2 м. Стеклянный конус упирается вершиной в центр круга так, что его ось перпендикулярна плоскости. Каков видимый радиус круга, если на него смотреть с большого расстояния вдоль оси конуса? Угол при вершине конуса 2а=60°, радиус основания г=га=0,2 м, показатель преломления стекла п > К2«1,4.
4.18.	Узкий пучок света, пройдя через стеклянное полушарие с показателем преломления п параллельно его оси симметрии, собирается на расстоянии х от выпуклой поверхности. На каком расстоянии от плоской поверхности полушария соберется луч, если пустить пучок с обратней стороны? Считать, что углы между лучами света и главной оптической осью малы *).
4.19.	На каком расстоянии от стеклянного шара радиуса г следует поместить точечный источник света S, чтобы его изображение S' оказалось с другой стороны от шара на таком же расстоянии? Показатель преломления стекла равен п. Изображение создается узким пучком лучей, близких к оптической оси.
4.20.	В полый цилиндрический сосуд, закрытый с обеих сторон плосковыпуклыми линзами, налита вода. Главные оптические оси линз совпадают с осью цилиндра. Вдоль оси на линзу падает параллельный пучок света. При каком расстоянии между линзами свет выйдет из устройства параллельным пучком? Показатель преломления воды равен п. Фокусные расстояния линз в воздухе равны F. Считать, что углы между лучами света и главной оптической осью малы.
*) При решении этой и следующих задач следует учесть, что если угол а мал, то sin a=s:tg a=sa,
95
4.21. Внутри стеклянного шара радиуса г=0,1 м слева от его центра вблизи поверхности находится точечный источник света S (рис. 4.21). На каком расстоянии справа от
центра шара радиус светового пучка, вышедшего из шара, будет равен г? Показатель преломления стекла и=2.
4.22. Внутри стеклянного шара радиуса г находится точечный источник света S, излучающий свет равномерно во
псе стороны. Шар пропускает в окружающее пространство (воздух) половину светового потока, исходящего отточки S.
Найти расстояние от точки S до центра шара. Поглощением света в шаре пренебречь. Показатель преломления стекла /г=1,5.
§ 3. Линзы
Рис. 4.23
4.23°. Две собирающие линзы с фокусными расстояниями F-l— 1 см и /72=5 см имеют общую оптическую ось (рис. 4.23). Расстояние между ними равно сумме фокусных расстояний. На первую линзу падает вдоль оптической оси параллельный пучок света ширины d=l см. Каким будет размер пятна на экране, расположенном на некотором расстоянии, например Fi+F2, от второй линзы?
4.24.	Вдоль главной оптической оси собирающей линзы
с фокусным расстоянием F=5 см движутся навстречу друг
другу два светлячка, находящихся по разные стороны линзы. Скорость светлячков одна и та же — у=2 см/с. Через какое время первый светлячок встретится с изображением второго, если в начальный момент они находились на расстояниях /1=20 см и /2=15 см от линзы?
4.25.	Фотографируя муравья с близкого расстояния, экспериментатор использовал удлинительное кольцо, которое переместило объектив фотоаппарата на расстояние 7=7,5 мм вперед, т. е. расстояние от пленки до объектива увеличилось на 7,5 мм. Резкое изображение теперь получалось, если на шкале расстояний объектива устанавливалось значение расстояния й=1,05 м. Фокусное расстояние объектива К=50 мм. Определить, на каком расстоянии от объ-
96.
ектива был муравей. Считать объектив тонкой линзой. Расстояния Ь, обозначенные на шкале, указывают значения расстояний от предмета до объектива без использования удлинительных колец.
4.26°	. Точечный источник света расположен на главной оптической оси собирающей линзы с круглой диафрагмой на расстоянии I от линзы. С другой стороны линзы расположен экран в месте резкого изображения источника света. Найти радиус светового пятна на экране, если источник
света отодвинуть в направлении от линзы еще на расстояние d (рис. 4.26). Радиус диафрагмы линзы равен R, фокусное расстояние равно F.
4.27°	. Четкое изображение лампы на экране возникает при двух положениях линзы, помещенной между ними. Найти фокусное расстояние линзы. Расстояние между двумя положениями линзы равно /, между лампой и экраном — L.
4.28.	1) Полая двояковогнутая стеклянная линза помещена в воду. Фокусное расстояние линзы в воде равно F. Найти длину изображения стрелки, расположенной на оптической оси между точками, отстоящими от линзы на расстояния F/4 и 3F/i. Что будет с изображением, если центральную часть линзы закрыть непрозрачным кружком, площадь которого составляет половину площади поверхности линзы? Внутри полости линзы находится воздух, стенки ее тонкие. Решение обосновать построением.
2)	Полая двояковыпуклая стеклянная линза помещена в воду. Найти длину изображения стрелки, расположенной на оптической оси вплотную к линзе, если длина самой стрелки равна фокусному расстоянию линзы F. Внутри полости линзы находится воздух, стенки ее тонкие. Решение обосновать построением.
4.29.	На поверхности жидкости, налитой в стакан (рис. 4.29), плавает плосковыпуклая линза с фокусным расстоянием F. Найти высоту h жидкости в стакане, если изображе-
4 Г. В. Меледии	97
ние точечного источника света S, расположенного на расстоянии L от линзы на ее оси, находится на дне стакана. Показатель преломления жидкости равен п. Расстояние L много больше диаметра.
4.30.	Полая двояковогнутая тонкостенная стеклянная линза помещена в воду на глубину h так, что ее главная оптическая ось перпендикулярна поверхности воды. Снизу
Рис. 4.29
на линзу направлен узкий параллельный пучок света. Пучок распространяется вдоль главной оптической оси. Найти расстояние от линзы, на котором соберутся лучи света. Фокусное расстояние линзы в воду равно F (причем F>h). Показатель преломления воды равен п. Считать, что углы между лучами и главной оптической осью малы.
4.31.	На главной оптической оси 00' собирающей линзы расположено плоское зеркало, вращающееся с угловой скоростью <о вокруг оси, проходящей через точку А и перпендикулярной плоскости рисунка (рис. 4.31). На зеркало падает параллельный пучок лучей, который после отражения фокусируется на экране. Найти мгновенную скорость
светового пятна на экране в момент, когда оно проходит точку В, находящуюся на главной оптической оси. Плоскость экрана перпендикулярна этой оси. Фокусное расстояние линзы равно F.
4.32. Стеклянный конус, собирающая линза и экран
расположены так, как показано на рис. 4.32. Главная оптиче-
Рис. 4.32
ская ось линзы совпадает с осью конуса и перпендикулярна экрану. Расстояние между экраном и линзой равно фокусному расстоянию линзы F. Вдоль оптической оси на конус падает слева узкий параллельный пучок света. Найти фор
98
му и наибольший размер пучка на экране. Показатель преломления стекла равен п. Угол между образующей конуса и его основанием равен а. Угол а мал.
4.33*	. Вершину конуса с углом раствора 2а рассматривают через лупу, имеющую фокусное расстояние F и расположенную от нее на расстоянии d(d<F). Найти видимый через лупу угол раствора конуса. Главная оптическая ось линзы проходит через ось симметрии конуса.
4.34*	. При фотографировании удаленного объекта перед фотоаппаратом на тройном фокусном расстоянии от объектива поместили тонкую линейку длины 2/. Плоскость линейки перпендикулярна оптической оси фотоаппарата. Объектив — тонкая линза диаметра 2г<2/. Найти длину тени на пленке, т. е. размер области, в которой линейкой будут затенены все лучи, исходящие от фотографируемого объекта.
4.35.	Жука фотографируют в двух масштабах, поднося фотоаппарат на расстояние d, равное сначала тройному, а затем пятикратному фокусному расстоянию объектива. Во сколько раз надо изменить диаметр диафрагмы объектива, чтобы освещенность изображения на пленке в обоих случаях была одинаковой? Считать, что диаметр объектива в обоих случаях много меньше d.
4.36*	. При фотографировании Луны с Земли с помощью объектива, имеющего фокусное расстояние F, получено
нечеткое изображение Луны в виде круга радиуса л. Четкое изображение Луны должно иметь радиус г2. На какое расстояние нужно сместить фотопластинку, чтобы изображение получилось четким? Диаметр линзы равен D. Областью изображения считать ту область, куда попадает хотя бы один луч от Луны. Дифракцию света на диафрагме объектива не учитывать.
4.37.	Параллельный пучок света рассеивается, проходя сквозь пластинку (рис. 4.37). Максимальный угол отклоне-
4*	99
ния лучей от первоначального направления для каждой точки пластинки равен а. Какой наименьший радиус светлого пятна можно получить, поставив за пластинкой собирающую линзу, имеющую фокусное расстояние F?
4.38.	Плоскую поверхность плосковыпуклой линзы, фокусное расстояние которой равно F, посеребрили. Найти фокусное расстояние получившегося зеркала. Свет падает со стороны стекла.
4.39.	Две одинаковые собирающие линзы 1 и 2, имеющие каждая фокусное расстояние F, расположены так, что их главные оптические оси составляют угол а и главная оптическая ось первой линзы проходит через центр второй линзы (рис. 4.39). На главной оптической оси первой линзы на расстоянии F расположен точечный источник света S. Найти расстояние между источником света и его изображением в паре линз. Расстояние между центрами линз равно I.
4.40.	Две одинаковые собирающие линзы 1 и 2, имеющие каждая фокусное расстояние F, расположены так, что их главные оптические оси образуют угол а и главная оптическая ось второй линзы проходит через центр первой (рис. 4.40). В фокусе первой линзы расположен точечный источник
Рис. 4.41
света S. Найти расстояние между источником света и его изображением в паре линз. Расстояние между центрами линз равно 2F.
4.41.	Две собирающие линзы с одинаковыми фокусными расстояниями F расположены на расстоянии F друг от друга (рис. 4.41). Оптическая ось первой линзы параллельна оптической оси второй линзы и находится на расстоянии h от нее. Найти расстояние между точечным источником света S, расположенным на расстоянии 2F от первой линзы на ее главной оптической оси, и его изображением в паре линз.
4.42.	Слева от собирающей линзы, имеющей фокусное расстояние F, на расстоянии 5F/3 находится непрозрачный экран с круглым отверстием диаметра Da—2 см, а справа 100
fT Нее на расстоянии 3F/4 — плоское зеркало (рис. 4.42). 'кран и зеркало перпендикулярны главной оптической оси ;линзы, которая проходит через центр отверстия. Слева перпендикулярно экрану падает параллельный пучок света. Найти диаметр светлого пятна на экране.
Рис. 4.42
4.43*	. Два плоских зеркала образуют двугранный угол, равный 90°. В угол вставлена собирающая линза, имеющая фокусное расстояние F, так, что ее главная оптическая ось составляет угол 45° с каждым зеркалом (рис. 4.43). Радиус
Рис. 4.44
линзы r=F. На главной оптической оси линзы на расстоя»' нии d=l,5 F находится источник света S. Найти положение одного из изображений источника света, которое находит-, ся на главной оптической оси.
4.44.	Найти изображения точечного источника S, создаваемые системой из линзы с фокусным расстоянием F и конического зеркала с углом при вершине, равным 90° (рис. 4.44). Ось конуса совпадает с осью линзы. Расстояние меж-, ду вершиной конуса и линзой равно 2F, между источником S и линзой равно 3F/2.
4.45.	Оптическая система состоит из собирающей линзы, имеющей фокусное расстояние F, и вогнутого зеркала радиуса R, расположенных на расстоянии d друг от друга так, что их оптические оси совпадают (рис. 4.45). На каком
101
расстоянии от линзы на оптической оси должен находиться точечный источник света S, чтобы его изображение совпадало с самим источником?
4.46.	Оптическая система состоит из собирающей линзы с фокусным расстоянием F и зеркального шарика радиуса R, центр которого находится на оптической оси линзы на
Рис. 4.46
расстоянии d от нее (рис. 4.46). Определить расстояние от линзы до точечного источника S, расположенного на оптической оси системы, при котором изображение источника совпадает с самим источником.
§ 4. Фотометрия
4.47°	. Светящаяся лампочка висит на высоте h над столом. На каком расстоянии от точки стола, находящейся непосредственно под лампой, освещенность стола уменьшается в п раз?
4.48.	Два плоских зеркала образуют двугранный угол, равный 90°. Точечный источник света S помещен внутри угла так, что расстояния от него до вертикальной и горизонтальной граней угла равны I и 21 (рис. 4;48). На расстоянии
Рис. 4.48
21 от вертикального зеркала параллельно ему расположен экран. Найти освещенность в точке экрана, расположенной на расстоянии I от горизонтального зеркала. Сила света источника равна J.
102
4.49.	Экран освещается солнечным светом, падающим перпендикулярно его плоскости. Как изменится освещенность экрана, если на пути света поставить стеклянную призму с углом при вершине а (рис. 4.49)? Грань, на которую падает свет, параллельна плоскости экрана. Показатель преломления стекла равен п. Считать, что отражения света от граней призмы нет.
4.50*	. Осколком плоского зеркала А неправильной формы пускают солнечный зайчик в точки Ви Сна вертикальной стене (рис. 4.50). Зайчик в точке В оказывается круглым, а освещенность в его центре втрое больше, чем на участках
Рис. 4.51
стены, освещенных только рассеянным светом. Какова освещенность в центре зайчика, попавшего в точку С? Лучи 5Л, АВ и АС лежат в одной вертикальной плоскости, луч АВ горизонтален. ^/5АВ=^/ВАС=45°.
4.51.	Перед сферическим зеркалом радиуса г, в фокусе которого находится точечный источник света S, на расстоянии I от источника помещена небольшая пластинка, плоскость которой перпендикулярна оптической оси зеркала (рис. 4.51). Найти отношение освещенностей левой и правой сторон пластинки. Высота, на которую поднята над оптической осью пластинка, равна h.
4.52.	Во сколько раз освещенность в лунную ночь в полнолуние меньше, чем в солнечный день? Высота Луны и Солнца над горизонтом одинакова. Считать, что Луна рас-, сеивает в среднем часть т]=0,07 солнечного света, падающего на нее, равномерно по всей полусфере. Расстояние от Луны до Земли /=4-106 км, радиус Луны r=2-103 км.
103
Глава V. ЗАДАЧИ-ОЦЕНКИ
Задачи-оценки — новый для большинства школьников и абитуриентов класс задач.
Для решения задачи надо понять рассматриваемое физическое явление, сформулировать простую (так как нужна только оценка) физическую модель этого явления, выбрать разумные значения физических величин и, наконец, получить числовой результат, более или менее соответствующий реальности.
Хорошо представляя явление или задачу, каждый сам может выбрать необходимые для решения величины и их числовые значения. Естественно, что спектр возможных числовых ответов широк (приводимые в задачнике — некоторые из возможных), а сам подход к получению ответа необычен, непривычен для школьника (в школах с такими задачами, как правило, не знакомят). Между тем грубая прикидка, оценка по порядку величины — почти обязательный этап начальной постановки эксперимента, проектирования установки, теоретической разработки, контроля за правильностью рас-суждений и выводов в процессе обсуждения сложных идей. Оценки иногда подсказывают путь точного решения задачи, дают возмог;-ность просто установить границы области применимости точного решения. Владение методом оценок, наряду с интуицией, является очень важным качеством исследователя при разработке и анализе новых идей,’ весьма существенным в творческой работе. Надо полагать, что способность решать задачи-оценки должна входить в ряд критериев при отборе претендентов на исследовательскую работу и, в частности, для учебы на физических факультетах вузов.
5.1.	Оценить, на сколько дальше спортсмен бросит гранату, если будет бросать ее с разбега.
5.2.	Оценить, с какой скоростью может бежать по Луне космонавт в легком, удобном скафандре.
5.3.	Оценить время вытекания воды из заполненной ванны.
5.4.	Пассажир летящего на высоте 10 км самолета видит восходящее солнце. Оценить, через какое время увидит солнце наблюдатель, стоящий на земле под самолетом.
5.5.	Оценить число оборотов (кувырков), которые совершит автомобиль, свободно упавший в километровую пропасть на полной скорости.
5.6.	Летящий горизонтально сверхзвуковой самолет внезапно встречается с препятствием, которое возвышается на 100 м над его траекторией. Оценить минимальное расстояние от препятствия, на котором летчик должен начать набирать высоту, чтобы избежать столкновения.
104
5.7.	На сколько и в какую сторону от вертикали откло-Shtch в высшей точке полета артиллерийский снаряд, запу-1«нный на экваторе в вертикальном направлении?
5.8.	Оценить размеры дирижабля, заполненного гелием. Грузоподъемность дирижабля равна 100 т.
5.9.	Оценить глубину ямы на поверхности воды под вер-толетом, зависшим над озером на небольшой высоте.
5.10.	Каким станет давление атмосферы, если вся вода в океанах испарится?
5.11.	Оценить давление воздуха в шахте, глубина которой равна 10 км.
5.12.	С какой наименьшей скоростью можно ехать на водных лыжах?
5.13.	Оценить силу натяжения цепи велосипеда при ез-
де в гору.
5.14. Лесоруб ударил топором по чурбаку. Топор застрял, войдя в чурбак наполовину (рис. 5.14). Оценить силу, с которой сжимается лезвие топора.
5.15.	Оценить силу натяжения бельевой веревки, на которой висит мокрое белье.
5.16.	С какой минимальной частотой человек должен вращать ведро с водой в вертикальной плоскости,чтобы вода не выливалась?
5.17.	Гимнаст делает на переклади-
не оборот — «солнышко». С какой
силой он действует на перекладину в момент, когда про
ходит нижнее положение?
5.18. Продолжим мысленно прямую, проходящую вдоль отвеса (нити с висящим на ней грузиком). Оценить для средних широт, на каком расстоянии от центра Земли пройдет
эта прямая.
5.19.	На сколько отличаются расстояния от уровня мирового океана до центра Земли на полюсе и на экваторе?
5.20.	Оценить среднюю плотность вещества Солнца.
5.21.	Оценить скорость опускания парашютиста с раскрытым парашютом.
5.22.	Оценить, при какой минимальной скорости ветра он может опрокинуть автобус.
5.23.	При какой наименьшей скорости велосипедист может перевернуться через голову вместе с велосипедом, если переднее колесо застрянет, провалившись в щель?
5.24.	Рассеянный велосипедист не заметил, как случайно наехал на вертикальную стенку. Оценить, при какой мини
105
мальной скорости шина при ударе деформируется до металлического обода.
5.25.	Оценить силу натяжения ремней безопасности, удерживающих человека в автомобиле, если автомобиль, движущийся со скоростью ц«30 км/ч, столкнулся со столбом, в результате чего у автомобиля появилась вмятина глубины /якЗО см.
5.26.	Оценить изменение значения силы давления передних колес грузовика на дорогу при переходе от равномерного движения к экстренному торможению.
5.27.	Оценить мощность, выделяющуюся в виде теплоты при экстренном торможении грузовика.
5.28.	Оценить, какую мощность развивает велосипедист на финише.
5.29.	Оценить работу силы сопротивления воздуха при пролете каплей дождя расстояния 1 м вблизи поверхности земли.
5.30.	Оценить отношение мощности, развиваемой кузнечиком при прыжке, к его массе.
5.31.	Пятью ударами молотка гвоздь забили в деревянную стену. Какую силу нужно приложить к шляпке гвоз-дя, чтобы выдернуть его?
'5.32. Оценить среднее усилие, развиваемое ногами человека при приземлении после прыжка из окна второго этажа.
5.33.	Оценить усилие спортсмена при толкании ядра.
5.34.	Человек случайно наступил на лежащие вверх зубьями грабли. Оценить скорость ручки граблей, ударяющей его по лбу.
5.35.	Оценить давление шариковой ручки на бумагу при письме.
5.36.	Длинная труба, закрытая снизу гладким поршнем, на котором помещен снаряд, опущена в озеро Байкал. До какой максимальной скорости сила давления воды может разогнать снаряд?
5.37.	В корпусе подводной лодки образовалось небольшое отверстие. Оценить скорость бьющей из него струи воды.
5.38.	Оценить давление газов в стволе ружья, возникающее при выстреле. Ружейная пуля при вылете из ствола имеет скорость около 8-Ю2 м/с.
5.39.	Оценить мощность, развиваемую при ударе свинцовой пули из малокалиберной винтовки о стальную плиту.
5.40.	С какой скоростью летела капля воды, если при ударе о неподвижную стену она оказывает на нее силу давления около IO1 Па?
106
5.41.	Оценить давление в центре Земли.
5.42.	Оценить наибольшее давление в полной цистерне машины для поливания улиц при ее торможении. При скорости пл<30 км/ч тормозной путь м.
5.43.	Оценить скорость пуль, вылетающих из патронов, брошенных в костер. Скорость пули при стрельбе из ружья равна приблизительно 800 м/с.
5.44.	Оценить, какую массу воды можно нагреть до кипения, если энергию, равную всей кинетической энергии молекул воздуха в учебной аудитории, передать воде, находящейся при комнатной температуре.
5.45.	Оценить скорость, которую приобрели бы осколки закупоренной бутылки с водой, если бы взаимодействие между молекулами воды внезапно исчезло.
5.46.	Представьте, что в какой-то момент у всех молекул воздуха, находящихся внутри лежащего на земле футбольного мяча, скорость оказалась бы направленной вертикально вверх. На какую высоту взлетел бы мяч? 
5.47.	Какая часть атмосферного кислорода Земли израсходуется при сжигании двух миллиардов тонн угля? (Эта цифра близка к данным о мировой добыче угля за год.)
5.48.	Оценить выталкивающую силу, действующую на человека со стороны воздуха в комнате.
5.49.	Какая масса воздуха уйдет из аудитории при повышении в ней температуры на 10 К?
5.50.	Осветительная лампочка имеет тонкостенный непрочный баллон и заполняется инертным газом. Из каких соображений выбирается давление газа? Оценить это давление.
5.51.	В стакан положили кусочек льда массы т=100 г и накрыли плотно прилегающей крышкой. Оценить силу, которая потребуется, чтобы оторвать крышку от стакана сразу по всему периметру после растаивания льда.
5.52.	Оценить силу, необходимую для того, чтобы оторвать от спины хорошо поставленную медицинскую банку.
5.53.	Оценить, на каком расстоянии человек в яркой одежде, уходя в сосновый лес, потеряется из виду (подлеска пет).
5.54.	Оценить время соприкосновения ноги футболиста с мячом при сильном ударе.
5.55.	Оценить среднее усилие, которое нужно приложить к краю надувного матраца, чтобы согнуть этот матрац посредине под углом 90°. Матрац не имеет поперечных перетяжек и хорошо надут.
107
5.56.	Оценить изменение давления в парной после того, как на раскаленные камни плеснули воду из ковша. (При оценке считать, что двери и окна в парной плотно закрыты.)
5.57.	Дробинка попала в канал шланга автомобильного насоса (рис. 5.57). Оценить скорость, которую она может приобрести при сильном нажатии на ручку <===	насоса.
5.58.	Оценить скорость воздуха, с которой можно выдувать его ртом через трубку.
5.59.	Оценить время уменьшения в два раза давления газа в спутнике Земли из-за °®?	сантиметровой пробоины в его стенке.
5.60.	За сколько ходов велосипедного на-_______ coca можно накачать футбольный мяч?
,ч „----5.61. Детский воздушный шарик наду-
Рис. 5.57 вают горячим воздухом. При какой температуре воздуха в нем шарик будет подниматься?
5.62.	Оценить максимальный радиус полости, образующейся при подводном взрыве на глубине 1 км заряда взрывчатого вещества массы 1 т. Энергия взрыва 1 г взрывчатого вещества равна примерно 4 кДж.
5.63.	Оцените расстояние между центрами ближайших светочувствительных элементов вашего глаза.
5.64.	Как известно, предельный размер детали на кинопленке определяется размерами кристалликов светочувствительного слоя. С уменьшением кристалликов четкость изображения на кинокадре растет. Оценить предельный размер кристалликов, при котором кинозритель перестает замечать рост четкости изображения на экране кинозала.
5.65.	Оценить, на каком расстоянии железнодорожные рельсы кажутся слившимися.
5.66.	Оцените размер изображения человека, стоящего в аудитории у доски, на сетчатке вашего глаза.
5.67.	Во сколько раз собирающая линза от очков увеличивает на единицу площади мощность света, приходящего от Солнца?
5.68.	Оценить размеры неоднородностей дорожки звукозаписи на долгоиграющей пластинке.
Глава VI. ЗАДАЧИ-ДЕМОНСТРАЦИИ
В задачах-демонстрациях надо объяснить физическое явление, которое демонстрируется в аудитории на экзамене. Здесь важно понять сущность явления и среди различных факторов выделить и указать главный.
108
В задачнике этот новый для экзаменов класс задач превратился в набор качественных задач по физике. Однако при желании практически все задачи легко воспроизводятся в виде демонстрации в школьном кабинете физики и тем более в вузовском демонстрационном кабинете. Некоторые из задач несложно «оживить» и дома.
6.1.	Объяснить, почему гвозди, повисшие рядом на магните, отклоняются от вертикального положения.
6.2.	Объяснить, почему гвоздь вытаскивается легче, если его согнуть и тащить, поворачивая при этом вокруг оси.
6.3.	Объяснить изменения характера отскоков подвешенного на нити упругого шарика после ударов о круглое препятствие при перемещениях последнего по горизонтали (рис. 6.3).
6.4.	Скатившись с горки, шарик 1 налетает на два таких же шарика 2 и 3, стоящих рядом. Крайний слева шарик 3
Рис. 6.3
Рис. 6.4
после этого откатывается, а шарики 1 и 2 стоят. Опыт видоизменяется так, что теперь шарик 1 налетает не на шарик 2, а на довольно толстую резиновую прокладку, стоящую между ними (рис. 6.4). При ударе через прокладку шарики 2 и 3 откатываются, причем оба вместе. Объяснить различие в поведении шариков в первом и во втором случаях.
6.5.	Грузик на нити подвешен к подставке, которая может скользить по столу. Грузик раскачивают, дергая за нить только в одном направлении. В первом случае движение маятника из отклоненного положения происходит свободно (рис. 6.5а). Во втором случае движение маятника ограничено препятствием, жестко соединенным с подставкой (рис. 6.56). Объяснить, почему в этих случаях различен характер движения подставки с маятником.
109
6.6.	На наклонной плоскости находится брусок, к которому прикреплена нить, перекинутая через проволочную Петлю. На нижнем конце нити закреплен грузик. Свободно висящий грузик не вызывает движения бруска. Если же
проворачиваются,
Рис. 6.8
грузик качнуть, то брусок начинает двигаться. Объяснить явление.
6.7.	На наклонной плоскости на равных расстояниях друг от друга размещены ролики. Крайние ролики легко средний закреплен. Когда брусок кладут на средний и нижний ролики, он съезжает. Когда его кладут на верхний и средний — он не съезжает. Объяснить явление.
6.8.	Гильза 1 может надеваться на пластмассовый стержень 2, сквозь который пропущены провода электрической цепи 3 для поджигания горючей смеси 4 (рис. 6.8). Сначала зажигание смеси демонстрируется без гильзы. Затем надевается гильза. В первом случае стержень закрепляется на легкой проволочке — пружинке 5. Смесь поджигается,
гильза взлетает. Во втором случае стержень закрепляется не на пружинке, а на массивном штативе. После зажигания гильза взлетает значительно выше. Объяснить эффект.
6.9.	Шланг, подсоединенный снизу к сосуду с водой, заканчивается двумя одинаковыми отверстиями. Если открыты оба отверстия, то бьют два фонтанчика одинаковой в&соты. Если же закрыть одно из отверстий, то высота второго фонтанчика сначала заметно увеличивается, а потом уменьшается, оставаясь все же выше первоначальной. Объяснить явление.
6.10.	Если в герметичный цилиндр вставить поршень, способный почти без трения скользить по цилиндру, и прикрепить к нему груз, то поршень сравнительно легко выскальзывает из цилиндра. Е&ли же налить в цилиндр воду, 110
то поршень держится несмотря на значительный вес груза. Объяснить явление.
6.11.	В герметически закрытом сосуде, частично заполненном водой, плавает игрушка — «водолаз». В сосуд вставляют трубку. Если в трубку подуть, «водолаз» тонет. Если перестать дуть в трубку, оставив ее открытой, то «водолаз» всплывает, а иногда так и остается под водой. Объяснить поведение игрушки.
6.12.	Пламя горелки коптит. Если поднести сверху вертикальную стеклянную трубку, копоть пропадает, однако появляется снова, если трубку закрыть сверху. Объяснить явление.
6.13.	Две одинаковых бутылки почти полностью заполнены водой. В одну из них вставляют доходящую до дна трубочку, верхнее отверстие которой закрыто пальцем. Бутылки одновременно переворачивают и в тот же момент открывают отверстие трубочки. Из бутылки с трубочкой вода вытекает заметно быстрее. Объяснить явление.
6.14.	В сосуде, закрытом подвижным поршнем, находятся газ и вата, смоченная ацетоном. Поршень поднимают. Объяснить характер изменения давления в сосуде. Давление измеряется жидкостным манометром.
6.15.	В холодную колбу наливают горячую воду и на горлышко надевают воздушный шарик. Если взболтать воду, шарик сначала раздувается, а потом опадает. Если снова взболтать колбу, эффект раздувания опять повторяется, хотя и слабее. Объяснить явление.
6.16.	Два одинаковых сосуда нагреваются до одной и той же температуры горячей водой. При этом водой ополаскивают один сосуд, а другой остается сухим. Затем оба сосуда ставятся на холодную воду, оба горлышком вниз. После их остывания уровни воды, втянутой в сосуды, заметно различаются. Объяснить явление.
6.17.	Колбу с закипевшей водой снимают е плитки. Кипение прекращается. Колбу плотно закрывают резиновой пробкой с продетым сквозь нее массивным медным стержнем. Вода закипает. Объяснить явление.
6.18.	В металлический сосуд с подкрашенной жидкостью введена тонкая стеклянная трубка так, что столб жидкости виден в выступающей части трубки. Если поставить сосуд на разогретую плитку, то столбик жидкости в трубке сначала опустится вниз, а потом пойдет вверх. Объяснить явление.
6.19.	Объяснить, почему прекращается вращение наДО" дящейся под постоянным напряжением Z-образной вертуШ
1Н
ки, если на эту вертушку надеть проволочный каркас, имеющий с ней электрический контакт.
6.20.	Между пластинами плоского конденсатора подвешен легкий металлический шарик. При подаче на конденсатор постоянного напряжения шарик притягивается к одной из пластин, касается ее и отскакивает к другой пластине. Такие перескоки повторяются многократно. Объяснить явление.
6.21.	Между заземленным и заряженным металлически-’ ми шарами на изолированной нити висит незаряженный металлический шарик. Объяснить его поведение при перемещении заряженного шара.
6.22.	Проводящее колесо может свободно вращаться вокруг своей оси, расположенной вертикально. Между центром колеса и его ободом имеется постоянная разность потенциалов. Если к колесу поднести магнит, оно начинает вращаться. При смене полюса магнита направление вращения изменяется на противоположное. Объяснить явление.
6.23.	Железный стержень вставлен в катушку. На стержень надевают кольцо. В первом случае это кольцо алюминиевое сплошное, а во втором — алюминиевое с разрезом. При пропускании переменного тока через катушку кольца ведут себя существенно по-разному. Объяснить явление.
6.24.	Две магнитные катушки, включенные последовательно и питаемые переменным током, подвешены близко друг к другу так, что их оси совпадают. Между катушками поочередно вставляются медный и железный листы. Почему притягивавшиеся катушки начинают отталкиваться, когда между ними вставлен медный лист? Почему картина меняется, если вместо медного листа вставить железный?
6.25.	Катушка из медного провода может колебаться между полюсами постоянного магнита. Как будут различаться колебания при разомкнутых и закороченных концах катушки? Объяснить наблюдаемые явления.
6.26.	На лабораторном столе стоят два динамика (источники звука). Сначала включают, а затем выключают один динамик. Потом включают и выключают другой динамик. На слух громкость и высота тона обоих динамиков одинаковы. Когда оба динамика включают вместе, слышны медленные колебания громкости звука. Почему возникают эти колебания?
6.27.	Как объяснить изменение громкости звучания струны, по которой течет переменный ток звуковой частоты, при перемещении вдоль нее магнита?
1Й
6.28.	В цилиндрической трубе возбуждаются гармонические звуковые колебания определенной частоты с помощью динамика /, установленного на одном из ее концов (рис. 6.28). Длина трубы при перемещении поршня 2 изменяется. На боковой стенке трубы находится микрофон 3, сигнал с которого через усилитель 4 подается на динамик 5. Объяснить, почему при перемещении поршня изменяется громкость звукового сигнала динамика 5. Можно ли этот эксперимент использовать для определения параметров колебательного процесса в трубе?
Рис. 6.28
Рис. 6.29
6.29.	От генератора 1 звуковых частот гармоническое напряжение подается на динамик 2 и на горизонтально отклоняющиеся пластины электронно-лучевой трубки (ЭЛТ) 3 (рис. 6.29). На вертикально отклоняющиеся пластины ЭЛТ подается сигнал с микрофона 4, который является приемником звуковых колебаний, возбуждаемых динамиком 2. Объяснить, в чем причина тех изменений, которые наблюдаются на экране ЭЛТ при перемещении микрофона 4 относительно динамика 2. Можно ли использовать этот эксперимент для определения скорости звука?
6.30.	Электрическая цепь, состоящая из двух катушек индуктивности и лампочки, подключена к генератору переменного напряжения. Если в одну из катушек вдвинуть железный сердечник, то свечение лампочки усиливается. Если же сердечник вдвинуть во вторую катушку, то свечение лампочки ослабевает. Нарисовать схему электрической цепи. Объяснить явление.
6.31.	Имеются два различных источника напряжения, обеспечивающих одинаковую интенсивность свечения электрической лампочки. При кратковременном подключении к выходным клеммам катушки индуктивности только один источник дает интенсивный дуговой разряд. Объяснить наблюдаемое явление.
113
6.32.	Почему при замыкании ключа 1 лампочка, включенная в цепь, как показано на рис. 6.32, горит слабее, чем при замыкании ключа 2?
6.33.	Диск с одной белой полосой приводится во вращение и освещается лампой пульсирующего света (стробоскоп). В свете лампы видны три полосы, расположенные
Рис. 6.32
под углом 120°. Найти по наблюдаемой картине угловую скорость вращения диска. Частота пульсаций лампы известна.
6.34.	Между двумя плоскими параллельными зеркалами расположен источник света — электрическая лампочка (рис. 6.34). На экране после включения лампочки появляются светлые и темные полосы. Нарисовать схему опыта и объяснить явление.
6.35.	Свет, пройдя стопу плоских тонких стеклянных пластин, погруженных в воду, создает на экране изображение букв: НГУ. В отсутствие воды изображение исчезает. Объяснить эффект.
6.36.	Включают источник света н регистрируют ток фотоэлемента. Затем на пути света ставят толстую плоскопараллельную прозрачную стеклянную пластину. Сначала источник света — обычная лампочка накаливания, потом — источник параллельных лучей. Фототок после внесения пластины в первом случае возрастает, во втором — падает. Объяснить эффект.
6.37.	После прохождения света через собирающую линзу на экране хорошо виден темный круг, окаймленный светлым кольцом. Объяснить наблюдаемое явление.
6.38.	Спираль лампочки накаливания проецируется на экран объективом. Часть линзы объектива перекрыта полоской из непрозрачного материала. При смещении лампочки из плоскости оптимальной фокусировки происходят ха-114
рактерные изменения изображения. Описать и объяснить наблюдаемое явление.
6.39.	Когда между лампочкой и линзой помещена прозрачная с плоскими стенками кювета с водой, на экране видно резкое изображение спирали лампочки. Если же кювету убрать, изображение становится размытым. Объяснить явление.
6.40.	На блестящей консервной банке есть плоская вмятина, от которой на экран отражается свет. В банку наливают охлажденную жидкость. Отраженный сигнал гаснет. Объяснить явление.
6.41.	При падении под небольшим углом на толстую стеклянную пластину узкого пучка света на экране отчетливо видны несколько пятен разной яркости (рис. 6.41). Объяснить наблюдение.
'к *	* -х
Рис. 6.41
6.42*	. Лазерный луч падает по нормали на прозрачную плоскопараллельную пластину, одна поверхность которой закрашена так, что способна рассеивать свет во всех направлениях. На пластине видна следующая картина: светлая точка в центре, темный круг с резко очерченной границей и светлый ореол снаружи круга. Объяснить явление.
6.43.	Сквозь молоко, налитое в вертикальный цилиндрический стеклянный сосуд, пропускают узкий интенсивный пучок света вдоль и поперек оси сосуда и регистрируют интенсивность прошедшего света. При разбавлении молока интенсивность продольного света не изменяется, а интенсивность поперечного — существенно увеличивается. Объяснить явление.
ОТВЕТЫ И РЕШЕНИЯ
Глава I. МЕХАНИКА
§ 1.	Кинематика. Относительность движения
1.1.	Ветер, дующий горизонтально, не может изменить вертикальной составляющей скорости дождя. С учетом этого имеем
«i/y = tg30°, u2/w = tg 45° = 1;
4 u2 = «i/tg 30°= lOctg 30° м/с =17 м/с
1.2.	4
2С1с2
Ь3- ◄ /==^-~й^2 =11 с> с > “
1.4.	При продольном ветре Z1 = 2Zu/(o2— и2). При поперечном ветре Z2 = 2Z/(u2— и2)1'2. Таким образом,
4 ЬИг = [1 — (w/у)2]-1/'2, u < у
1.5.	Если лодка неподвижна, то расстояние между передним и задним фронтами звукового сигнала равно ст0. Из-за движения лодки (за 'время т0 она пройдет путь от0) это расстояние уменьшается на от0. Отразившись от дна, сигнал придет к лодке со скоростью v-j-c и пройдет через приемник звуковых сигналов, находящийся на лодке, за время
ст0— отQ .
Tq т отсюда у — с-------:—
Т0Н“Т
1.6.	Каждый осколок вращавшейся в момент взрыва бомбы за время t удаляется от оси цилиндра в радиальном направлении на расстояние Zf, в другом, перпендикулярном ему,— на расстояние со/?/. Таким образом, искомое расстояние
4 z2 = (o2w+/i)1/2 ►.
1.7.	В процессе восприятия звука от самолета надо различать два момента: 1) начало прихода звука, тогда а-|-<р=л;/2, т. е. 4 а = л/2 — <р 2) последующее восприятие звука, приходящего из других точек траектории, нежели в начальный момент, пршэтом 116
условие а-|~ф = л:/2 уже не выполняется и само рассмотрение становится более сложным. Действительно, когда звук придет из точки В в точку D, самолет уже окажется в точке С (рис. 0.1.7), причем
BC = vt, BD = ct,
АВ = Л tg а =— ft tg <p + ^, BD — ct =h (1 + tg2 a)1/2, tga = —-tg ф+у (1 4~tg2 a)i/2.
Введем число Маха hk = vlc. Тогда, преобразуя последнее выражение, имеем
(1—М2) tg2 а2 tg ф tg а + tg2 ф-М2 = 0;
отсюда
◄ ai,2 = [— tg <Р ± М (tg2 ф-|-1 —М2)1/2] (1 —М2)-1 >.
В дополнение к ответу полезно отметить некоторые особенно
сти восприятия на слух движения сверхзвукового самолета.
Человек с помощью своих органов слуха довольно хорошо определяет направление на точечный источник звука. В тот момент времени, когда он, находясь в точке D, услышит звук, пришедший из точки В со скоростью с, самолет окажется в точке С далеко от
точки В. В точку D придет фронт ударной волны, рожденной сверхзвуковым движением самолета. Будет слышен характерный «хлопок»,
так как почти одновременно к волны из некоторой окрестности В'В" точки В. Далее начнут одновременно приходить уже существенно более слабые звуки из областей траектории самолета В'С и В"С”. Человек в принципе должен был бы отметить, что вдоль реальной траектории самолета движутся от точки В в противоположные
наблюдателю придут сферические
стороны два точечных источ-
ника звука. Эти звуки очень быстро затухают, и в действительности на этот реально существующий эффект никто не обращает внимания.
1.8.	К моменту t, когда пассажир поравняется'с первым вагоном, пройденное им расстояние vt должно быть больше расстояния а/2/2, пройденного поездом, на длину I поезда:
vt —
q<?
2
= 1;
отсюда ii>2 =

2а1 \
V2 /
Решение существует только при	(при меньшей скорости
пассажир вообще не поравняется с первым вагоном поезда). По
117
смыслу условия следует выбрать ответ
Если бы "пассажир, поравнявшись с первым вагоном в момент t±, продолжал бежать дальше с той же скоростью, то он опередил бы поезд, а в момент

2о/
у-
с ним снова поравнялся бы первый вагон набирающего ход поезда (рис. 0.1.8).
1.9.	Пусть т—время опоздания. Тогда длина вагона по условию задачи	“
1вгД±Д2_^=ф(2,+И1+(1).
Это же условие можно записать иначе:
2т/11 = 2т/2 +
отсюда
tl+iiti»— /1 2(П-^)

1.10.	4 t = Vс
1.11.	4 *=//£/2=о,1
1.12.	По горизонтали второй камень движется равномерно. По вертикали оба камня движутся с ускорением g. Таким образом,
(=о cos а-(( —т),
118
ГРР т—искомое время между бросками, vt—^- = vslna-(t — т)—~Т'> ;
где (nsin а)2 > g/. Знак «-|~» в решении квадратного уравнения выбран из условия, что соударение должно произойти лишь после того, как первый камень начнет двигаться вниз (/ > v/'g). С учетом этого имеем
= /-----— =1,2 с ►.
и cos а
1.13.	Из сравнения вертикальных и горизонтальных перемещений камня и утки имеем
О'/2
v sin ос*/ —= v cos a*/ — h ctg а-j-u/j
отсюда
2u
ft = — (у cos а—и) tg2 a, v cos a > и 
При v cos a > 2u траектория камня такая, как на рис. 1.13a, при и < v cos а < 2и —как на рис. 1.136.
1.14. Рассмотрение равноускоренного движения мяча по вертикали приводит к уравнению
,	. gt2 , и sin а Г/a sin a \ 2 2ftli/«
ft = ysina.Z—,	»=------------------—------
2	g L\. g ) gj
Знак «-|~» отброшен, так как он дает время «полета» по траектории, ие имеющей излома. Расстояние, пройденное мячом по горизонтали за все время движения 2t, равно
l=v cos a-2Z = — sin 2a Г1 — f 1	1/21,
g L \ (wsina)2/ J
где ft < (v sin a)2/2g, так как есть удар о потолок. Подстановка числовых данных дает 1 = 5 м £>.
1.15.	Скорость шарика ^максимальна в момент удара о стенку. Из симметрии задачи и с учетом того, что скорость шарика у стенки перпендикулярна касательной плоскости, имеем
= М/С1
119
Скорость шарика при пересечении оси лунки (она же горизонтальная скорость) минимальна:
ymin = amax cos отсюда < Omin = у ctg а = 5 м/с
1.16.	Расстояние по горизонтали, на которое летят осколки, начавшие движение под углом а к горизонту, равно
2
I = sin 2а.
g
Радиус круга
у2
R =— sin 2а'!, g
Осколки, вылетевшие под некоторым углом а из интервала
, _ _ п ,	,1	. Rg
а'<а<——а', где а =— arcsin—=-, 2	2 tP
вылетят за пределы круга радиуса R.
Отношение массы вылетевших за круг радиуса R осколков к общей массе будет равно отношению площади S кольца на по-верхности взрывающегося шара (размеры кольца ограничены углами а' и п/2— а') к площади поверхности шара So. Если г — радиус шара, имеем
Таким образом,
1.17.	За условие минимальности угла бросания а примем касание мячом передней и задней точек дужкн кольца (рис. 0.1.17).
______	Тогда sinP = r//?. Если время по-S'"7ч.	лета мяча равно /, а его скорость
/	/	при броске равна о0, то
t	/ = o0cosa-Z,
..	Н—h = va sin a-1—~~.
R=2r	2
**——•——>4
Рис. 0.1.17	Горизонтальная составляющая
скорости мяча при касании передней точки дужки t>x = t>ocosa, а вертикальная v4 = va sin a—gt. Тогда ctg p = — Vy!vx, Решая получившуюся систему уравнений, 120
находим
, Глн —« , г |	.	,_0 .
a=arctgl2—------Ь(-^2 21г2)1.,2 ] 	◄ а = 45°
Решение задачи несколько упрощается, если рассматривать движение мяча по обратной траектории: из кольца в руки баскетболиста.
1,18.	Если перейти в систему отсчета, в которой конус покоится, то станет ясно, что осколки, летевшие вертикально вверх и вниз, в новой системе будут двигаться вправо под углами 45° к горизонту, т. е. вдоль образующей конуса, и на него не попадут; Все осколки, летевшие правее, а их 4 половина таким образом, попадут на конус.
1.19.	В системе отсчета, движущейся со скоростью осколки равной массы разлетаются в произвольных направлениях с равными по модулю и противоположно направленными скоростями v', Максимальную в неподвижной системе отсчета скорость имеют осколки, летящие в направлении движения пучка;
u = v-}-v’, u^4~t'2= v'2i отсюда
<	=	—u2)1/2 = [u (и —2ц)]1/2, u^s2v
1.20.	В системе отсчета, связанной с мухами, ядро подлетает к рою под углом а, причем
v COS ОС   / О I	.о >
(У2-|- И3)1/3
и поэтому проходит в рое путь, равный
Окончательно
.. dnR2 , „Г. , ! U \ 21 1/2
N = п----= nndR2 1 + —
cos а	I 1 \ v ) J
1.21.	После удара о неподвижную стенку скорость шарика меняет лишь направление. Чтобы вычислить скорость шарика после одного удара об удаляющуюся со скоростью и стенку, надо перейти в систему отсчета, в которой стенка покоится. В этой системе отсчета скорость шарика до удара равна — и. После упругого удара проекция скорости изменит знак: станет равной —(о0 — и). Если затем вернуться в первоначальную (лабораторную) систему отсчета, где стенка движется со скоростью и, то здесь скорость шарика о=о0—2и. Слагаемое —2и будет добавляться после каждого удара о движущуюся стенку. Следовательно, v — vo — 2nti, где ц — число ударов о движущуюся стенку.
121
Таким образом, в случае а), когда происходит 20 ударов, 4 р = 33 см/с а в случае б), когда происходит 19 ударов, v= 17 см/с
1.22.	Скорость шарика при ударе о пол v0 = \r2gh (рис. 0.1.22а).
Ту
а
Рис. 0.1.22
Пусть искомая скорость шарика равна v {рис. 0.1.226). Перейдем в систему центра масс шариков, где они движутся навстречу друг другу со скоростями v/2. Роль непроницаемого каменного пола при их ударе сыграет плоскость симметрии АВ (рис. 0.1.22в). Таким образом, ясно: чтобы шарик разбился, нужно выполнение условия и/2 = v0 = У 2gh; отсюда v = 2~y 2gh
1.23.	Шарик падает равноускоренно, при ударе о плиту меняя направление и модуль скорости. Проще всего можно провести рассмотрение движения в системе отсчета, связанной с плитой. В этой
системе шарик имеет начальную скорость »0, направленную вверх,
и периодически ударяется о неподвижную плиту. В момент l(l = v0/g его скорость обращается в нуль, дальше движение периодическое. Время подъема и падения (полупериод этого движения) одинаково. Таким образом,
График зависимости скорости шарика от времени в системе отсчета, связанной с плитой, изображен на рис. 0.1.23. Зависи
мость скорости от времени в интервале между началом движения и первым ударом шарика о плиту v ——(далее см. график).
Время первого, второго, ..., n-го ударов:
ti — ~ Go + V fo-j-Zg/i) , /2 = — (ц0-}_ з}/" Oo-(-2g/i) , ,,,, S	ё
tn — ~ [ио4~(2п = 1) У
122
1.24.	Относительно стакана шарик движется вдоль его стенок с ускорением geos а, Таким образом, время движения стакана до n-го удара шарика
#„=(2п-1) 1/—— •
' У g cos а
Стакан совершает вдоль наклонной плоскости равноускоренное движение без начальной скорости с ускорением a = gsina:
S/i - g sin2a 1 = (2n — i)2 /г tg q; s5 = 0,81 m
§ 2. Динамика
1.25.	Введем обозначения: L — искомый путь, Z—путь, пройденный после окончания действия силы F, v — скорость тела после разгона, а — ускорение при разгоне. Из кинематики следует, что
,	, , at2
L = l-\—, v = at.
По второму закону Ньютона имеем
ma = F — ktng.
Кинетическая энергия расходуется на работу против силы трения:
tnift	F / F \
krngl^^-- отсюда =
1.26.	tzfflax = g cos a(F — tg а)
1.27.	вщах — g cos a (k— tga) = l,l м/с2 (см. ответ к задаче 1.26).
1.28.	По второму закону Ньютона имеем
та= N sin a—kN cos а, tng = kN sin cos а, где TV —сила нормального давления бруска на цилиндр; отсюда
<4 a = arctg-^i^-
1—fetz/g г
1.29.	f — sin 2«
1.30.	, Сила трения kmg cos а направлена против результирующей силы VFmin+ {mg sin a)2 . Приравнивая силы, получаем
◄ Fmin = mg У&2 cos2a—sin2а при fc^tga, fmin=0 при k < tg a
123
1.31.	Т — (k2 — ki)	- g cos а при k2 > fei > tga
1.32.	По второму закону Ньютона имеем
ma = F — T^ — kmg, ... , ma=^Tn_1— Tn — kmg,
F ma — Tn— kmg', отсюда a = ———kg.
Таким образом, Т,= F (1 — ijn), i=l, ... , n — 1 ►,
1.38.
F
a = — (cos a — k sin a) — kg, если F cos a > k (mg-}-F sin a);
a — 0, если F cos a< k (mg-rF sin a) ►.
1.34.
F
a = — (cos a4-k sin a) — kg,
если F sin a < mg, F cos a > k (mg — F sin a);
a = 0, если F sin a < mg, F cos a k (mg — F sin a)j F	F
«гор cos a, aBepT	sin a—g, если F sin a > mg
1.35.	По второму закону Ньютона имеем
tnia = N — m^g, m2 (а’ — a) = m2g — Т, m^a' — Т—kN, где N — сила нормального давления, а' — ускорение грузов относительно стола. Тогда
◄ Г =	(a+g) при kmi < '”а
/712 T
Если а' = 0, то
— т2а = т^—Т, T = f <. km1(a-[-g).
Таким образом, T=m2(a-(-g) при kmt > m2
1.36.	По второму закону Ньютона имеем
ma = mg sin 0 — 2kN,
где сила нормального давления
N = mg
cos fl
2 sin a '
Таким образом, . f . Q k cos fl \ a k a — g Sin fl---------;   при tg fl > —; ,
s\	sin a J H r sin a
k
« = 0nPHtgfl<—.►
124
1.37.	Пусть угол 'между вертикалью и нитью, прикрепленной
к грузу массы т2, равен а, а ускорение тельно стола равно а'. [Тогда ускорение тельно земли равно а—а’, горизонтальная составляющая ускорения груза массы т2 относительно земли равна а — a'sin а, а вертикальная — a'cos а. Запишем второй закон Ньютона:
тг (а-—а') = — F-^-ktnig, т2а’ cos а = m2g — Т cos а, т2 (а—a’ sin а) = Т sin а.
Два последних уравнения при исключении угла дают уравнение
т2а' = — Т + т2/ a2 + g2.
При наличии проскальзывания (а' > 0)
груза массы mi относи-груза массы mi относи-
1 'mg
Рис. 0.1.38
* T=i^V^+k8-a),
Без проскальзывания (а' = 0)
T = m2/a2+g2, r«2 + g3<^-*g-4^a>.
/721	/722
1.38.	Пусть модули сил натяжения пружин AD и CD равны | F | и | Fi | соответственно, причем | Fi | = |	| bs Fi (рис. 0.1.38),
а модуль силы натяжения пружины BD равен |F2|^F2. Условие равновесия по вертикали дает
2F1-K 3/3 4-F2 = mg,
По закону Гука имеем
F1 х — /р) ____________________1о_______
F2~ v.(l2 — l0) ~ (/3 — 1) /0 ’
F2 = (/ 3-l)Fi.
Так как после разрыва пружины AD сила ее натяжения обратилась в нуль, то сила, действующая на грузик, стала равной —Fi. Из (1) находим
V’’3F1+(yr3-l)f:'i = /ng, т. е. Fj = mg/(2 У~3-1);
отсюда
^a=-Film, |a| = g/(2K 3-1) ►.
125
1.39.	По второму закону Ньютона в первых двух случаях имеем
т1 — та
m-ich. = ntig — XiXi,	gi
m2 — m0
m2a2 = m2g — x2x2, a2 = —-—~g.
mo~r
В третьем случае, когда к пружине жесткости щ последовательно подвешены два груза,
тоа = —mog-\-T, m1a=^m1g — T1-}-T2, m2a = m2g — T2,
где Т~Тг — сила натяжения нити, прикрепленной к грузу массы т0, Г1 и Т2 — силы упругости пружин жесткости xt и х2; отсюда тгЧ-т-з — т0 а = а—;.
6 «i --т2---т0
Так как 7\ = ххХр Т2 = и2х^, где х± и х'2 —удлинение пружин при последовательном соединении грузов, то общее удлинение пружин x = x1'-|-x3' = 7'1/xi4-7'2/x2. Окончательно
х =
(fflo + wx) (ffii + m8) Xi + (^o + '4>) ffliXz mi (m04-mx + m2)

1.40.	Пусть не выскользнула часть каната длины х, тогда выскользнувшая часть каната имеет длину I— х. Обозначим массу каната буквой tn. Запишем второй закон Ньютона для всего каната и для его висящей части:
ma = mg (l — x/l), m (1 —x/l) a=m(l — x/l) g~T;
отсюда (
T = mg(x/l) (1—x//).
Исследуем это выражение на максимум: г2 — z-[-a = 0, где г=х//, a — T/mg. Максимум будет при г =1/2, тогда Traax = mg/4. По условию задачи, чтобы канат не порвался, должно быть Т< Го = = mgl0/l. Поэтому, полагая Tmax = mg/i mgl^/l, получаем условие целостности каната
mglKmgldl,
^шах — 4/о
1.41.	Импульсы, передаваемые призме ударами шариков, перпендикулярны поверхности, а их горизонтальные составляющие на одной и той же высоте по обе стороны равны. Однако число шариков, упавших одновременно слева и справа, пропорционально площади проекции грани на горизонтальную поверхность. Следовательно, сила слева, где угол меньше, будет больше, чем справа, и призма поедет в сторону большего угла, т. е. в данном случае направо
126
1.42.	1) Скорость шарика при падении с высоты h равна п = =У2gh, а средняя сила удара п шариков о поршень
_	/ Др \ о
fcp= -гт- = 2mvnt.
\ /Ср
Таким образом,
Р = Ро + (Mg+2 V^gh mnt)/S |>.
2) Каждый шарик при упругом ударе о поршень, падая с высоты h, передает ему импульс т• 2v = т• 2У2gh один раз за все время t движения от поршня и обратно, причем время t равно удвоенному времени падения шарика: t = 2У 2h/g. Таким образом, средняя за время t сила действия одного шарика
„ Др m-2v	т-2У 2gh
F1^^~i-=^y^=mg-
Для средней силы действия N шариков получаем
Fcv= NF1CV — Nmg.
Результат не зависит от высоты h и, если бы это было известно заранее, получился бы сразу при h — О. Тогда шарики оказывали бы давление на поршень только в результате действия своего веса. Окончательный ответ:
•4 P = Po + g(Nm + M)/S >.
§ 3. Вращательное движение. Колебания
1.43.	Частота возникновения той ситуации, когда Солнце, Земля и Марс оказываются на одной прямой, определяется разностью частот обращения этих планет вокруг Солнца:
V__J______1 .. Гм—?з
Т3	Vu 
В земных годах период наступления таких событий
.___1_______
vT3-TM_T3-
Число t не целое. Чтобы узнать, через сколько лет повторится великое противостояние, нужно выяснить, сколько раз в t содержится его отличие от целого числа t — [/]. Это и дает ответ:
◄ W —М) « 687/45 я 16 лет
127
В действительности великие противостояния Марса наблюдаются каждые 15 или 17 лет.
1.44.	По второму закону Ньютона имеем
ты2г = N + mg sin а,
где Л/—-сила нормального давления. Чтобы не было проскальзывания, должно выполняться условие
mg cos а k (т<о2г — mg sin а),
откуда
со2 (cos а-J-sin а) при k = l.
Таким образом, <»min = (V Sg/r)1^2
1.45.	«й оз= (у- ———„V''2 ПРИ а<Р (рис. \ I sin а 4-sin р ]
/ 0 tp" В \ 1/2
го = ( 4—:------—д-	при a > Р (рис. 0.1.455)	.
\. I sin а — sin р J 1	'
0.1.45a),
Рис. 0.1.45
Рис. 0.1.46
1.46.	Сила тяжести mg н сила нормального давления N соз. дают результирующую горизонтальную силу oigtga, являющуюся центростремительной (рис. 0.1.46):
о»3/г = oigtga, v= Krgtg a
1.47.	Угол вылета молота равен 45°. Примем, что п > 1. Второй закон Ньютона дает
v3// = ng; отсюда Z.max = v2/g = nl ,
1.48.	По второму закону Ньютона имеем
/ли2 (J4-x)=2xx; отсюда х =тЮ ? „ |>. ' 1 '	2х —око3 г
При 2х < тч>2 равновесие неустойчиво, при 2х > /ксо2 — устойчиво.
1.49.	7'i = /n1w2/i, Та = (011/14-012^2) и2 £►.
1.50.	Векторная сумма силы натяжения троса Т и силы сопротивления воздуха F направлена по радиусу г2 (рис, 0.1.50) и равна 128
по модулю mco2r2, сила Г перпендикулярна радиусу r2. С учетом этого имеем
ггк»2г^ — Т cos а.
Из Л АОВ по теореме косинусов имеем г3 = г24-/3— 7 г Л cos а. Таким
образом',
2/«п/д>2	.
*>•
1.51.	Можно считать, что тела участвуют в двух движениях: во вращении вокруг центра масс системы и в поступательном дви
жении вместе с центром масс. Положение центра масс относительно.
тела массы тг находим пз уравнения
xi (mi-4-т2) = отсюда л,=/-----—2—,
mi+«2
Второе тело находится на расстоянии х2 — 1— xt от центра масс, Так как суммарная скорость тела массы т2 равна v, а скорость тела массы т1 равна нулю, то имеем систему уравнений
г?ц.м4-®*2 = ц, Цц,м — сох1 = О, ximi = x1ml,
где и— угловая скорость вращения, а г>ц. и--скорость центра масс;
отсюда
т -г
'’ц, м —	i •
Сила натяжения 7’=^-/н/о'у K/Xi. Окончательно
. Т _	v
mi + т2 I
Можно было бы решить задачу, перейдя в систему центра масс, инерциальную в отсутствие внешних горизонтальных сил. В этой системе сила натяжения Т обусловливает вращательное движение каждого из тел вокруг центра масс. Скорость каждого тела легко вычисляется. Если известны скорость, масса и радиус вращения, можно найти силу Т.
1.52.	Из второго закона Ньютона получаем
-2^- = 2Tcos 60°,	=27~ cos 30° + х (К 3-/"2)^
2	2
Б Г. В. Меледии	.	129
(здесь Т— сила натяжения нити); отсюда
1.53.	Так как скорость v бусинки постоянна, сила трения kN уравновешивает «скатывающую» силу mg sin а, Из второго закона Ньютона получаем
/ mv2 , V , ,	,,	( mg sin а X2
cos3 а I -j-(mg cos а)2 = № = /—--- I ;
отсюда, сокращая на tn, имеем
K-Rg (7 sin a \2	, 1 1/4 t l
“““cos “Г npntga>A>.
CvO Сл< В \ rV /	I
1.54.	Из условия равновесия нижнего шара имеем
NX=T cos a.
Верхний шар движется по окружности радиуса I, поэтому то2 . ,г ——= mgsina— Г,
Из закона сохранения энергии следует . ту2 .	, .
mgl ——g—|-mgZ Sin a!
отсюда
Nx = mg cos a-(3 sin a — 2), пока sin a > 2/3,
Л7х = 0 при sin a sg 2/3 —нижний шар отходит от вертикальной стенки .
1.55.	Угловая скорость вращения звезд
«!= К2б/И/Г3;
угловая скорость вращения планеты
w2= К 20/И/х3.
Приравнивая их, получаем х = г ►  Условиям задачи удовлетворяет также еще один ответ: х = г/2 |>, соответствующий случаю, когда планета находится в точке, вокруг которой обращаются обе звезды.
1.56.	Звезды движутся по окружностям радиуса /д и га. Эти окружности имеют общий центр —центр масс. Расстояние между звездами равно Г1 + г2- Таким образом,
mim2	vi, 2 Т
~ (и + гг)2 ’ Г1,2~ со — ~2nVi'2?
130
•тсюда
Т
mi=Sr to +t'2)2’ m*=йг to+*2)’
1.57.	Разложим силу тяги F двигателя на составляющую Гц, лежащую в плоскости окружности, описываемой при движении спутником, и составляющую F перпендикулярную этой плоскости. Сила гравитационного притяжения Земли, действующая на спутник на расстоянии r — h-\-r^,
Frp = tngr23/r2.
Второй закон Ньютона для спутника в проекциях на выбранные направления дает
F । —fro sin а =0, Лгп cos a — F., ——~—.
-L 1	р II г cos а
Из условия задачи период обращения
„ 2лг
7 =— cos а = 1 сут,
поэтому
„ г-	fnv2
г । (= ггр cos а —
г3 4л2г \ m I g —х------cos а = 0.
г cos а г2 Т2 /
Таким образом
F± = mg
-—ГЗ г.-т- sin а я 0,02mg я 200 Н (Гз+Л)
1.58.	^гпах— 2t1lg/'&
1.59.	'В выражение для периода колебаний вместо ускорения свободного падения g войдет лишь его составляющая g' = gcosa (так как составляющая силы тяжести, действующая вдоль стержня, компенсируется силой упругости), а в качестве длины маятника войдет некоторая «эффективная» длина V = I sin (3:
Т = 2лК Г 1g' — 2лУ I sin P/(g cos a)
1.60.	Маятник начинает колебаться относительно точки подвеса с периодом Т—2лУ l/g и начальной скоростью ®. Начальная скорость относительно земли равна нулю. Остановить подвес так, чтобы мгновенно остановился и маятник, можно в тот момент, когда скорость маятника относительно земли равна нулю (маятник при этом проходит положение равновесия). Такая ситуация 5*	131
будет иметь место через промежуток времени nT = x/v, где п — натуральное число; отсюда
Р=^==2^'/'Т (ПРИ
§ 4. Законы сохранения энергии и импульса. Работа. Мощность
1.61.	Используя закон сохранения энергии и второй закон Ньютона, получаем
mv2 , mv- m
~2~:=mSl> =:zI - lnS-
Таким образом, .	, T—3mg	о
◄	= z т—mg ПрП T^3mg ►'
При T < 3mg нить оборвется раньше, чем достигнет гвоздя.
1.62.	Скорость шарика при подлете к ракетке v=y2gh, а при отлете от нее v' = У 2ghn = vY~ п. Так как удар упругий, то в системе отсчета, движущейся со скоростью ракетки и, шарик отскочит вверх со скоростью v-^-u, а относительно земли — со скоростью v' = (v-]-u)-[-u = v-]-2u=y nv; отсюда	и=У gh/2 (У п—1)
1.63.	Угол касательной к винтовой линии с горизонтом равен а, причем
Ло
Sm а = —т;------г-;-- .
(/1о + 4л2г2)1/г
Из закона сохранения энергии mv2/2~ mgh. Сила, с которой бу' синка действует на спираль,
Р =V ^центр + ^норм,
где
2
г	tflVrop
г центр — —~
mv2
r[l-H/io/2ra-)3] ’
_	2лл
W+4„,
2 , /2liB V»
л f- те
*	1 + (/10/2лг)2
1.64.	В момент удара скорости грузов равны. Одни из грузов опустился на высоту h, а другой поднялся на такую же высоту, 132
Закон сохранения энергии дает
miV2 । m2v2
1 1 2~
migh — m2gh.
В теплоту перейдет энергия W — mtV-/2. Таким образом, количество теплоты
4 Q = W = migh^^
1.65.	Так как брусок свободно плавает, масса вытесненной им воды равна массе бруска т. После перевертывания бруска центр масс вытесненной воды поднялся на высоту (/—d)/4. Центр масс бруска остался на уровне поверхности воды, поскольку брусок снова находится в равновесии. По закону сохранения энергии количество выделившейся теплоты равно убыли потенциальной энергии воды:
4 Q=mg(l—d)/4 « 4,7.IO"3Дж
1.66.	Введем начальную и конечную (hi и й3) высоту уровня жидкости относительно нижней поверхности плавающей пробки. Пробка, упав, свою потенциальную энергию mg(H-(-hi) израсходует на подъем воды (mog (h2 — fti)/2) и на выделение некоторого количества теплоты Q. Следовательно,
Q = mg (Н-f-йх) — m„g (h2 — /ц)/2,
где m = pnr2h, то = рол (R2 —г2) h2.
Условие плавания пробки имеет вид
ph = p0h2.
Условие вытеснения пробкой части воды в сосуде
htiiR2 = й2л (R2— г2);
отсюда
L -Ро \ а / J
1.67.	4 Л = Ро§/гз(Я-/1/2)
1.68.	Потенциальная энергия взаимодействия двух точечных зарядов, находящихся на расстоянии г друг от друга, описывается формулой
4л80 г
где отличие от выражения для кулоновской силы
р_____1
е 4ле0 г2
133
только в показателе степени г. Используя аналогию между законом Кулона и законом всемирного тяготения /7= G/n1m2/r3, запишем выражение для потенциальной энергии двух притягивающихся (поэтому появляется знак «—») точечных масс:
F = -G^.
Г
Вне шара поле сил тяжести таково, как если бы вся масса была сосредоточена в центре шара. Поэтому для тела, находящегося на поверхности шара или очень близко к ней, в качестве г надо брать радиус шара. Для земного шара у его поверхности имеем
r0 = -G^3. ,
гз
В то же время сила тяжести на поверхности Земли
mg=GmM.3/r^, отсюда 0Л1з = gr^_
Подставляя полученное для ОМз выражение в формулу для Wo, получаем
№0 = — mgr3.
Если речь идет о Луне, то вместо гз надо взять радиус Луны гл и ускорение свободного падения на Луне gn = g/6. Таким образом, для Луны находим
Wo = — mgr^fi.
В момент начала движения корабля к Луне его кинетическая энергия по условиям задачи равна нулю, в нуль обращается и потенциальная энергия из-за большого расстояния г до центра Луны. В момент включения двигателя кинетическая энергия корабля равна mv2/2, а потенциальная равна —mgr ^5, Закон сохранения энергии дает
. mo2 mgrn	grn
0 = ~2----— , откуда v2 = -j-t
Считая a —5g (влиянием добавки к ускорению, равной —g/б, можно пренебречь), из кинематических соотношений для равноускоренного движения с начальной скоростью v получаем v2 = 2ah = 10g/i, Подставляя выражение для о2, имеем
grnft= lOgft, откуда й = гл/30 к 60 км >.
134
1.69.	Пусть масса тела равна т. Тогда из закона сохранения энергии имеем (см. решение задачи 1.68)
т. е.
g#3 — 2g/?3 = —§7?з =—; отсюда й = /?з = 6,4-!03 км
1.70.	Запишем закон сохранения энергии, сразу учигывз.: скорость тела относительно Земли и его потенциальную энергию, в виде (см. решение задачи 1.69)
/И(го + г3)2	mo2 m(va — v3y
----2--------mSR3 = -j- , '2 3 — mgR3 = 0,
где v3 = l/Т; отсюда
v = 2 [°з (v3 + W]1/2 = 2 [ 1	r 2gi^) ]1/2.
Так как /?з = //2л, то у = 4,6км/с
1.71.	Минимальная скорость прохождения петли достигается, когда центр масс части каната, имеющей массу т' —	ока-
зывается подняты*.! на высоту г:
шу?п!н/2 = m'gr; отсюда	ош-1п =2 г К ng// >.
1.72.	Скорость жидкостей достигнет максимума, когда снова возникнет положение равновесия: более плотная жидкость окажется в низшей половине квадратной трубки, а менее плотная—в верхней. Центр тяжести жидкости, занимающей половину (верхнюю или нижнюю) квадрата, находится на расстоянии Z/8 от ближайшей горизонтальной стороны. Смещение центра тяжести каждой жидкости после перетекания Л/ = 1 — 2//8 = 3//4. Тогда изменение потенциальной энергии
ЛЕпог = ('«1—«з) g-3Z/4 = (pi —р2) Slg-3l/2, а максимальная кинетическая энергия
п (т14~ тз) итах ,	,	\ сг г
Е кин = '----§-----= (Р1 + Рг) Sfomax'
Из закона сохранения энергии Л£пот = /:кии получаем
◄ ^тзх=
1.73.	Из закона сохранения энергии следует, что
z (I — /0)2_2 то2
~2	“2~‘
135
Скорость бусинок вдоль осп х равна vx = о cos а. Таким образом,
гх= (Z—/0) |Лх/2т cos а
1.74.	Используя закон Гука и закон сохранения энергии, получаем
XjX — Х2 V х), 2	2	1 2	2	’
где х— координата нового положения равновесия, в котором скорость максимальна. Таким образом,
\т (Xi + x2)]
►.
1.75.	Начальное равновесие для удлинения х0 дает ххс= 44g (здесь х —местность пружины). По закону сохранения энергии имеем
~2” Н" Ч~ т) S (•’max	*o) — ~ g > •’max 7= X(j.
Подставляя л-0 = Mg/и, получаем хтах = (44-|-2m) g/x. Из второго закона Ньютона получаем
та = mg — N, (44 -ф т) а = (М -ф т) g — хх;
отсюда
тих
т. е. 4Zmax =
тхХгпах
М + т ~'
mg
М + 2т М + т
1.76.	В момент отрыва скорости шарика и конца пружины одинаковы. Отрыв происходит при х=0 (х—смещение конца пружины от положения равновесия). Кинетическая энергия массивной пружины пропорциональна квадрату скорости: £'пр = ао2, где а — коэффициент, пропорциональный массе пружины. Из закона сохранения энергии имеем
kx* 2
kxo
2
mv2
9	1	2
=	и2--уЦ-a-— D2,
йх2
2
Зтп?.
____i_L
2
•|-at>2,
an2.
Из первых двух уравнений находим а:
m/2-|-a = 2m/34-2a/3, откуда a = m/2.
Подставляя а в последнее уравнение, находим vx:
◄ их
136
1.77.	Пусть каждый шарик прошел до остановки путь I. Тогда закон сохранения энергии дает
mvl
2 -^- = 2kmgl.
Число оборотов п = 1/п!0, 4 n = vl/2nkgl0
1.78.	Закон сохранения энергии дает
mgH = s^fat kl'ng C0S ai + sina2 k2ms C0S “+mgh'
Окончательно
. ,	1 — ki ctg ai .
4 h = H -----......- b.
1 <^2 cig a2
1.79.	Пусть x, — максимальное смещение правого тела влево, а х2— его последующее смешение вправо от своего начального положения. Тогда из закона сохранения энергии получаем
mv" wcf	KXi wcl ,
-2-= — + ktngXi, — = —(Xi + %2).
Требование сдвига левого тела дает условие
хх2 = kmg.
Решая систему трех уравнений, получаем xi = 3x2 и 4 v — = kg У 15т/к
1.80.	Если брусок массы т2 остается неподвижным при смещении х бруска массы mit то сила F совершает работу по растяжению пружины и против сил трения (при условии, что в конечный момент скорость бруска массы m.i обращается в нуль):
„ их2 . .	г их . ,
Fx = ~2—\-kmiSx< т- е- f = y +
где и — жесткость пружины.
Уравнение движения второго бруска:
m2a2 = хх— kmzg.
Брусок массы и2 сдвинется при условии а2 > 0, т. е. при условии хх > ktrtig. Минимальное значение Emin получим, если положим wc = kmzg. Таким образом,
4 /?min = ^(«i + "’2/2)
1.81.	Масса воды, выходящей в единицу времени из брандспойта, равна pf>vS. Мощность, передаваемая насосом выбрасываемой
137
воде, удовлетворяет соотношению г)Р = р0о$-н2/2.
Из кинематических соображений имеем 1= (oa/g) sin 2а, Окончательно
sin 2а= lg (p0S/2r)P)a/8
1.82.	Вначале колеса проскальзывают и часть мощности двигателя затрачивается на работу против сил трения, Сила тяги в
tup
Рис. 0.1.82
этот период равна силе трения f=kmg, т. е. ускорс-'  автомобиля a —kg и v=at—gkt Как только проскальзывание прекратится, вся мощность двигателя пойдет на разгон автомобиля. Критическая скорость
Р __ Р С'к₽~ f ~~ kmg'
Время достижения критической скорости
t - Vkv- Р к₽ a mg2k2'
Дальнейшая скорость определяется из закона сохранения энергии
где время t отсчитывается от начала движения. Это уравнение удовлетворяется начиная с момента i-t^ (рис. 0.1.82):
<2Р / Р \	Р
0= 1/   I — 5 ДЛЯ t > /кв= угк г т \ 2mg2k2)	р т g2k2
1.83.	Пусть v'— скорость пули после пролета сквозь первый груз, «1,—скорость первого груза после пролета сквозь него пули, 138
и2 — скорость второго груза. Скорость пули после попадания ее во второй груз ©" = и2. Закон сохранения импульса дает
—	mv' = 2mut.
Из закона сохранения энергии имеем
mv2 mv'2 ,ти2 . _ mv'2 _2mul n
~2	2 ‘ 2 ‘	2	2
Таким образом, Qi = mv’(v— v'), где v' — VQ2/m, т. e.
Qt = 2v УтО2 — 4Q2 при Q3 < mv2/4
1.84.	Пусть Zo—расстояние от пушки до лагеря, а ©0 —начальная скорость ядра массы т. Введем массу барона М и скорость © ядра вместе с бароном. Тогда, используя закон сохранения импульса и выражения для расстояний Zo и Z, получающихся для тела, брошенного под углом а к горизонту в поле силы тяжести, получаем
v2	v2
mv0 = (M-^-m) v, 10 = —sin 2а, Z = —- sin 2щ
g	g
отсюда
- Zo — I / m \ 2 _ 1 + 2m/A4 _35
~~(1+т/Л1)2~~36	*
1.85.	x =	.
1.86.	Различие времен падения Z3 и Z3 обусловлено различием начальных вертикальных скоростей. Вертикальная составляющая импульса системы равна пулю, массы частей равны. Отсюда с учетом закона сохранения импульса вертикальныеЪачальные скорости частей равны п противоположны, причем скорость первой части у01=?у0 направлена вниз, скорость второй части i'03s — г?0 направлена вверх. Для первой части Zi = l'o/i + gZj/2; отсюда га =.- (2h — Для второй части /г== — vat-2 +gZ2/2, подставляя у0, получаем
+	Z3-2/i = 0;
отсюда однозначно определяется Z3 = 2/:/gZ1
1.87.	Закон сохранения импульса дает
(mi + nh) и = туи, и (1 + щ1/7?г3) В>-
1.88.	На основании второго закона Ньютона, записанного в виде F = hp/\t, имеем
Др = f ср • 2т = (F0/2) -2т = Fot.
139
Из условия линейности зависимости силы от времени следует, что Fcp = f0/2 (аналогично, например, при выводе выражения для энергии сжатой пружины используется линейность закона Гука).
Изменения импульса каждого из шариков в проекциях на начальное направление полета определяются следующими выражениями:
Щ1О1 = Г от, m2v2 — m2v = — Fox-, отсюда
v1 = Fox/mi, v2 = v — F0x/m2
Зная конечные и начальные скорости и массы, находим энергию, перешедшую в теплоту:
м3у2 m2vl m4v2i .	F„t:2
-----2-----Q = vFaX-—^~^
1.89.	По закону' сохранения импульса имеем
[т— (Дтх-г-Л^ + М] « = (Дт! —Дт2) v' =
= (m/2) (cos а2 — cos a,) v cos.a,
где ааа1«а2, Д/ni и Дт2—массы осколков, вылетевших в соответствующие отверстия в сфере, и — искомая скорость сферы и прилипших к ней осколков после взрыва. Доля вылетевших осколков от всей массы т пропорциональна площади отверстия Si, 2 = 2«.'/ii,2=2лг2(1—cos aI>2):
Д;П1+&т2 — (т/2) (2 —cos ai — cos a2)xm-m cos a.
Окончательно
. m cos a (cos a, — cos a2)
< U = V----2(щ^^ + ЛТ)------	•
1.90.	Скорость сближения шарика с короткой стороны рамки останется после предыдущего соударения с рамкой той же самой по модулю. Это легко понять, рассмотрев соударение, например, в системе центра масс, где после упругого удара модули импульсов тел не изменяются и, следовательно, не меняются модули скоростей, а направления переходят в противоположные. При рассмотрении движения в лабораторной системе с привлечением законов сохранения энергии и импульса после сравнительно длинных выкладок, получается, естественно, тот же результат. Таким образом, t = 2l/v .
1.91.	В системе отсчета, связанной с лентой, начальная скорость направлена под углом 45° к первоначальному направлению и | ®1 | = v V 2 (р ис. 0.1.91a). Направление скорости под действием трения в этой системе отсчета не меняется, так что конеч-140
иая скорость с2 здесь направлена под углом 90° к конечной скорости •о' в лабораторной системе отсчета. Из рис. 0.1.916 видно также, что | с2 | = | v' | = v/У 2. Подчеркнем также, что оба рисунка указывают переход из лабораторной системы отсчета в систему
Рис. 0.1.91
отсчета, связанную с лентой. Шайба по ленте проходит путь 1У 2. Закон сохранения энергии дает
mvl mvl т (п „ у2 \	3	„ ,	., ст-
отсюда
1.92.	С учетом закона сохранения импульса при ударе
тли — m2v = (mi + т2) и
и закона сохранения энергии до и после удара, из которого следуют соотношения v=y 2gH и u=V2g/i, имеем
1.93.	Законы сохранения энергии и импульса дают
Mu2/2 = 2mv2/2, Mu = 2mvV 3/2, М = Зт/2 ,
1.94.	Угол разлета покоящихся шаров равен 60° (удар упругий, трения нет, шары взаимодействуют вдоль прямой, соединяющей их центры). Обозначим проекцию скорости каждого из этих шаров на направление вектора и через v;x. Из законов сохранения энергии и импульса имеем
mu2 2m^i . mu'2	„	,	, У 3
“у-= —+—. mu — 2mv,x-]rmu , v.x = —u,
141
где i — 1, 2 — номер шара; отсюда
.	2 К 3	,	1
«3 = и, г/' = — -§-«>•
1.95.	Продольная составляющая скорости иц—иУ 3/2, направленная вдоль оси реакции ОС (рис. 0.1.95), сохраняется.
Перпендикулярная составляющая скорости v^ — v/2 и направлена вдоль АВ.
Используя законы сохранения энергии и импульса (сохраняющиеся продольные составляющие из уравнений уходят), имеем
3m (о/2)2 т (и/2)3 2'2	~ 2	‘	2	’
3m (и/2) — т (о/2) = ЗтиА + тив
отсюда «лх = 0, uB1_ = v. Таким образом, uA = vy 3/2 и направлена по ОС (рис. О.1.95),пв = К("К 3/2)2 4-d2 и направлена по 0D; отсюда
tga = OE/OC = 2/‘3/3, < a = arctg (2 У 3/3) « 49° > ,
1.96.	Проведем ось X через центры шаров, а ось У—через точку их соприкосновения по касательной (рис. 0.1.96). Из-за гладкости шаров {/-составляющие импульсов и скоростей шаров ие изменяются после удара, т. е.
vyA = чвуА = v sin 30° = v/2.
Из-за упругости удара и равенства масс шаров с учетом сохранения {/-составляющих их импульсов шары после удара обменяются .«-составляющими импульсов и скоростей, т. е.
vxA = 40xb = 2v cos 30° = о'К 3;
отсюда искомый угол у = л/3-(-₽, где
tg & = oxA/VyA = 2V 3, т. е. 4 y = n/34-arctg (2 У1) х 134° > .
142
1.97.	Запишем для первого соударения законы сохранения: о „ , . 2/ада 2mu'a . 2ти = 2ти' -\-mvi,	---1—2~<
отсюда О1 = 4й/3, причем существенно лишь отношение масс соударяющихся шаров, а не сами массы. Это же отношение справедливо и для последующих ударов. Значит, и для последующих ударов vn = =4п„_1/3. Таким образом, можно выразить скорость последнего шара через и:
◄ oJ9M = (4/3)ls’aa ► .
1.98.	Обозначим через проекции скоростей шаров на направление, перпендикулярное направлению движения доски, и через v — скорость доски сразу после удара. Из и импульса имеем
Ми“  (2ш-|-ЛЦ о2 , 2ту1
законов сохранения энергии
2—, Ми = (2т-\-М) v;
2
2
отсюда
и
v, =-^======== , v . —  м/с « 0,7 м/с , /l+2m/.M	К 2
1.99.	Центр масс стержня займет низшее положение, когда стержень встанет вертикально. При этом сила натяжения нити из-за малости угла отклонения останется вертикальной. Поэтому горизонтальная составляющая импульса, равная нулю, не меняется, откуда mvg — mvc. Закон сохранения энергии дает
/ПРд m'l'o	г—-~
—2~-|—ij— = mgl-\-3mgl-, отсюда vg — vc = 2 у gl.
Таким образом, стержень в этот момент вращается вокруг центра О отрезка ВС. Расстояние О А —20 В, поэтому vA = 2v3^ifgl .
1.100.	По закону сохранения энергии скорость грузика А в момент прилипания грузика В из уравнения 2my2/2 = (2/3) mgl равна i'o = V (2/3) gl. Та к как
mo2 2 , т 2
—=з-^-=Т^<^
то нить не натянута. На рис. 0.1.100 координаты ха и у0 грузика А в момент времени t определяются соотношениями х0 = vot, у0 —
143
= 1—gt2l2. Момент появления натяжения задается условием Z; отсюда получаем квадратное уравнение:
о’о/2+i 0 0; отсюда /=
т. е. =
1.101.	До момента достижения шариком нижней точки выемки брусок стоит; скорость шарика в этой точке
р=/2^ (й+г).
При достижении точки В из требования максимальности высоты и из закона сохранения импульса и энергии в верхней точке подъема имеем
с'верт = 0, (Af-J-m) vrop = m К2g (/г + л), mgr = <М+?) °г.ор;
отсюда /гтах = гт/М = 4 см
1.102.	Потенциальная энергия воды за счет уменьшения высоты ее центра масс на АЛ уменьшится на величину A IF, перейдя в кинетическую энергию тележки и воды:
А = mg A/i = -j- gl cos a = -g- u2 + -g- (v0 rop + t>0 BepT).
Так как относительно тележки вода движется под углом а к вертикали со скоростью о, а тележка движется со скоростью и, то горизонтальная составляющая скорости воды относительно земли v0rop = i>sina — и, а вертикальная — о0 верт = о cos a.
В соответствии с законом сохранения импульса имеем т (v sin а — и) = Ми.
Закон сохранения энергии дает
-g- [(у sin a — m)24-(vcos a)2]-|--g- и2 = -^- gl cos a;
отсюда
m {	gl cos a	\ 1/2
7Л \2(l+m/M) [1 + (1 +m/M) ctg a]/
1.103.	Так как нить нерастяжима, то заданное соотношение скоростей для концов нити возможно лишь при условии, что скорость трубки относительно стола и = о/2 и направлена в ту же сторону, что и скорость половины нити с концом в точке А, т. е. влево. Из законов сохранения импульса и энергии имеем
.. v , т , , ,, ,
М -g--]—g- v — mv -j-Mu ,
Mv2 mv2_mv'2 .Mu'2
T’+"4	2 г-f
144
В системе отсчета, в начальный момент движущейся вместе с трубкой, уравнения выглядят проще:
mv2 Ми"2 । mv"2
~	2 1 Г"
O — Mu"-\-mv", ц'=ц"+-|-'
отсюда
w / 1 //t
2\ ~ V (m + MTfii
1.104.	Средняя сила удара одного шарика о поршень
/?ср = Др/Д/, &p = 2mv = 2m V~2g (Н — h).
В среднем один шарик передает поршню импульс Др за время Д/ = 2г, где т находим из уравнения h = ot+gr2/2. Таким образом,
Mg=nfcp =
ning
(i —нТн'г^2^!:
отсюда
. „	(тп-\-М)2
~ mn(mn + 2M)h
1.105.	Введем горизонтальную и вертикальную составляющие ускорения бруска а, ор п аверт. На брусок действуют сила тяжести mg и сила нормального давления N со стороны клина. На клин в направлении горизонтали действует лишь горизонтальная составляющая силы нормального давления со стороны бруска — N. Учитывая это, можно записать второй закон Ньютона для горизонтального дзиженпя клина н бруска и вертикального движения бруска:
Ma— Л' sin a, пга,-ор= Д'sin a, maBepT — mg—У cos а.
Проекции ускорений бруска и клина здесь записаны в системе отсчета, связанной с горизонтальной плоскостью; при этом направление ускорения бруска не совпадает с направлением вдоль наклонной плоскости, так как сама плоскость движется с ускорением а. Если перейти в систему отсчета, связанную с клином, то здесь ускорение клина равно нулю, а ускорение бруска будет направлено под углом а к горизонту и будет равно по модулю агор + я. На вертикальную проекцию аверт переход в новую систему отсчета не окажет влияния. Таким образом, мы можем добавить еще одно необходимое уравнение:
яверт , ----5—==tg а. агор4*я
Решая совместно систему четырех уравнений, получаем
т sin 2a
а = 8 2(M+msin2a)
145
Можно предложить и иное решение задачи, воспользовавшись законами сохранения энергии и импульса. Введя горизонтальную агор и вертикальную оЕерт скорости бруска, высоту h, с которой он спустился без начальной скорости, и, наконец, скорость клина и, из законов сохранения импульса и энергии имеем
morop + M« = 0,
Переходя в систему отсчета, связанную с клином (в ней он покоится), с учетом движения бруска в этой системе под углом а к горизонту имеем
Vb8pt =tg а.
РгорЧ- «
Если учесть связь между ускорением, путем и скоростью при равноускоренном движении 1>верг ='/'2авертЛ и соотношение между йверт н а, то приходим, используя всю систему уравнений, к ранее поученному ответу:
_ т sin 2<х	.
а~~ £ 2 (М-~т sin2 а)
§ 5.	Статика. Момент сил
1.106.	4 / HBC=arctg (1/2) w 26,5° >.
1.107.	Учитывая, что линии действия всех трех сил, действующих на шар (сила тяжести, сила натяжения веревки, сила давления со стороны стенки), проходят через центр шара, получаем
N	г	... mgr
— ----------------—; отсюда N ----------- -Г7777Г ►>
mg [^+iy-r2]112	[/(/-i-2r)]1/2
Результат можно получить, сравнивая и моменты сил относительно какой-либо оси.
1.108.	Треугольник, образованный отрезками, изображающими силу натяжения нити Т, силу нормального давления и силу тяжести mg-, подобен треугольнику, образованному нитью и радиусами /? и г, проведенными к концам нити и точке соприкосновения шаров (рис. 0.1.108). Из подобия треугольников получаем
T = mg (l-}-r)/R
1.109.	1) Из условия равновесия системы имеем
mig cos (6 — а) = m2g cos (Р + «);
где угол Р определяется условием cosP = //2/?; отсюда, исполь-
,	(ас aA-b c4-d\
зуя известную из алгебры связь 1если у= —, то
146
получаем
cos (fl—a)-f-cos (fl-j-g) cos a cos fl Ictgg nij — m2~cos (fl—a) —cos (fl-f-a)- sin a sin fl ~
Окончательно
4 6 m»+mi
К этому же результату приводит условие, что при устойчивом равновесии центр тяжести занимает самое низкое положение:
tga=^L ОО' = (—^-------О'С=
& О С \т1-^-тг	2 /	г 4
где О — центр тяжести системы тел, О' —геометрический центр стержня, С—геометрический центр сферы.
2) Сила реакции опоры -N д, действующая на точку А стержня, направлена по нормали к сферической поверхности, т. е. по радиусу; сила реакции NB, приложенная к точке В стержня, направлена перпендикулярно стержню (0.1.109). Угол АВО равен a
(О—центр сферы), угол АОС равен 2a. Условия равновесия по горизонтали и вертикали:
Nд sin 2a+ Уд cos a = Mg, Na cos 2a = Уд sin a,
•^y,4 = Mgtga, NB = Mg—— |>.
VvO lA
1.110.	4$ F = F0ctg(a/2)
1.111.	Рассмотрим силы, действующие на одни полуцилиндр, и их моменты относительно линии его соприкосновения с опорой. Так как нить невесома и нет трения, то сила натяжения нити во всех точках по модулю одна и та же и равна силе тяжести, дейст. вующей на неизвестный груз массы М. Отсутствие вращения приводит к следующему равенству моментов:
mgZ-bMmiUgr = Mming.2r; отсюда -4 Mmin=ml/r >.
147
1.112.	Рассмотрим левую половину нити (рис. 0.1.112). Сила тяжести mg/2 направлена вертикально вниз. Сила натяжения нити Тв в нижней точке направлена по горизонтали. Сила натяжения в точке подвеса
4 T=[(mg/2)2 + To2]1/2
Нить в точке подвеса образует с вертикалью угол, равный ^а = = arcctg (mg/2T0) >.
Рис. 0.1.113
1.113.	На бусинку действуют силы тяжести и силы натяжения слева и справа, равные по модулю; отсюда (рис. 0.1.113)
£ ACD= £DCB, AD = DE = x, АС = СЕ.
Из подобия Л ACD и Л ЕВР получаем
AC L — 2AC	яп . „
—=—.—р—, откуда AC = xL!l.
X I 1 хх
По теореме Пифагора для ДЕВЕ имеем
ЕЕ2 + ЕВ2 = ЕВ2, т. е. (/ — 2х)2 + /г2 = (Е— 2xL/l)2.
Из последнего уравнения находим
1 Л h \
2\	К(В/02~1/
1.114.	Из условия равновесия кольца 3 имеем 2mg = 3Fcosa, где F — сила натяжения нити. Из условия равновесия кольца 2 имеем 3F—3F cos a —mg=0; отсюда cos а = 2/3, r/H =Ag а= V5/2, т. е.
4 H = rctga = r-2yT/5 >.
1.115.	В системе отсчета, связанной с ракетой,
Ap = 2mosina, Apx = Apsin a = 2mvsin2a,
148
где х—ось ракеты. Масса пылинок, ударяющихся о ракету в единицу времени, bmlkt—pvS. Таким образом,
◄ FTarH = 2piAS sin2 а
Можно было написать сразу
^тяги = Spy (у — v cos 2а) = 2pv2S sin2 а.
1.118.	Число пылинок, ударяющихся о ракету в единицу времени, пропорционально скорости ракеты о, импульс, переданный ракете каждой пылинкой, также пропорционален v. Поэтому
Т'тяги”^*' сопр С/Э у' V = у2-
Следовательно, чтобы v'/v = 2, нужно, чтобы Ттяги/Ттяги = 4
1.117.	Сила тяжести, действующая на шар, пропорциональна г3, где г — его радиус; сила сопротпвле-пя пропорциональна квадрату скорости шара и его поперечному сечению, т. е. пропорциональна v2r2. При установившейся скорости сила сопротивления равна силе тяжести; отсюда
02Г2 СП Г3, V2 ОО Г, V-t/Vi = \г Гз/И,
т. е. если г2/Г1 = 2, то ц2/у1 = /2
1.118.	Вертикаль, проведенная через точку подвеса, должна пройти через центр тяжести. Можно решить задачу, и не отыска-вая положения центра тяжести, а исходя из того, что моменты силы тяжести, действующей на каждую из половин прута, равны по модулю. Проведя вычисления, получаем один из эквивалентных ответов;
. . 1 .1
•«a = arctg—	или а = arcsm
3	у 10’
3	л	1
или а —arccos	, пли а~-А—arctg —	18,5°
/10	4	2
1.119.	Из условия равенства нулю суммарного момента сил относительно оси, проходящей через точку О приложения искомой силы имеем N — tng/2, где N — сила реакции опоры. Проецирование силы на ось, направленную вдоль стержня, дает
F ц—mg sin ос У sin а = 0, т. е. f,, = (mg/2) sin а ►.
1.120.	Силы реакции опоры NAi в, равные по модулю силам, действующим на гвозди А и В, направлены по радиусам из-за отсутствия трения. Условия равновесия по горизонтали и вертикали дают
Мд sin а—Мд sin р, NA cos a-J-Ma cos $ = mg.
149
Таким образом,
,,	sin В	sin а .	„ ,
«я Ул = то -.	',—NIr=mg .	,—г, 0<p«=n— а >.
ssin(P+a)	ssin(P + a)
1.121.	Введем угол у = л—₽. Тогда
N л cos а-]- Л"в cos y = mg, N д sin a— NB sin y = 0,
где Na и Nb — силы, действующие на гвозди А и 5:
. ,,	sin 8	sin a	„ _ ,
Решение задачи можно провести и другими способами, например написав условия равенства нулю полного момента сил относительно точки О.
1.122.	Запишем равенство моментов сил относительно точки подвеса:
mig (г—x) = m2gx;
отсюда найдем плечо х силы тяжести:
Таким образом, a = arcsin [(1 ~[-l/r) (1 -T^/Wi)]-1
1.123.	Записывая равенство сил натяжения, создаваемых грузами, с учетом того, что эти силы направлены по касательной
Рис.
0.1.125
к поверхности, или равенство моментов сил тяжести относительно вертикали, преходящей через ось цилиндра (моменты сил реакции опоры ври этом равны пулю), получаем
1.124.	Имеем
—- F2L-j~mg (L— /) -|-Afg£/2=0, — /щТ-]- mgl-{-MgL/2 = 0;
. . 2m Л4 (1 — «) , к.
отсюда
150
1.125.	Записывая моменты сил относительно точки А (рис. 0.1.125), получаем
FJ/2 + NJ - mgl/2 = О, FJ/2 - NJ-\- mgl/2 = 0,
Так как Fx = kNi, F2^=kN2, то
г 2	mg-[~F2	.	2F1F.>	-
-p- = -j7-— ~~—f~ > отсюда <«4 m = —=rr = 60 кг fe>.
Ft Ni tng — I-i	gJ-i — FJ
1.126.	Пусть длина ВС равна I. Сила натяжения пружины равна Т, силы нормального давления равны 2, силы трения
а.
Рис. 0.1.126
равны Fi, г. Колодка не вращается, если сумма моментов сил относительно оси, проходящей через точку 5, равна нулю. В соответствии с рис. 0.1.126а имеем
NJ sin a — Tl sin a-J-WiVj cos a = 0, г ,	kT sin a
F i = й-Vj =  ---—-----.
sin a-j-fe cos a
В соответствии с рис. 0.1.1266 имеем
N-z I sin a — 77 sin a — klN2 cos a — 0, r- ,	kT sin a
F 2 = kN2 — -----7-----.
sm a—k cos a
В итоге получаем
T,2 = ^i1f|^^ при k < tga >.
При fc^tga вращение против часовой стрелки невозможно, 1.127. Условие отсутствия проскальзывания:
F cos a k (F sin a-{-mg).
Условие опрокидывания (рис. 0.1.127):
Fl cos a rnglft,
151
где BO — l/2— плечо силы тяжести, ОА = Z cos а —плечо силы Г; отсюда
tga = L^, Fmin==^|/A2+(1_2A)2 при	а —О,
. г- mg	1 _
Fmin = -y- при
1.128. Так как пластина движется поступательно с постоянной
по модулю и направлению скоростью, сумма сил и их моментов
fcosa
'7^77777777^7^77777",
Рис. 0.1.127
Рис. 0.1.128
равна нулю. Точки С\ и 02 — центры квадратов (рис. 0.1.128) — точки приложения сил трения. Пусть вес каждого из квадратов равен mg, а I—длина стороны. Тогда из-за отсутствия вращения, в частности, относительно осн, проходящей через точку А перпендикулярно плоскости пластины,
, К 2 . fn \ . tV2 .(я \ kimgl—^—sin ^-aj = k2mgl-^-sin -j+aj, (1)
/ тс \ тс	,гт	2
sin I ± а = sin cos а ± cos 22 sin а = —(cos а ± sin а).
Таким образом, из (1) имеем
1—tga	k2	.	, ki — k2	,,o .
т.—A— = £, откуда	a = arctg;——« 11
1-J-tga ki'	Ai+^2
(есть вариант ответа:	a = n/4—arctg (fe2/fti) « 11° ►).
1.129.	Имеем
AZi-)-iV2 = mg,	fi = k-yNi, f2 = k2N2,
где Ni и /Va —силы, действующие на опоры, а и f2~силы трения опор о поверхность стола. Уравнение моментов относительно центра тяжести:
(fi + /2)	— N$) lj2 = 0.
152
Си стема уравнений дает
F- mgCfei + M 211+d^-k^l]^-
Уравнение моментов относительно любой из опор, например правой:
N1l-\-Fd — mgll’2, естественно, дает тот же результат.
Сложным и непривычным здесь является, видимо, то, что различные части бруска имеют различные коэффициенты трения и, следовательно, действуют различные силы трения, находить которые нужно с помощью условия равенства моментов сил.
1.130.	Имеем (см. решение задачи 1.129)
ЛС = mg cos а, /у + Л = mg sin а,
где /1 = #1У1,	Уравнение равенства моментов сил отно-
сительно центра тяжести:
(Л + «Л+(Л'2-^1)/ = 0.
Система уравнений дает для искомого угла
«3 tga =
£1 + ^2 2-j-(fei — /д2) /://
► .
1.131.	Сравнивая моменты сил относительно горизонтальной оси, проходящей через точку О' — точку соприкосновения колес с плоскостью, по которой они катятся, имеем
f(r+R) = FTnrR, f = kF-,
отсюда A = FT„ l = kFl (1 -]-r/R)
1.132.	« k = -/.C°S“ R sin ct-|- r
1.133.	Условие равенства моментов отно-
сительно центра катушки (рпс. 0.1.133):
FR = (f1 + F2)-2R, где F1 = kt^1, F2 = kN2. Рис. 0.1.133
Равновесие по горизонтали дает Nt — kN^, а по вертикали приводит к условию +	=	+ Из этих уравнений получаем
F = mg	при	1>.
Условие заклинивания: k > kr = 2 —1 получается из уравнения 1—2/гх— fei = 0) |>.
153
1.134.	Условие равновесия шара по вертикали: mg + 2N sin а = 2k N cos a,
где V—сила нормального давления доски на шар. Равенство моментов относительно оси, проходящей через точку А, дает Mg (L/2) sin a —N (L/2); отсюда
.	m-j-2A4 sin2 a m-i-2M sin2 a
2M sin a cos a M sin 2a
1.135.	Запишем условие равенства моментов сил относительно горизонтальной оси, проходящей через точку О (рис, 0.1.135):
Nl cos a = mg (1/2) cos a—k Hl sin a;
отсюда
.у_.	,п8
2(\+ktga) *
Чтобы сдвинуть лист, необходимо приложить силу F > f = kN. Таким образом,
р • — km8
m,n 2(l+*tga)
1.138.	Условие равенства моментов дает
f (I-h) = V VP-(l — h)* = N Vh (21 —h), где ,V — сила нормального давления, a f— сила трения. Брусок не может двигаться при
Vh(2l-lt) ’
отсюда
(1+fe2)
К14-62 7 ‘
Знак « + » не годится, так как h < I.
1.137.	Пусть Т—сила натяжения троса (рис. условия равенства моментов сил относительно точки новспия подпорки с землей получаем TH — hFotga. вертикальная сила, с которой можно вытянуть столб подпорки и троса, F^Fa. Следовательно,
Т < (h/H) Fo tg а.
0.1.137). Из О соприкос-По условию при наличии
Таким образом,
max = (W) ffltga
154
1.138.	Максимальная нагрузка на стержни и, следовательно, максимальная разность сил —Nit действующих на опоры перекладины, будет тогда, когда брусок находится в нижнем положе, нии. В этом положении сила натяжения каждого стержня равна
Рис. 0.1.135
Рис. 0.1.137
Ti и Т-2 соответственно. Запишем условия равенства сил и моментов сил, второй закон Ньютона для движения бруска по окружности и закон сохранения энергии (длину каждого стержня при. мем равной г):
^1+^2 —Л\1—Л (d-H)-7'2d = 0,
отсюда
Ni-Ni = 5mg	>.
1.139.	Условие поворота:
(L —d)2	d2 . _.
Mg-...-iL < Mg -077 + ^.
где сила Т определяется из второго закона Ньютона для вращательного движения:
тп2 т	,,,	. mv2
—j—=T—mg, mgl(l — cos a) = —%— .
Таким образом,
3 M ( L .
1	COS a — 7;----- -7-:--1
2	2m \ 2d j
При M (L/2d— 1) поворот происходит при любом а, при (L/2d—1) брусок не повернется ни при каком а из заданного интервала углов.
155
1.140. Отклоним игрушку на угол а и посмотрим, что произойдет с центром тяжести. Если он опустится, то положение равновесия будет неустойчивым, игрушка упадет. На рис. 0.1.140
,•—-s	отрезки ОА и О'А' вертикальны,
, /	')	С'В J_O'A', т. е. отрезок СВ
А	- \	горизонтален; легко доказать,
что С'0'В = 2а, а А0'А' = а. Тогда Д/г = СЛ —ВЛ' =
=СЛ-(О'Л'+О'В) =
=	— (2R cos а 4—cos 2а) =
=R (1 — cos а)2 > 0.
Таким образом, центр тяжести опустится, игрушка упадет.
Можно было бы проанализировать, слева или справа от точки опоры при отклонении игрушки пройдет вертикаль, опущенная из ее центра тяжести С, и изучить характер равновесия, рассмотрев возникающие силы и моменты сил. Естественно, что вывод получился бы тем же: при отклонении игрушки возникают силы, которые увеличивают это отклонение, поэтому игрушка должна упасть
§ 6.	Механика жидкости и газа
1.141.	Масса воды, находящейся в воздухе, m — p0Svt, где/ — время движения до падения на землю. Выбирая начало координат на поверхности земли, для движения по вертикали имеем
(/ = 0 = /!-)-osin a-t — f [nsin а + (ц2 sin2 a + Sgft)1/2].
Окончательно
[у sin a + (o2 sin2 a-(-2gft.)i/2]
1.142.	Ручка половника и оставшаяся жидкость должны расположиться в итоге так, чтобы вертикаль OD, проведенная через точку подвеса О, прошла через центр тяжести оставшейся жидкости и была перпендикулярна поверхности (рис. 0.1.142):
OD±_BC, CD = DB, DF = h.
Объем области, занятой жидкостью,
Е = лй2 (г—Л/3).
156
Так как BAJ_OA, ODJ_CB, CD = DB, то
ЛК = Кб = г,	AOD—£ ABC—а.
Следовательно,
sin «=	, h = r—rsina = r( 1-.
отсюда < V= (28л/81) r3
1.143.	Пусть С — центр масс ускоренной воды. Поверхность воды уже не горизонтальна, перпендикуляр к этой поверхности направлен вдоль вектора —a-Pg, направление которого совпадает с направлением результирующей силы, приложенной к центру масс. Если линия действия этой силы пройдет мимо площади опоры, то система перевернется. Критическим условием является прохождение линии действия результирующей силы через шарнир,
Рис. 0.1.143
т. е. через точку О (рис. 0.1.143). Обозначив через хс горизонтальную координату центра масс, через у с — вертикальную, получаем условие ag = Хс/Ус- Центр масс трапеции можно найти, например, через центр масс треугольника и прямоугольника:
Г 1 1 а z_i_ 1 к 1 а \ Л х —1	12 а
L 3 2 ?т2 Г 2g j J hl 2	12/i g ’
[ I a I а ,, I f,	, a \ / a \ , I 1
i/C=L3'7 2^ g	l hi-
ll	I a 7l2 a2
2	2 g~* 24h g2
Подставляя в вышеприведенное условие найденные значения Хс,
Ус, имеем
157
отсюда с учетом условий а < g и h > Z/2 однозначно получаем ◄ Л = 2к7+1+И-&-+з (l—FJ
1.144.	Пусть р—давление жидкости. Тогда Гх=Е-}-р8ъ и F = p(Si—Sa), откуда 4 i —
1.145.	Используя закон сохранения энергии и условие несжимаемости жидкости, получаем
= PogSh -^PogS -7- > “ =
mtr
где р0 — плотность воды, и—скорость истечения воды. Тогда
Г / Q \ 2 Ч	О
< А= FK— F„ = p0S-j Циу j — gftj для v > V 2gh -у
При v< V"2gh.Sa/S происходит отрыв воды от поршня.
1.146.	Условие равновесия стакана:
Учитывая закон
S (х—h) p1g+mg = p2gxS.
Паскаля, получаем
(р2 — pi) S
расстояние h от верхней грани куба до поверх-имеем
1.147.	Вводя ности жидкости,
Pg^3 = [pig (Л+Z—x) + p2gx] /а —Pig«2; отсюда
«4 х=/-£—£1- >.
P2-P1
1.148.	Запишем условие равенства моментов силы давления воды и силы тяжести, действующей иа тело, с учетом отсутствия подтекания. В качестве оси вращения выбираем правое ребро нижней грани:
^-[2-4/3 + (h-4Z) Z2] = 3Z3 (2рр) g-у-, < h = 5Z >.
1.149.	Пробкой вытесняется объем воды nr2Z. Следовательно, уровень воды повышается на ДА (объем aR2A/i = №Z). Изменение высоты центра тяжести воды равно ДА/2 + А—г. Искомая работа равна изменению потенциальной энергии вытесненного объема:
/ »-2/
◄ 4 = nr2Zpog^^
•f-A —rj
158
1.150.	Обозначим высоту уровня жидкости над пробкой через Н. Условие отсутствия движения пробки:
PcpS f+PoS,
где PcP = Po+pg(#+A/2), S = dh, f=k(pgH+p0)ld; отсюда
Это условие для высоты уровня жидкости над пробкой справедливо при
kl<h< y^kp^l/pg, если k > 4p0/pgl}
У2kpul/pg < h < kl, если k < ipo/pgl,
1.151.	<1 F — pgP cos a-f-pog^ P»+ (2/2) sin a] . Первое слагаемое— составляющая силы тяжести, действующей на кубик, второе— сила давления столба воды над кубиком; эта сила давления направлена по нормали к грани.
им г Swpg „ SHpg SHpoa 3SHpg
1.152.	F1==—F2=—-----------^i=3F± = —-Ti; отсюда
4 P = Po/2 >.
1.153.	Полная сила давления
F== Pogft2^
2 sin a
направлена по нормали к бруску. Брусок находится в состоянии покоя, если
р0[>№1	, (	, Pog^T .	\
 2 =k ( mg-\- etg a j ,
откуда
^fe==_e^_=0,i.
PogA2Z + 2mg
Брусок придет в движение, если k < 0,1 |>.
1.154. Введем I—расстояние, на которое выступает пробка над плоскостью дна. Из подобия геометрических тел P/h2 = S0/S. Максимальная выталкивающая сила возникает, если вода лишь доходит, до верха пробки. Тогда
hS lSa
(Ро ~ Р) g ~з-Pog ~ — Pog (/! - 0 So = °-
Исключая Л, получаем
4 р = ро [ 1 + 2 (S0/S)3/2 _ 3s0/S] ►.
169
1.165.	1) Условие поворота пластинки относительно правого края трубки:
(Р — Ро) gn.R*h (d+r)^ pognr3 (Я—х).
При выполнении этого условия пластинка начнет поворачиваться, вода войдет в трубку, равновесие нарушится и пластинка упадет, если сила тяжести, действующая на нее, больше результирующей выталкивающей силы; отсюда
х Н-(R/г)^ h (1 + d/r) (р/ро -1).
Пластинка падает без вращения, если
(Р — ро) gnR2h	Pognr2 (Н—х),
x^H + (R/r)*h (1—р/ро).
Сравнение ограничений, накладываемых на х, приводит к следующему результату:
x^H-/R/r)2h(p/p0-l) >.
2) Пластинка перевернется и всплывет при нарушении равенства моментов относительно левого края трубки:
(Ро—р) gnO (d—г) •  pog№ (tf— x), x^H+(R/r)2h(\— d/r) (1—р/ро)
1.356. 1) Из геометрических соображений и из определения центра тяжести следует, что плечо выталкивающей силы относительно правого края трубки равно а/6-f-r. Клин упадет на дно, повер' нувшись, если
(Р~ Po)ga24	S=Pog№ \Н — х),
х^Н f 1 — А аа	f£+lk
1 \	р0/	\6r J
Чтобы клин упал без вращения, необходимо выполнение условия для силы
- Г 9/lj ч з а2Ь ] а2Ь	гт t a2b (, р\
Сравнение ограничений, накладываемых на х, приводит к следующему результату:
.	а2Ь ( р , \
<	——а — — 1
2лг2 \ ро ) г
160
2) Плечо вытекающей силы относительно правого края трубки равно а/6—г. Клин перевернется и всплывет, если
1.157.	mglS^ipgh, A = m/2pS
1.158.	Жидкость, перетекая из'левого колена через отверстие в правое, разорвет столб жидкости: ту жидкость, что была над отверстием, поднимет, а ту, что была под отверстием, опустит
Рис. 0.1.158
(рис. 0.1.158). Из условия равенства давлений в левом и правом коленах на уровне отверстия следует, что ниже отверстия жидкости расположатся одинаково. В левое колено войдет столб другой жидкости высоты й0/2 и столько же войдет другой жидкости ниже уровня отверстия справа. Таким образом, уровень жидкости в правом колене изменится только на величину х, равную изменению уровня в левом колене.
Обозначим h высоту уровня жидкости справа выше отверстия до начала перетекания, Н — высоту уровня жидкости слева. Из условия равенства давлений внизу (Л + Ло) p2 = #pi, из условия равенства давлений на уровне отверстия, после того как перетекание прекратилось, /1р2 + -ф1 = (# —^о —*) Pi имеем
1.159.	Выразим давление через среднюю плотность жидкости:
p = pcpgh=p0
1.160.	Выразим массу жидкости через среднюю плотность;
At=pcpftos=2i±£M=pl)Ao (1+^)
6 Г. В. Меледии
161
отсюда
a/(5 + 2/i0—^- = 0,
. М Л Л° ~ "ёТ" 1"
h
аМ X
2<$Ро /
при аМ < Sp0.
При условии плавания кубика его погружение эквивалентно увеличению общей массы жидкости:
h==
2 (Л1 + т) а X 1/2 Spo J
а
Окончательно
a L \ ' Spo /	\ ' Spo /
,, т Г. а(Л4 4-т/2) I	_ о
АЛ « — 1--------------1=------ при аЛ4 < Sp0.
оро L	р0Л J
1.161. Сравниваем моменты сил относительно центра погруженной части бревна:
mg//4 = Т’-3//4,	7’ = /ng/3 = 8-102 Н ,
1.162. Условие равновесия шара дает
Р8У — p0gV72 = pg-4К/3,
где ро — плотность воды, р — плотность шара; отсюда
4] р = Зро/4 = 0,75.103 кг/м3
1.163.	Условие равновесия дает
т	tn	Pi	Pi ь.
Plg-v—mg = p2g-; mg,	=
Pl	Р2	Pl	Р2
1.164	Используя	равенство	моментов	сил,	получаем
р2— 3pi/4-|-pi/4 — ро = О, отсюда < Ра = Ро + Р1/2
1.165.	mrain = -^r3po G+y-') >.
1.166.	Обозначим через а угол, который нить образует с вертикалью: sin а= 1/2, а = 30°. Выталкивающая сила вдвое больше силы тяжести, т. е. вверх на шар действует сила FA~mg. Сила натяжения нити Т, сила реакции стенки N и сила FA, векторно складываясь, дают нуль:
^ = tga=l/K3‘.
Сила, с которой шар давит на стенку, равна по модулю силе N, отсюда	N = mglY3
162
1.167.	Из равенства моментов сил следует, что
N sin —mg — cos а, т. е. N=~~^~ C*S а
1.168.	Тонкий слой жидкости полностью передает ее давление. На призму действуют три силы: сила тяжести, сила сопротивления и выталкивающая сила, сумма которых равна нулю. Таким образом,
fconP = (p2 —Pi) gVsin а $>.
1.169.	F = (2/3) nR3 (pi + р2 — 2р0) g = 2,5-10-2 Н из усло-
вия равенства сил, действующих на каждый шарик.
1.170.	Деформация пружины пропорциональна нагрузке. Тонкий слой жидкости передает давление—в результате можно находить выталкивающую силу обычным путем:
(Ра — Pi) Vg . х~ 1о	, ,, , . ( Рг Л.)пяк1
pi к g	— ‘0	\ Pl /
1.171.	В отсутствие воды для движения центра шара вокруг оси цилиндра с угловой скоростью со необходимо, чтобы со стороны цилиндрического сосуда на шар действовала сила
Р — (4л/’3) г3рсо2 (R —г).
Если сосуд наполнен водой, то
Е = (4л/3) г3(р — Ро) “2 (R — г) >.
1.172.	Внутренняя воздушная цилиндрическая полость, образовавшаяся при вращении сосуда с водой, имеем объем nR2h/4, т. е. расстояние от оси полости до поверхности воды равно R/2. Расстояние от оси вращения до центра масс стержня, таким образом, равно P/2-J-//6, а расстояние до центра масс части стержня, вытеснившей воду, равно P/2-J-//3. Центростремительное ускорение стержню сообщает действующая на него со стороны жидкости выталкивающая сила. Точно такая же сила действовала бы в отсутствие стержня на жидкость в объеме погруженной части стержня и сообщала бы этой жидкости необходимое центростремительное
ускорение:

отсюда
.	2 ЗЯ + 2/
р— 3 Ро + /
1.173.	Потенциальная энергия бруска в обоих случаях одинакова, а потенциальная энергия воды—нет. Как несложно убе-6*	163
воды до центра тяжести бруска.
Рис. 0.1.174
диться, равновесия в промежуточных положениях между указанными положениями нет. Поэтому равновесие бруска устойчиво в том случае, когда центр тяжести вытесненной воды расположен выше: если сторону квадрата в сечении бруска принять за единицу, то
Лу = 1/4, h\\ = 1/3 j/”2 , h\} < h\, где h—расстояние по вертикали от центра тяжести вытесненной Значит, положение // устойчиво. К такому же результату приводит анализ устойчивости, проведенный исходя из сравнения вращающих моментов сил при небольшом отклонении бруска от положения равновесия.
1.174.	К моменту времени, когда растаяло 70 % льда, остался пористый лед, имеющий толщину слоя h (1—Р). Слой воды, проникший в поры льда (рис. 0.1.174), имеет толщину	= h (1 —р) р/р0.
Тогда масса воды в порах льда
«о = (1— а)ро$Л1 = (1— «) (1—6) phS.
В результате таяния образовалась масса воды т'о — ap/iSp.
Если т'о — т0 > 0, то лед всплывает [(т. е. для всплывания льда должно быть « + 6 > 1, что реализуется по условиям задачи: ос[3 = 1,3). Высота уровня воды под слоем льда т’ — т0	0
h2 =----р—=й (a+P —1) —.
р|О	'	Ро
Искомая полная высота уровня воды
# = /i! + /i2 = M(a + 6—1) + (1—₽)] — = ah —=0,54й £>.
Ро Ро
Если убедиться в том, что в результате таяния заданного количества льда оставшийся лед всплывает, то ответ можно получить сразу. Действительно, если лед плавает, то от его таяния уровень воды в сосуде не изменяется: лед, плавая, вытесняет воду массы, равную той, что имеет сам. Значит, при таянии образуется столько воды, сколько вытеснялось, и уровень воды не изменяется. Исходя из этого, предположим, что весь лед растаял, и найдем результирующую высоту уровня воды:
hSap = HSp0; отсюда Я=айр/ро = О,54Л
164
1.175.	По закону сохранения энергии имеем (Ро—р) gl (1+ К2 );
1.176.	Введем сечение трубки S. В заданный момент
palS = pghS-, отсюда a = ghjl.
Столб жидкости высоты Н приобретает ускорение а под действием рилы тяжести и силы давления со стороны жидкости, подпирающей столб:
paHS — pgHS—pS; отсюда P = pgH(l—h/l)
1.177.	Пусть p0oS— масса воды, поступающая в единицу времени из насоса при скорости истечения и через сечение S. Тогда Мощность насоса без дополнительной трубы
При наличии трубы высоты h мощность насоса
P = £2^+Po«S^.
где и — скорость воды, протекающей через выходное отверстие дополнительной трубы. Наконец, из кинематических соображений имеем
□2 = 2gtf, u2 = 2g (Я—Л).
Решая совместно эти уравнения, получаем
Можно решить задачу значительно проще, заметив, что иапор воды в обоих случаях одинаков, так как полная высота фонтанов одна и та же. Тогда, используя формулы P = uF, P0 = vF, получаем сразу
1.178.	Сила давления на брусок одной струи F = bp/kt = p0o'2S\
Тогда
mg = np0o'2S' = np^vSv',
165
так как из условия неразрывности 'струи следует, что Наконец, из кинематических соображений имеем
о'2 = о2 — 2gh.
Решая совместно эти уравнения, получаем
1.179,	Если стержень движется со скоростью v, то, так как жидкость несжимаема, она должна протекать в противоположную сторону между стержнем и стенками трубы со скоростью и, причем 8ги = S2u, т. е.
лг2а = п (R2 — г2) и; отсюда u — vr2/(R2— г2).
Скорость жидкости и предполагается одинаковой всюду, кроме небольших участков около торцов стержня. Однако если длина цилиндра / > г, то вкладом этих участков в энергию системы можно пренебречь.
Из закона сохранения энергии при подъеме стержня на высоту h следует
р2лгг/ъ,2/2 + р1л (Р2— г2) lu2/2 = (p1 — pi) nr2lgh.
Таким образом,
Г 2gft (1 — Р2/Р1) ] V2 „ yi = g(l—Рг/pi) . iPz/pi + r2/^2-'2) J ’ ' 2й рг/Р1 + гЖ2-г2)
1.180.	Изменение импульса выделенного объема жидкости за малое время
pS2v2 Д<в2 —Р$Л Д^о1 = (81р1 — ЗзРг + ^о) М,
где pSjVjA; — масса жидкости, прошедшей через сечение S-f за время Д/, Fo— сила, с которой труба действует на жидкость. Искомая сила
F = —F 0, F — —Sfpi 4-S2P2 —р (SjVj—S2v2), ◄ F = S2p-2—Sipi — pSjt/i (1—S1/S2)
Закон сохранения энергии для объема жидкости, протекшей за время At между сечениями / и //, дает
AmvZ Amv2
P1S1V1 At — p2S«v2 \t = —9----5----1- Д Q.
’Здесь в левой части уравнения — работа силы давления, произведенная над интересующим иас объемом жидкости, а в правой части —изменение кинетической энергии 'этого объема жидкости и количество те<уюты, выделившееся между сечениями I и // за 166
время ДЛ Учитывая неразрывность течения жидкости, получаем Дт - pSi^! Д/ = pS2o2 Д/
и, окончательно, количество теплоты, выделившееся в единицу
времени

1,181.	Пусть скорость движения поршня равна и, скорость движения поверхности S равна и. Тогда за время Д< в результате смещения поршня на расстояние и А/ в движение со скоростью и придут частицы среды, находящиеся в объеме vAtnr2 (их плотность равна р!, остальные частицы, плотность которых равна P2^-pi, уже движутся со скоростью и). Таким образом, из условия сохранения массы следует, что вещество, находящееся в объеме оД^лг2, 'при смещении [поршня на расстояние у должно распределиться по объему и М яг2, причем здесь оно будет иметь плотность р2—ру:
ир3 М яг2 = и (р2 — pi) Д/ яг2, и = ир2/(р2 — Pi).
Импульс частиц в объеме v А/ яг2 равен kt яг2и и равен импульсу силы F:
F kt pjo Д/ яг2и = яи2 -PjPL. ду
Р2 — Pi
отсюда

v Г^(Р2—Pl)!1/2 L л^РгРа J

1.182.	Скорость жидкостей максимальна, когда они проходят положение равновесия. Из закона сохранения энергии с учетом того, что в положении равновесия p1/!i = p2/!2 (гДе hf п h2—высота столба жидкости в одном щ другом коленах трубки, когда жидкости находились в первоначальном положении; р£ и р2— плотности жидкостей, налитых в трубку), получаем vmax = ^e^i
Глава И. ТЕПЛОВЫЕ ЯВЛЕНИЯ
§ 1.	Газовые законы. Тепловое расширение
2.1.	a = p0S/16m
2.2.	Масса жидкости m = pZfS/2-f-pftS/2. Введем давление Запертого между стенками воздуха р и р0—атмосферное давление.
167
Условие равновесия и закон Бойля—Мариотта дают Р = Ро + Р§(Н—ft), р(Я—Л) = р0Я;
отсюда
◄ Ро=
4g (pHS— m)2 S 2m — pHS

2.3.	Скорость максимальна в момент, когда поршень проходит положение равновесия. Таким образом,
◄ Р = Ро+^ 0
2.4.	Из условий равновесия в начале и в конце нагрева получаем
mg + Mg—иг2 (Pi —ро) = О, mg —пт2 (р2—ро) = О.
По закону Шарля имеем
Ро Ра
То Л ’
т т Т Ро —(m + A4)g/nr2] Тг~Т°
2.5.	Зависимость площади дна сосуда от температуры имеет вид
$ = $0(1+а/)2.
Масса жидкости не изменилась. Поэтому
р==—£2— ~ __£»— ь. ** ' (J +ct/)2	1 + 2с:/
Попытки прямо учитывать расширение жидкости в итоге дают тот же результат.
2.6.	Внешняя труба А и стержень С в сумме удлиняются на AZ = 2a]ZA7'. Внутренняя труба В должна компенсировать это удлинение:
2axZ А7, = а2Т АТ, откуда L~2la1/ai ►.
Так как L < I, это возможно лишь при a2 > 2at.
2.7.	Так как лист удерживается на крыше силой тренпя, очевидно, что при его нагреве имеются точки листа, неподвижные относительно крыши. Обозначим расстояния от них до нижнего края листа х0. При сжатии листа в процессе охлаждения будут неподвижными некоторые другие точки листа. Обозначим расстояния от этих точек до нижнего края листа х'.
При нагреве любая часть листа, расположенная ниже неподвижных точек, сползает вниз, а часть листа, расположенная выше, движется вверх. В результате этих движений на верхнюю часть 168
листа действует сила трения
fi = kmg —cos <p,
направленная вдоль вует сила трения
крыши вниз, а на нижнюю часть листа дейст-
f2 — kmg cos ф,
направленная вдоль крыши вверх. Имеется также составляющая силы тяжести mg sin ф, направленная вдоль листа вниз. Так как лист нагревается медленно и его части движутся с очень малой скоростью, то с высокой точностью можно предполагать, что лист в любой момент времени находится в равновесии. В частности, сумма сил, направленных вдоль крыши, равна нулю:
— kmg ( 1 — -у J cos ф-J-^mg-p- cos ф — mgsin <р —0,
откуда
х0 = /
sin ф + & cos ф 2k cos ф
Таким образом, при нагреве нижний край листа сместится
относительно крыши на расстояние
Лг	(f , . 8Шф + & СОЯф
При охлаждении размеры листа сокращаются, нижняя часть листа движется вдоль крыши вверх к неподвижным точкам, а верхняя — вниз. Это движение приводит к изменению знаков у сил трения. С учетом этого изменения получаем, что расстояние от неподвижных точек до нижнего края листа при охлаждении
,__ Ik cos ф — sin ф
%0 2k cos ф ’
т. е. нижний край листа поднимается при охлаждении на расстояние
. , а.1	, ,&созф —sinw
Ах' = йе (^2 — ^1)----------•
0 2k ’	cos ф
За одни сутки изменение положения нижнего края листа
1	ctZ
AZ = Дх0 —Ах' =— (Z2 —/1) tg ф.
169
За п = 30 сут устойчивой погоды смещение будет в п раз больше:
AL=-g-(/2 —/t) tg <р = 8 см
2,8.	Уравнение второго закона Ньютона для движения поршня по окружности:
тсо2х= (р0— р) S.
Из закона Бойля—Мариотта имеем
рх = p-tl,
где р—давление между поршнем и запаянным концом вращающейся трубки; отсюда
Очевидно, что должно выполняться условие 4то>211ра8 < 1, иначе поршень из трубки вылетит.
Для того чтобы -ыясипть, оба ли корня уравнения подходят, обратимся к рис 0.2,8, на котором изображены графики двух функций:
тег , , , У1 = —-?х2 + /, уг = х.
Ро$
Пересечение этих графиков дает два корня Xf и х2 исследуемого квадратного уравнения. Нетрудно видеть, что корень хг соответ-
н
ствует неустойчивому положению равновесия поршня, а корень х2— устойчивому. Следовательно, нз физических соображений подходит лишь один корень, соответствующий знаку «—» в скобках. Таким образом,
PqS Г, Л	1/2 1
2та>2 [	\ Pt>S /	] ’
х ~ I (1 -ф- при ты21 < paS
Рис. 0.2.8	' р»ь '
2.9.	Условие равновесия столбика ртути: Po = pg(H—й) + и, где рг—давление воздуха над ртутью, a S—сечение трубки. Закон Бойля — Мариотта даетТрг = p0V/Sh. Из этих двух уравнений получаем
170
откуда
2.10.	Система вначале находится в состоянии равновесия, по» этому
Ро + pgHi= Р1>
где pi — давление газа под поршнем в начальный момент. Из за» кона Бойля—Мариотта следует, что
PiAS = (p0 + pg/72) (//j + й —х —Я2) S.
Таким образом,
2.11.	Воздух не проходит в отверстие, пока толщина слоя воды удовлетворяет условию
Ро + Pog (J1 — х) — Ро	’
('2Л4-£2-К + Ла = 0, x = h+-^-Г U+тДУ-Л2] >
\ Pog}	1 2p0g |Д 1 2pogJ J
Знак «+» отброшен, так как х < h. Окончательно имеем
4x=h+±при
2 Pog L \ Ро J J Ро	Ро
2.12.	Закон Бойля — Мариотта дает
Poh=-P (h—x).
Из условия равновесия воды на дне сосуда следует, что
Po + pog* = P + Pogx;
отсюда
x2_(P^A.l+h'\x + hl = 0. \P0g	}
Окончательно, контролируя ответ при малых I и h, получаем
4 х=1 /—4-/4-л-Г( ₽»-4-/+лу_4й/Г/21 и
. 2j \pog ‘ L\Peg J J J
« 2h £^L при < 1 >.
Po	Po
171
2.13.	Условие равновесия воды (рис, 0.2.13):
Po + pgh=P+pghi.	(1)
Сохранение объема воды приводит к условию
3ft/2+3ft1/2==/7.	(2)
По закону Бойля—Мариотта имеем
p0HS/2 = p (H-hj) S/2 + р (H-hi) S.	(3)
Из (2) ht = 2H/3 — h, из (3) рйН = Зр(Н—hi') = p(H — 3ft), т. e. p = p0H/(H — 3h). Подставляя полученное значение p в (1), имеем
, р0Н .	277 —3ft
Ро + pgh - н 3h + pg з , откуда
◄ й=^( при ►
2.14.	В первом шаре давление р1 = сс/7?1, а во втором — р2 = — Pi + a/Rt- После того как остался один шар, давление p = a./R, объемы газа соответственно равны
y1 = 4n(/?J —Rl)/3, V2 = 4nRl/3, V = 4nR3j3,
По закону Бойля—Мариотта находим
Pi^i + РгУг = pV\ отсюда R =	+ Rl >.
2.15.	Из условий равновесия получаем
PoGS = (po + Ap) /2S = (Po — Др) xS, где Др — изменение давления газа при повороте пробирки. Если Z1^2/2, то капля выпадает. Таким образом,
Х = ‘Zl^— Zt ' ^2 <	< 2^2
2.16.	Вначале
р£Р=О, pAp,=--p^2pg’i: PcD = Pgh = Р/%-
После поворота трубки
PC'D' = P1, PP—Pl + pft, РАВ' = Р2,
где АВ’ = х, CD‘ = 3h — х. Закон Бойля—Мариотта дает
(р/2) ft = pj (3ft—х), ph = p2x.
Из условия равновесия капелек ртути имеем (рис. 0.2.16)
Pi — Pi^pgh — р/2, PF = Pi + pgh = pl + P/2.
172
Для pF получаем уравнение
брр — 12ррр + 5р = 0, откуда р/? = р(1 ± 1/К б).
Знак «—» отбросим, так как давление растет. Окончательно
4 p> = p(l+l/f'6) >.
2.17.	Так как
mg S
mg .
"25+^°’
то правый поршень сразу опустится до горизонтального участка трубки. Введем высоту поршня в левом колене х. Тогда по закону Бойля — Мариотта имеем
р (2х + 3/г) S = p0.6hS.
Из условия равновесия поршней следует, что
p = po + mg/2S.
Таким образом,
.	3 . pa — mg/2S
X=2hl^+^S- ПрИ mS/2S < р°
(при tng/2S < р0/5 левый поршень перемещается вверх: х > Л),
х — 0 при mgl2S^s р0
2.18.	Введем искомое смещение х, начальную и конечную высоты уровня ртути и Л2, начальное и конечное давления
Рис. 0.2.16
воздуха pt и рг, плотность ртути р. Условие неизменности объема ртути:
(1-f-X).
173
Из условий равновесия поршня до и после изменения температуры
,	, hi , hi
Pihi=Pi^i^=Pg — h1 = pg—, i	г,	i hl
pih1=ptcpfii = pg-^- h2 = pg-^~
имеем
Р1/Рг = (hi/hz)*.
Используя уравнение газового состояния, получаем
Pi T1V2 _Ti(l—x)_3(l—x)	/ hi V _ ZZ + xV
77~'tv7~—tv	1 \jy j \ i j 1
отсюда
„	/33-5 ,
x =----2---I |>.
2.19.	(pi ± Др) (V TA V) =—RT, p2 (2V T AV) = 2 — RT,
Знаки «+» и «—» соответствуют вдвиганию или выдвиганию поршня.
2.20.	Из условия равновесия поршней начальное давление в верхнем и нижнем отсеках равно соответственно 2/?0 = р0 Ц- mg/S и 3/>o = po + 2ffl£'/S. Используя закон Бойля—Мариотта, получаем
3pt)lS = х (F~'-2/y,-S), 2palS = (I-—х) (F -(-p0S),
где х — искомое расстояние, F — сила дополнительного давления на верхний поршень; отсюда
<( х—1 (3— У б) = 5,5 см |>.
Второй корень уравнения не удовлетворяет условию х < I, 2.21. Введем смещение поршней х. Объем воздуха между поршнями после их смещения
V' = Ио+х (S2 — Si),
Из закона Бойля —Мариотта имеем
Ро'/о = Р'С',
где р' — давление воздуха между поршнями после их смещения. Из условия равновесия стержня получаем
(Р — p')Si=(pa — р') S2.
174
Совместное решение этих уравнений дает
Ур	Р — Рр
Si — S.j P+P0S2/S1
2.22.	Пусть внешнее атмосферное давление увеличилось до значения р. Тогда поршни сдвинутся вправо на расстояние х. По закону Бойля — Мариотта имеем
ppV = p (V — Szx + Sjx), откуда	х=
2.23.	Условия равновесия до и после изменения температуры дают
PiSj — P2S2, piS{ = piS^.
Уравнение состояния газа дает
V1P1	V'tpi V2P2	V2P2
тп ~ г; ’ л - Т’ 1
Учитывая, что Vi/Vi = 3/2, 1/:/У2=1/2, получаем
t^/t'^vIvz/ViV^s, <4	>.
2.24.	Условия равновесия для пробок дают (рис. 0.2.24)
Р1$ — (~Урр$< PiS — f — pjS, где f—сила трения между поверхностью трубы и пробкой.
По закону Бойля—Мариотта
ISpo = pixS, ISpo = p2(l—x) S,
Таким образом,
<4 f=Pos/Vr 2 >.
2.25.	Условие равновесия левого поршня (рис. 0.2.25)5
paS—хх —* piS=0.
in
176
Условие равновесия правого поршня: P1S + F~P<)S = 0.	(2)
Из закона Бойля—Мариотта имеем р01=Р1(21—х).	(3)
Из (1) и (2) следует, что F — V.X, т. е. x=F/n. Из (2) р1 = Ро~-F/S, из (3) Рй1 — (рй — F/S) (2/—f/x). Таким образом,
F2—F (2kl + p0S) + PolkS = 0; отсюда
4 F = x/ + pos/2—K(xZ)2 + (p0S/2)2 > (F—i-O при x—► 0).
2.29. Следует рассмотреть два случая: 1) kmg^v.l, 2) kmg<v,l, 1) Камера покоится:
,	. с , о Sl Sl
(p — po)S = *l, Р-2-^г = Ро — -,
отсюда
Т	[	х/ \
<4 -~- = 2 Н—) ПРИ kmg^nl
' 0	\	рЪ J
2) Камера покоится до момента достижения максимального значения силы трения покоя. Найдем соответствующую этому моменту температуру Г'. Деформация пружины x = kmgln, кроме того
kmg PS(l-px) PliSl
P~Po—~S~’ г-------------
T^( ,kmg\ Л To \+PoS)V
То *
После возникновения проскальзывания камеры процесс увеличения объема идет при постоянном давлении:
Т V 21S	2
Т’'"_7Г— (l+x)S ~ l + kmg/пГ
Подставляя в это выражение значение Т', получаем
4 тгг-=2 (1	при kmg < и1 >.
10	\ Р(Р /
2.27.	4 m = Ph (PoS-{-mg)lRT
2.28.	F = mg(PB/P—!) >.
2.29.	За время t в трубку входит объем воздуха Ki = SiV1Z, а выходит объем К2 = 52У2К Так как это равные массы, то из обобщенного газового закона PiVt/Ti = Р2Уо/Т2 находим
Pi Тг Si
◄ У2’=~п"т~ S" V1
Ра г 1 оз
176
2.30.	В сосуде, падающем свободно, газ находится в состоянии невесомости. Поэтому искомая разность сил давления F = mg, Массу газа можно найти из уравнения газового состояния:
pV = ^- RT = £^- Т.
Ц То
Таким образом,
F=mg = p.g-^^ >.
Pov о1
2.31.	Имеем p2/p1 — m2/mt = n, Pi = pi-mg/S', отсюда
<4 pi — mg/S (n— 1), pz = kmglS (n— 1) >.
2.32.	Газ просачивается сквозь поршень, пока его давление по обе стороны не станет одинаковым. Тогда его можно не учитывать при сравнении сил, действующих на поршень. Тем самым задача свелась к совсем простой: в одном отсеке давление равно нулю, в другом (длины L-^-x) давление стало р'. По закону Бойля — Мариотта имеем р’(L-\-x) = pL. Условие равновесия поршня дает F = p'S; отсюда
4 x — L (pS/F— I) при pS/2 < F < pS, х = 0 при pSs^F, x = L при pS!2^F
2.33.	Условие равновесия поршня до проникновения гелия:
Рне + тЯ/5 = РОа,
где ро —давление кислорода. После выравнивания концентрации гелия по всему сосуду имеем
PO, = mg!S.
Закон Бойля — Мариотта дает
pHJ.S = 2p'H<JS, pQhS = р’о^ (h+x)S.
Окончательно для смещения поршня получаем
, ptfeS	mg .	mg .
◄ X=/l"rni- ПрИ ₽не< S ’ X = h ПрИ	>•
2.34.	Уравнение состояния газа:
paSH _ pSh То ~ Т '
где h — расстояние от дна перевернутого стакана до уровня втянутой в него воды. Условие равновесия втянутой в стакан воды:
pS-<-mg=paS, h = H^m/paS;
ITT
отсюда
«-350 к -80 °с
§ 2.	Пар
2.35.	Из закона сохранения энергии и условия равенства объемов платины и испарившейся воды имеем
-2	р2 С2
2.36.	Запишем! баланс тепловых энергий:
(Л —/2) ~с<?2 U —^2);
отсюда
t	\-Я1Л:л-1к1ъ <3 1=72°С>.
l?2 '	\	<?2 /	Q!2.
2.37.	Уравнение газового состояния: .. пт рК =— RT, Iх
где р, V=--Vo — .М/о0 и it — давление, объем и молярная масса пара (р—105Па, V=i л, ц = 0,018 кг,-моль, Т = 373 К); отсюда
т = upV;RT = 0,59 г
2.38.	Давление воздуха в котле при 100°C pt = 2>105 Па, так как насыщенный водяной пар при этой температуре создает давление р2 = 10’’ Па. Используя условие неизменности объема (объемом воды, получившейся при конденсации охлаждающегося пара, пренебрегаем, так как он мал), получаем
р'=Р172/Г1 = 1,5-10? Па
2.39.	Насыщенный пар при 100°C создает давление, равное атмосферному, ро=1О? Па. Из уравнения Клапейрона—Менделеева получаем
poh__p(h + x)
Т’о Т ’
где р—давление воздуха под цилиндром при 100°С, Т0 = 273 К,, Т = 373 К, х—искомое смещение уровня воды. Равенство давлений на глубине х под водой дает Ро + р = Ро + Ро£-1с. гДе Ро—плотность воды. Учитывая, что Pa*=PogH и Н— 103 см, получаем уравнение

178
Окончательно
х = (1/2)(/Л24-4ЛЯ7’/7’0—ft) « VhHTjT0 х 37 см. >
2.40.	При температуре Z2 =100 °C давление пара воды р2 = = 105 Па —рх/3. Следовательно, парциальное давление воздуха под поршнем станет равным 2рх/3. Из уравнения газового состояния найдем новое расстояние х от поршня до поверхности воды (Л = 279 К, 7’2 = 373 К):
Pih 2р,х .	3 Т2 ,	л ,
<< х = у yrh х 2h х 4 см >.
2.41.	При изменении объема от V до V/2 пар сжимается, но не конденсируется. Далее происходит конденсация. Причем давление насыщенного пара при дальнейшем уменьшении объема от V/2 до V/4 остается постоянным и равным 2р. Поэтому сконденсируется половина моля пара, т. е. m = 0,5vp = 9 г
2.42.	Пар под поршнем должен частично сконденсироваться. Давление его после начала конденсации и до конца ее равно 2р. Над поршнем по закону Бойля—Мариотта давление легко вычисляется:
V	V 4
₽1-₽ Их — Р V + 3V/4 ~ 7 р'
Из условия равновесия поршня имеем
7	4 \	10
PxS-|-mg = 2pS, откуда < mg—(2—^-jpS=ypS >.
2.43.	Из-за отсутствия трения из условия равновесия для левого поршня давление pt слева и справа от него должно быть одним и тем же. Тогда и объемы в левом и правом отсеках до конденсации пара должны быть одинаковыми, т. е., пока рх<2р, длина каждого отсека равна. (2L —Z)/2 = T—Z/2. Таким образом, смещение левого поршня x = Z/2 при pj sg 2р (Z<L)
При дальнейшем вдвигании поршней пары воды конденсируются и давление, одинаковое в обоих отсеках, будет постоянным и рав. ным 2р. Значит, и расстояние между поршнями Z'будет постоянным и равным L/2, так как по закону Бойля — Мариотта pL = 2рГ. При этом 2L — I—L[2 — x, откуда
х = ЗТ/2— Z при LsgZ<3£/2 (объемом конденсировавшейся воды пренебрегаем),
x — L при I > 3LJ2 ►,
179
2.44.	Давление р насыщенных паров воды при постоянной температуре остается неизменным:
.	и l+2h
4 р = ро—Pogft
2.45.	Масса пара равна массе воды под поршнем, поэтому
2.46.	Так как пар иасыщеннный и конденсируется, то при сжатии его давление постоянно. Работа по сжатию
А~	= pSM=p&.V = р (V— Уо),
где V и Ро — конечный и начальный объемы пара. К пару можно применить уравнение газового состояния:
pV0=^RT, pV--—R7\ ц	|Л
Это дает
Я п; г; \	--^0 п'Г	^4 А	Лц
A = p(V — Vo) =-----отсюда Дт = -5^->.
11	м /
2.47.	Количество теплоты Q работа, совершаемая при изометрическом вдвигании поршня,
4 A = pV~ — RT= £- RT g>.
§ 3.	Графики для тепловых процессов. Закон сохранения энергии в тепловых процессах
2.48.	Участки 1 — 2 и 3— 4 графика (рис. 0.2.48) — изобары, описывают процессы, при которых давление газа не изменяется. Участки 2—3 и 4 — 1 того же графика — изохоры, описывают процессы, при которых объем газа не изменяется, и поэтому давление прямо пропорционально температуре.
2.49.	Из уравнения Клапейрона—Менделеева для одного моля газа pV ~ RT следует, что угол наклона а нзохоры к осн температур таков, что tg а — р/Т= R/V, т. е. максимальный наклон соответствует минимальному объему и наоборот; Kmjn соответствует изохоре OB, Ктах— изохоре OZJ:
4 Гт;п — RTi/pz, Гтах = RT^/pi ►.
2.50.	На графике в р, К-координатах цикл имеет вид прямоугольника площади, равной p0V0. Уравнение газового состояния:
Vq = RT^/po,
180
Работа, совершаемая газом,
Л=-АрАР,
т. е. численно равна площади прямоугольника на графике в р, V-координатах. Таким образом,
A = p3V<) = RTf)
2.51.	Из пропорциональности между р и V на участках 1—2 и 3—4 графика (рис. 0.2.51) следует, что
Pi/рг — Иг/Рг. Рз!Р± = ^s/V4.
Из изотермичности процессов, описываемых участками 4—1 и 2—3, следует, что
p2V2 = p3V3.
По условию V2 = l,Z4- Из уравнений получаем
V1V3
V4V2
vtv3
Так как pV = RT, то на неизотермических участках, где давление пропорционально объему, температура пропорциональна V2,
Рис. 0.2.48
Рис. 0.2.51
Поэтому график процесса имеет в Т, Р-координатах вид, представленный на рис. 0.2.51.
2.52.	По условию задачи Ul = cTll U3 = cT3. Из уравнения газового состояния имеем
Pi^i-R^i, Р?У 2~ RT
Работу А, совершенную газом, выразим через среднее давление:
A=pcv(V2-V1)=^±-^(V2-V1),	=	(1/2 —1^).
Из закона сохранения энергии следует, что
◄ AQ=At/ + A = (V2-V1)^-^ + l)p1+-^-j >.
181
2.53.	Начальное и конечное состояния газа имеют одну и ту же температуру, так как величина pV в состояниях 1 и 2 одинакова, т. е. внутренняя энергия газа не изменилась. Значит, количество теплоты Q, сообщенное газу, равно работе, совершенной газом:
Q = =	)т=Т'о!/
2.54.	Охлаждение происходит на участке 3—причем приращение внутренней энергии
= У. R (Т. - TJ = Q - Л (V3 - IS),
где Q —количество теплоты, отданное газом при охлаждении. Из условия задачи п из уравнения газового состояния следует, что г т рУг рУх т рУ1
T1==-R~'
Окончательно
Q	Vi (Р2 — Р1) |>.
2.55.	По закону сохранения импульса mv — (Л14“т) и- Разность кинетических энергий в начале движения поршня и в конце, когда кслебахшя затухнхт, равна энергии, перешедшей в теплоту;
отсюда
лТ	тг}2
ДГ = 2ДТЙТ^)~>’
2.56.	Запишем закон сохранения энергии в системе отсчета, движущейся вправо со скоростью центра масс 2v:
^ + сТ0 = сТ; отсюда 4 Т’=7’о + ^>.
2.57.	Работа A=nigV0/S-{-p0V0 = cv(T— Т,,), где Уо— начальный объем газа над перегородкой, v—число молей в сосуде, Г —То —изменение температуры. По закону Клапейрона —Менделеева p0'2Va = \RT(). Таким образом,
^7=Г0Г1+^+^1 Г>.
2.58.	Обозначим через Vf, 2 число молей воздуха в первой и второй колбах. Уравнение газового состояния для начального и 182
конечного состояний газа в обеих колбах дает
PlV = \\RTi, p2V = v2RT2 = v2R7\, PiV — ViRT'l, pzV = p'iV = v2RT2.
По закону сохранения энергии
с (vx v.,) 7\ = c (vi Ту -J- v2T2).
Так как количество газа не изменяется, то
Vi4-v2 = vl + v2; отсюда pi + p2 = 2p, 2/Т1== l/Ti + l/Ta.
Окончательно
2.59.	Число молей газа в нижней и верхней частях сосуда v	v = Л, ‘ ИИ'\ hS
После того как убрали перегородку, давление ио всем сосуде стало равно peJr mg/S. Тогда, используя уравнение газового состояния, получаем
(ро+4-') (2/t-x)S = (vx + v2) RT2^(pa+p+^f\hS^-,
р AV = ^ро Sx — с (ту-)-t’s) (Т2— Tj) =
с , „ [	,	, mg \ [ Т2	, \
— -jj-hS ^р. + р-]-™J —
Из этих уравнений получаем
с + R Ро + tng/S
2.60.	Пусть х—изменение длины пружины после того, как поршень отпустили, т. е. х—новое расстояние между поршнем и правым торцом сосуда. Закон сохранения энергии:
zZ2/2 + сТ! = хх2/2+сТ’2.
Уравнение газового состояния:
PiSllTi=P2SxlT2,
Условие равновесия:
p2S=xx,
183
Совместное решение этих уравнений дает r т 2cT\ + ?.R l2==11^T1 + ^Sl
2.61.	Пока поршень неподвижен, уравнение газового состояния имеет вид
pohS = RT().
После начала движения поршня сила натяжения нити иа поршень ие действует, тогда уравнение газового состояния приобретает внд
(Po + mg/S) 2hS = RT.
Из закона сохранения энергии имеем
Q = с (Т— Т’о) + mgh + p„hS.
Совместное решение этих уравнений дает
«4 Q = mgh (1 + 2c/R) + (с + Р) То &.
2.62.	Количество теплоты Q переходит во внутреннюю энергию газа при нагреве его до температуры Пив работу А по расширению газа:
Q = ДП + А = с (Т1' —Т)+р ДК, где ЛК = 2К — V = V.
Из уравнения Клапейрона — Менделеева, записанного для начального в конечного состояний газа, следует, что
(р/2) К = PT, p-2V — RT'; отсюда 7,' = 47’.
Работа А = р ДИ = pV = RT'/2 = 2RT. Приращение внутренней энергии Д77 = с(7’/ — Т) = ЗсТ. Окончательно «4 Q = (3c + 2P)7’
2.63.	pS = NA-piS, aSpp= NR-p±aS, где N — сила реакции стержня. Из этих условий равновесия находим значение давления во втором отсеке: р2 = р(а|3—1)/(а—1). Объемы отсеков
Ki=yS, V2=^-S + — aS, Ps=^-aS.
Так как температура во всех отсеках одинакова, поршни после нагрева останутся на прежних местах. Тогда, вводя полное число молей газа v, из закона Клапейрона — Менделеева имеем
PiVi + Р21/24-РзКз = 'у7?7’1,	р1==р, p3 = fjp;
отсюда
c,a2P —1 пп, pSl——- = vRTi.
СО |
184
Аналогично для нового состояния газа имеем f , , , ’ , _____________________________________________1
P1V1+P2V2+ PsVo — vRT2, pi^p', рз = $р’, p-2 = p'	;
отсюда
I (-1 f ОС В 1	p, m
p SI—!—Г =vT?r2. a— 1
Из закона сохранения энергии Q + vcTt = xcT2 находим v. В итоге искомое изменение давления
. .	, QR a—1 ..
2.64.	По закону сохранения энергии m Mv2 Mv2 r.
Q = — chT-[—— , где —2~• = Fl = pSl.
Здесь учтено, что при постоянном ускорении поршня давление р также постоянно. Уравнение газового состояния для постоянною давления дает
pkV = -~R&T.
I»
Таким образом,
Q=— cbT-\-pbV= — сКТ + ~ RKT = — (с + ^)ДТ; |Л	|Л	|Л
, отсюда
дг=-_	(3 = _0£_ .
(c^-R)m’ 4 c-f-R г 2 •
Окончательно
Mv2 [. . с 'i
2.65.	Пока поршень покоится, вся теплота идет на нагрев газа: 7’=Q/c + Tl) при Т<Ткр
Найдем, используя условие равновесия и закон Шарля, критическую температуру Тк?, при превышении которой поршень начнет двигаться:
(Ркр~~Ро) $ = Рк$/ТКр= Po/Tq', отсюда
Т'кр —	^о> Qkp = c^'o//Po‘5.
185
Запишем закон сохранения энергии (первое начало термодинамики)
Q-Qkp + ^ = c(T-7’kP) + Pkp(V,-Vo),
о а7р Ъ
где
Рис. 0.2.65
-?=ls{V~Vo) = ~\ Р°
Таким образом,
Q -QкР- с (Т-Ткр) +	(’/- V»),
Так как pKVV RT, рк?уа = ЯЛср, то
РкрН“Ро	. , .	\ ...
—2— (V — I о) = ^-\1 -г ( ~~ кР)-
Окончательно
<; Q — QKp = (Т— Т'кр) +~2~ 1 т ) ] ПРП > ^кр’
r = TKp + (Q-QKp).jc+4f 1 + т^)Ь
I L z\ 1 кр / J
График зависимости Т от Q оказался ломаной линией (р ис. 0.2.65) состоящей из двух отрезков прямой. Точка излома
Qkp = c7’°^s’ Гкр=(1+рУ 7’°^-
Глава III. ЭЛЕКТРИЧЕСТВО И МАГНЕТИЗМ
§ 1. Напряженность, потенциал и энергия электростатического поля
3.1. Второй закон Ньютона для электрона, движущегося по окружности радиуса г с постоянной угловой скоростью w, можно записать в виде
mw2r = еЕ^
где Г —напряженность электрического поля, возникающего в проводнике на расстоянии г от оси цилиндра в результате смещения электронов, Окончательно
Е = rrtto2r/e
186
3.2. Для заряженного шара, создающего сферически симметричное электрическое поле напряженности Е у поверхности шара, ймеем
Е = —; отсюда Q =4ле0/?зЕ зз 5,5-109 Кл |>. 4ле0/?3
3.3.
• 4Z2 cos2 (а/2)
?i-4Z2 sin2 (а/2)
◄ tg (а/2) = 'j "
\ ?i 1
3.4.	Заметим, что силы упругости кольца компенсируют нормальные составляющие сил электростатического взаимодействия.
1)	?i = ?2= ± ^3 К 3-4neomgZ?2 или
<71 ~-—</2, ?i?2 = — з у 3-4ne0,<Kg/?2 £>.
2)	Q = </ —3 К 3-4nea/z;g7?2/<7 или
<2=? + 3/ 3-4ле0я^2/9
3)	?з = ?4=1/"2-8лг,)т^/?2/(71 —<?2)
4)	q1 = — q2 = q — 8^e0mgR2/q или
?i = ?2 = ? + 8nE„m^2/(? g>.
3.5.	£ = i. «^[„v+rayr11, tsc-j/a-p..
[	2'11 / J	^QtTlg
3.6.	Введем поверхностную плотность заряда на внешней и внутренней поверхностях плит (внутри проводящих плит поля нет): щ, От и ог, 02. Закон сохранения заряда дает
01=01 + 01,	02 = 02-4-02.
Из-за отсутствия поля внутри металла имеем /	Г	п	Н
01 = СТ£, 01=—02>
отсюда
О1'=О2=(О>4-а2)/2,	(Т1 = —02 = (01 —о2)/2 >.
3.7.	Прочность прямо пропорциональна площади сечения проволоки, т. е. квадрату ее диаметра. Сила натяжения, обусловленная кулоновским взаимодействием, обратно пропорциональна квадрату диаметра кольца и прямо пропорциональна квадрату заряда (это видно нз закона Кулона, в частности по соотношению размерностей). Таким образом, увеличив диаметр кольца и проволоки втрое н оставив его заряд без измеиеиия, уменьшим тем самым в 9 раз силу натяжения н в 9 раз увеличим прочность. Поэтому
187
разрыв нового кольца произойдет при увеличении заряда в 9 раз (Di, а—диаметр кольца, dlt 2—диаметр проволоки):
41/Di   di	Q2 —d^Dz n
ql/Dl ~ dl' <7i "rfiOi “
3.8.	Из принципа суперпозиции получаем	q> = 2q>o,
E = V"2Ea
3.9.	Из соображений симметрии следует, что каждая из двуя палочек в точке О создает одинаковый потенциал q> и одинаковую
по модулю напряженность Е, направленную перпендикулярно палочке (рис. 0.3.9а). Используя принцип суперпозиции, получаем 2ф = (р0> т. е. ф = ф0/2
Поля надо складывать векторио (рис. 0.3.96). Получаем ромб, у которого стороны равны £, а короткая диагональ равна Го-Треугольник со сторонами Е, Е, Еа— равносторонний, т. е.
!f]=-Z^o направление вектора Е перпендикулярно оставшейся палочке. (Возможно красивое решение с добавлением третьей палочки, создающей полный правильный треугольник с Е — 0 в центре.)
3.10.	После соединения шаров 1 и 2 заряды на них одинаковы и равны е//2. После соединения шаров 2 и 3 заряды на них одинаковы и равны 7/4. Если пренебречь влиянием заряда шара 2 на заряд шаров 1 и 3, то после соединения шаров 1 и 3 заряды иа них
1 ! q , <? \ Зф ?!—?з- 2 2 + 4_J~ 8
и искомое отношение таково:
<71 :<?2:<7з = 3:2:3 &.
Если учесть влияние заряда шара 2 на заряд шаров 1 и 3, то
ft—д<7 i <72<7з + д<7 , <72 Л_ <7 Г) J- г (1	1 М
188
и искомое отношение
.	Г3 г /.	1 \ 1 1 Г 3 . г (1	1 \1
◄ ?1:?2:?з= [у--(J - у^) ] : T.i|^+T (J ~yj) ]
3.11.	Потенциал заземленной сферы равен нулю, в частности ои равен нулю в центре сферы, там он равен сумме потенциалов полей зарядов q, — q и индуцированного заряда Q сферы:
Q , q q п	Л / 1	1 \
—- + 4-—-т- = 0; отсюда Q^=qr[^-------у- .
Г ll l2	\ *2	*1 /
Заряды 1 и 2 действуют на сферу с силами
/ 1____Ms	Q"r < х___1\
4ne0Zi 4ле0/1 \ h /	4ле0/2 4ле0/г h)
Суммарная сила, действующая на сферу,
=(^+^)1/2=-^-2 (4-45 1^'2’ 4ne0lil2 \ з '1 )
> Л| $ -tga = — =-------- t>.
f2 Я
3.12.	После первого заземления второго шара и его последующей изоляции, исходя из равенства нулю потенциала в центре заземленного шара
Ф1 4* <₽2 =	---4 ~~—<= О,
1112 4лг,|Г ' 4пе0/
имеем
4+4=»- ^—т
откуда qxlq^—iir.
После повторения всей шаров имеем
процедуры заземления и изоляции
^+^=о
г Р ’
<71 =
СГ3 Й2 рт3
П3 ’ ~7 F
 Ш
откуда qi'q':> = — l)r.
Таким образом, искомое соотношение qi/q2=—l/r Результат может быть получен сразу при рассмотрении последнего заземления второго шара: q'2lr + qill = 0.
3.13.	Электрическое поле в данном случае сферически симметрично. Внутри заземленной сферы поле равно нулю, значит, потенциал в этой области (В частности, в центре) одинаков й такой же, как и на поверхности внутренней сферы, т. е. равен нулю. Учтем,
189
что этот потенциал создается зарядами на внешней и внутренней сферах. Потенциалы складываются алгебраически, поэтому, разбивая заряды, находящиеся на поверхности сфер, на- малые доли так, чтобы можно было для‘каждой доли пользоваться формулой потенциала точечного заряда, имеем для центра сфер
Д<71, I ! &Qi । ,AQzv_n R .“г г	г
Л<714- ... + kqn = q, AQi ~ЬД@.№ Q,
т. e. q/R-1rQ/r = O, Q——qr/R, где Q—заряд на внутренней сфере.
Напряженность поля на внешней сфере создается обоими зарядами q и Q так, как если бы они были точечными и были помещены в центр системы. Это можно увидеть, например, с помощью картины линий напряженности электрического поля. Таким образом, на расстоянии I от центра
р____Q _____ Q (I________\
4ne„Z- 4ле0/2 / R ]
3.14.	Второй закон Ньютона дает
nw2_ е" / _ j_______2-1 \ _ е2 3 /~2 —4 .
г —4ле„г2(<	4	2/2/ 4ле0г2 4/2 ’
отсюда
.	( е2 3/2 — 4\1/2
4леог 4 / 2m J
3.15.	Полная кинетическая энергия системы равиа сумме кинетических энергий обеих частиц: WK — 2WK. Второй закон Ньютона дает
mv2__ е2
г 4ле0-4г2 ’
где 2г —расстояние между электроном н позитроном. Тогда кинетическая и потенциальная энергия системы
2ту2 __ е2	е2
*	2	4ле0-8г ’ п 4ле0-2г ‘
Таким образом, l₽,n/W,K = —2	Тонкость и трудность задачи
в том, что источник электростатического взаимодействия здесь не расположен в центре вращения.
3.16.	Введем расстояние между частицами г н заряд каждой из них q и —q. Частицы будут двигаться вокруг общего центра масс по окружностям с радиусами
"	„	«2	mt
tl = r---:--j Г^ = Г----------
пц	m>4- mi
190
и скоростями н v2 такими, что
m1v1 = m2v2.
На основании второго закона Ньютона
_ mi (т2 + mt) 01
4ne0r2 Г1	т2г
; отсюда
?2 _ fflj (т2 + ffli) vj
4ле„г	m2
Пусть скорость одной 'частицы увеличили в п раз. Тогда на основании закона сохранения импульса обе частицы будут продолжать двигаться даже при наименьшем п с некоторой скоростью и после удаления друг от друга на очень большое расстояние (их относительная скорость в этом случае равна нулю).
Закон сохранения импульса дает
mt (п — 1) v’i = (mz-Jf-mi) и.
По закону сохранения энергии имеем
q2 min^vi ,	__(тг-\-т2)
4ле0г +	2	2	2
Подставляя в это уравнение v2, г и и из предыдущих уравнений, получаем квадрагное уравнение для п:
2т\
—- п — т2
откуда
..	f t/"a f 1 ।	\ il
4 П~7п~ И 2 1+^“ —1
I \	/ J
Ошибочным было бы рассмотрение движения частиц без учета движения центра масс.
3.17.	Выражая кинетическую и потенциальную энергию атома через заряд электрона и диаметр атома, вычисляем полную энергию атома W. Работа по ионизации атома равна полной энергии W, взятой с противоположным знаком:
те- _mv'1	_ е2 ти- 2ег
к 2	4ле0-2г 4ле0й ’ п 4neorf8
л2	«2
Г=ГК + 1ГП = ---^—r,	А=-р—г=14 эВ >.
1	4ле0</	4ле0</	3
3.18.	По закону сохранения импульса mv sin a=const и энергии
е2 . 2mv2 cos2 a . 2mv2 sin2 a  e2 2mv2sln2a
4ne0Z '	2	'	2	~ 4ne0rmin *	5
191
получаем
•4 rm\a = l p + ^S-Z/zw2 cos2aj >.
3.19.	Имеем	L = ЛС = /[(1 -J—sin a)2-|~(l — cos a)2]1^2 = I V"$'
Второй вакои Ньютона в нижней точке дает
mv2
-r = T-mg.
Из закона сохранения энергии следует, что my2 . .	q'1 q2
—— = tngl (1 — cos a) 4———r —	у,
2 s '	' 1 4ne0Z 4ле0£
Таким образом,
T=mg(3 — /"2)4-2 f 1------LA лд2 „ >.
\	/ 3 / 4jie0Z2
3.20.	В верхней точке на шарик действуют по вертикали сила
СТ2 тяжести mg и проекция кулоновской силы -------- г— . Чтобы
4лео’5|4 5Z2
было движение по окружности, необходимо выполнение условия mv2 ,	g2
—г- 52- mS 4----s - -.
1	4ле0-5/ 5Z2
Учитывая закон сохранения энергии mva mv2 — = —+ 2m^>
получаем /	0^	\l/2
< »о ( 5gZ+ ------А _ - )
\	4ле0«5 у 51т/.
3.21.	Из закона сохранения энергии и условия потенциальности электростатического поля можно написать
2mvo . л I г. 2mv2
~2~ + 2с<ТЕ=-2-'
Из второго закона Ньютона имеем 2mv2 _ q2 I —	4ле012'
В итоге получаем
I 1 ’	4ле0/?
192
3.22.	Сплошное равномерно заряженное кольцо в однородном поле вращаться не будет. Заменим мысленно кольцо с зазором на сплошное, где на месте зазора добавим еще и отрицательный заряд —q' той же линейной плотности, что и положительный на сплошном кольце:
, qi
"q ~ 2nR ’
где R — радиус кольца. Скорость будет максимальна в положении равновесия, когда заряд —q' (или зазор, что эквивалентно) пройдет дугу (л/2) R.
Так как поле однородное, изменение потенциальной энергии при этом
-А<£П0Т = ?ЕЯ=^.
Закон сохранения энергии дает
zV/ti3 qlE	.	-./W
—ь— =	отсюда «я V— I/ -Н-г
2 2п	¥ лМ
3.23.	Шарики будут подниматься по спицам лишь при условии, что кулоновское отталкивание преобладает над силами тяжести, т. е. при
9'2	.
—ь—- > mg tg а.
4ле0Е2 s °
Если шарики остановятся, не достигнув концов спиц, то из закона сохранения энергии
д2 /_1_________1
4ле0 \ L	L-\-2h сtg а
находим максимальную высоту Л:
/1=='й—~—?----Р"
Sne.gngL 2
Этот результат справедлив при I sin а, т. е. при
— «£ mgL tg а (L-\-2l cos а).
4л £q
В противном случае шарики соскользнут со спиц. Из закона сохранения энергии находим скорость шарика на конце спицы:
„ mv2 q2 I1	1	\ „	, .
2—r-=T-t— -у- — , , о.----- — 2mgl sin а.
2 4ле0 \ L L 21 cos а / 1
= 2ingh
1 Г. В. Меледии
193
Высота подъема определяется только вертикальной составляющей скорости шарика на конце спицы:
h = I sin а
, u2 sin2 а
V~2g~
Таким образом,
при <?2 > 4ле0 mgl tg a (L + 2/ cos а)
,	, .	„	о2/ cos а sin2 а
h = I sin а cos2 а 4- -------—, ------------г ».
1 4ле0 mgL (L -{-2/ cos а)
3.24.	Средний шарик будет покоиться, а скорость крайних находим из закона сохранения энергии:
2mv2 q2 , 2<у2	Г 5
“^^2Z+4^/; °ТСВДа * V ЪщЪт
3.25.	Из закона сохранения энергии имеем
q2 . z/2 q2 , z4/2	q2
-Н—г4—тг——•—т-,4-~к— ; отсюда	7. = -,—	►.
4ле0/	2	4ле0.4/' 2	4ле0-2/3₽^
3.26.	Скорость максимальна при равенстве сил трения и электростатического взаимодействия:
Закон сохранения энергии дает
mv2 q2 ( 1	1 \	.
2—-^\T—^rkms(x~^
Окончательно
•* "р“ 5 *"8 ►
(в противном случае тела не сдвинутся с места).
3.27.	По закону сохранения энергии находим
4ле0/0 4ле0/ 1 s
отсюда однозначно (второй корень уравнения не годится)
1=	_^2 прн 3^5
4ne.omgklo г 4ле0/2
(если это условие не выполняется, / = /о, так как тело в движение не придет).
3.28.	Скорости окажутся максимальными в момент t, когда все три заряда окажутся на одной прямой. При этом электрические 194
силы будут скомпенсированы силами натяжения инти. Скорости частиц будут перпендикулярны нитям. Обозначим через Vi скорость среднего заряда, а через п2—скорости крайних зарядов. Из соображений симметрии и с учетом законов сохранения импульса н энергии имеем
о mvi
mvi_ = 2mv2, —
<72 4ne0Z ’
^нач
2mvl _дц7	дц7 _п? _________в?
-----— tiiVajj, aw эл—w нач	ivkoh, 2
7	1 / 2^ ,	\	5	.
кон 4ле0 \ I 21 J 2 4ne0Z ’
отсюда
д
911 4ле0-21
Таким образом,
V1 (4ле0-6/nZ)1-2 ’ °'2 (4ne0.6mZ)1<2
3.29.	Отталкивание сменяется притяжением на некотором расстоянии х от точки В, когда сила, действующая на движущуюся частицу, обращается в нуль, т. е. 9^2/(х-{-Z)2 = </а/х3; отсюда однозначно Xi = Z/2 (х2 < 0, т. е. после прохождения точки В, и поэтому отброшено). Чтобы дойти до точки В, нужна энергия
mv2 	9q2	q2   4q2
2	4ne0(x1-|-Z)	4ne0X!	4ne0Z’
откуда v = yr8q2/4ne.0ml .
3.30.	Частица должна дойти до такой точки, где силы, действующие на заряд q3 со стороны зарядов qt н q2, 'равны; тогда в дальнейшем притягивающее действие заряда qr окажется сильнее.
Узка! _	-г /' кг I Л 1	..1 Л
(Z-i-r)3 г2 • г l.\V kd /’ r + Z /к. ГЫ/
Из закона сохранения энергии имеем
(| <71 1 + 1-721-2 И <71 I I <72 I ) = -Ц^ (/I <721 - Г| <71 I )2; отсюда
◄ п= [	КО2 |1/2 при I qt [ > Ы,
а = 0	при |<7s|<|?i| >•.
3.31.	Нижний шарик подпрыгнет при условии mg = q2/4ne,l>l2, где Z—наименьшее расстояние между шариками при отпускании 7*	195
верхнего шарика. Закон сохранения энергии дает
, q2 , 9а . x(/i —Z)2 mgh—, v = mg I-г-2—г Н----—sj
s 4ле0Л s 4ле0/ 1	2
т. е. с учетом условия 92/4ле0/2 = mg получаем
р_	,,h> *h-2mg = .
z/i-j-2mg >th-\-2mg	'
отсюда
. _ х/г2 2mgh
— xh-\-2mg v.h-\-2mg‘
Условию минимальности заряда удовлетворяет знак «—»:
,	, х/г—2mg	.	,г-.-----, vh — 2mg
г = /г , , п 5 , откуда < <7min= V 4ne0mgh , , _
xh-j-2mg J '	s nh-\-2mgr
3.32.	Для падения на землю шарику достаточно достичь с нулевой скоростью точки, где его ускорение равно нулю (силы уравновешены). Равенство сил дает
92
—-т-2—s’ + mS = 0.
4ле0г2 ' ®
Из закона сохранения энергии следует, что
.	.	92 , mv2 q2	q
mg (г—Г)—-г1——к~ = —г2—>	г =   1	;
4ле0/	2 4ле0г	4nsomg
отсюда
” „min _ кад (775=-')	™ ►
(в противном случае omin = 0).
3.33.	Запишем равенство отношений величины F Ы к изменению импульса шарика при его упругих ударах о внутреннюю н внешнюю сферы. Время соударения различных шариков Д? будем считать одинаковым. Тогда
р-4лг2 р'-4л7?2
2mv 2mv'
Из закона сохранения энергии имеем
mv'2_	, У1У2 /1	1 \
2 ~ к+ 4ле0 \ г R )•
Таким образом,
n'-Jj-'V Г1 , R-r li/а
[ + 4ле0 WKrR J •
196
3.34.	Энергия цепочки
W 1 р2 । 2<?2 , 3?а	। 134»
1 4ле0 \ 6г	4г 2r J 4ле0 6г *
Энергия тетраэдра
Г2= ’
4ле0 г
Искомая работа
1	Ч/72
Л = -!—^- = 0,07 Дж
4ле0 6г
3.35.	Из закона сохранения энергии имеем
mvj то2 7^2
2	2 4ле0г ’
так как потенциал в центре шара равен <71/4леог. Окончательно
-< v = ( Уо4
2?i4a Х1/2 4ne0mr J
4 м/с ► .
3.36.	Чтобы пролететь сквозь кольцо, частице достаточно достичь его центра со скоростью, равной скорости кольца, где потенциал равен <72/4леог. Законы сохранения импульса и энергии дают
rWo-fa+m,) v, -у—--------2----+4^7’
отсюда

Г 2дгд2	11/2
[ 4ле0г	tn1mi J
►
Решение можно получить иначе, если сразу перейти в систему центра масс.
3.37.	Из законов сохранения энергии и импульса имеем
ту1  т-тУг . т2у2 , giq2
2 ~ 2	2	' 4ле0г ’
m2v =	-j- m2v2,
отсюда
t =
2г
У2 — У\
Поскольку поле внутри шара отсутствует, то заряд движется по инерции с относительной скоростью а2— Уу. Таким образом,
<=2r[n2-f 1+ — \ 29192 У1/2 тгП2 > 9192
“ L \ 'miJ 4ne0rm2 J ’	mx-|-m2 2 4леог
3.38.	Пусть li—длины пролетных трубок, V;—соответствующие скорости частиц в трубках, « = 1, 2.....п. Электрическое поле
197
внутри металлической трубки равно нулю, и частица летит по инерции:
2	2	2	2
^=eU,2™=eU+^=2eU, ^=ieU.
Условие равенства времен пролета частицей трубок дает
— = —; отсюда Ц = 1 — =1 i, i = l, 2, ... , п
Vi Vi	Щ
3.39.	Чтобы пролететь сквозь правую сферу, частице достаточно достигнуть левого отверстия на этой сфере. Потенциальная энергия заряда в центре системы равна д^/^яе^г, а в левом отверстии— (4/3) 9172/4л£ог- Таким образом, условие пролета из энергетических соображений может быть записано в виде
2 '4л8вг 3 4леог' Таким образом,
◄ 4nin= ]/4 ^тт
3.40.	Между сферами частица остановиться не может: ближняя сфера тянет ее к себе сильнее. Рассмотрим случай остановки частицы внутри одной из сфер. По закону сохранения энергии имеем
___________91<?2	<71<?2
I г x-j-l ’
где qi—заряд сферы, q2 — заряд частицы. Таким образом, х = 0,Зг. Для случая остановки частицы за сферой (т. е. когда х > 1,2г-{-г) получаем
__2<7х?2__	<71<7а	919а
I X—I х-\-1’
откуда х — 2,1г, но х должен быть не меньше 2,2г. Следовательно, этот вариант не проходит и окончательно	х = 0,Зг
3.41.	Максимальная скорость шара достигается в положении равновесия. Из законов сохранения импульса н энергии имеем
ср .	д’-___ mv2 . 2тиг_____д'1
4ле0/*' 4ле0-2/	2	2	4ле0- 1,5/’
2ти + mv = 0,
где v и и—скорости незакрепленного н закрепленных шаров в момент, когда незакрепленный шар находится посередине спицы.
198
Таким образом,
2	<?а
9 4яе0/т
3.42.	Закон сохранения импульса:
ти = 2mv,
Условие разрыва нити:
Закон сохранения энергии: ти2 д2	д2
2	4яеох	4яе0- Юг'
Окончательно
. _ Г 9'2 / Ут 1
v L 2/ге \ д' Юг
/ д'2 \i/2
V — I —=- ) \m-5r )
где д'—'д1УАпгп.
1/а у у	1
при -7-<7Г-:
г д	2г
УТ 1 .
при —>27^
§ 2. Электрическая емкость. Конденсаторы
3.43.	Так как пластины замкнуты, то они имеют один и тот же потенциал, т. е. поле внутри конденсатора равно нулю. Это значит/ что на  пластинах появились заряды, создавшие поле, напряженность которого равна по модулю и противоположна по направлению напряженности внешнего поля. Зная напряженность поля, вычислим заряд:
9=t/C = e0ES=O,9.10-i<’ Кл.
Заряд на одной пластине равен д, на другой —?
3.44.	Проще всего удовлетворить условию задачи, пронеся одну пластину мимо -другой по кратчайшему пути и расположив их на расстоянии d одну от другой (могут быть и другие перемещения). При этом работа сил взаимодействия зарядов с полем неподвижной пластины по ту и другую сторону от нее компенсируется. Остается лишь вклад внешнего поля: Л =— gE-2d Можно найтн работу и через изменение энергии системы:
Л = 1Увон	Wяач = С (Е Еконд)2 d2/2 С (Е“ЬЕКОНД)2 ^2/2,
•< А = — 2СЕЕ коял (Р = — 2д Ed >.
199
3.45.	В электрическом поле на плоскостях пластины наводятся заряды q н —q такие, чтобы внутри пластины поле было равно нулю. Поэтому q—UC=EdC. При выключении поля происходит постепенная разрядка возникшего во внешнем поле заряженного коидеисатора, при этом вся энергия конденсатора переходит в теплоту:
4 Q = 4ne0£2 Sd/8n = e0£2 Sd/2
Закон сохранения энергии позволяет при решении избежать детализации процессов, происходящих при переходе из начального состояния в конечное.
3.46.	При удалении пластины заряды стекают на пластины другого конденсатора. Поэтому
q — CU, 2q = C-2U, 4 t/' = 2t/, W = 4U2C/2 = 2CU2 >.
3.47.	Через время t высота электрона над плоскостью АВ у = = at2/2, где а — его ускорение, а вертикальная составляющая скорости vy = at. Через время Z/o0 — t координата у = 0, т. е.
Однозначно, так как t < //у0, получаем
3.48.	Пусть ускорение и время движения частицы между пластиной А и сеткой равны aj и tlt а между сеткой и высшей точкой траектории а2 и Z2. Потенциалы <р^ = 0, <рд = <р, <рв = 3<р, ускорение —2а1 = 2еф/т?. Тогда для движения частицы из высшей точки траектории имеем а2^/2=//2. Из закона сохранения энергии следует, что
mv20 mv2 cos2 а
-|-2«<p.
Условие равенства нулю вертикальной составляющей скорости в высшей точке дает
п0 sin а—Gj/j —a2i2 = 0.
Искомая длина
4 Т = о0 cos а- 2 (Zi + /2) = 2 (4 — У 2)1 ctg а
3.49.	Ток прекратится, когда потенциал сетки G'i нз-за осевших на ней электронов станет равным потенциалу катода, т. е. нулевым. Так как тока нет, всю систему электродов можно рассматривать как систему плоских конденсаторов. В частности, анод, две поверхности сетки Gi и сетка С2 образуют два конденсатора. Искомый
200
заряд q равен сумме зарядов этих конденсаторов:
9 = 91+9г> гДе 91 =—U1C1, qt = —U2C2.
Следовательно,
3.50.	фл = <^)/2, фв = 2(^/3 или фв = (£’/3 в зависимости от расположения конденсатора емкости 2С. Окончательно
◄ <Рлв = Фл —<Рв= ± S/S= ± 1В >.
3.51.	При сведении схемы к более простой эквивалентной нетрудно заметить, что трижды повторяется один и тот же элемент, который понижает напряжение вдвое. Таким образом, искомая разность потенциалов
3.52.	Пусть число пластин i = 4, тогда C = 3e0S/d (три параллельных конденсатора), аналогично при i = 6 С = 5С0, где Со = = e0S/d, и т. д. Окончательно, когда полное число пластин равно 2п,
С=(2л—1)С0 = (2л—1) e0S/d
3.53.	Потенциал шара и ближайшей к нему пластины одинаков, потенциал заземленной пластины равен нулю. Так как потенциал бесконечно удаленной точки равен нулю, а шар по условию задачи удаленный, фактически шар н конденсатор оказываются соединенными параллельно:
q q0 — q	qn ~
1----= я~' > откуда q = - ,
4лё0г С 1	' 1 + С/4леог
3.54.	Пусть на обкладке конденсатора окажется заряд q, тогда на шаре останется заряд Q—q. На другой обкладке возникнет заряд —q, а на шаре радиуса R — заряд + q (из-за сохранения заряда). Приравнивая разности потенциалов между обкладками и между шарами, получаем
4ле0? Q—q q	„Г, , ( 1 , 4ле0\']-1
—откУДа ◄ ?=Qp + r( -% )—
3.55.	Заряд соединенных друг с другом обкладок конденсаторов равен нулю, их общий потенциал обозначим через Ur. Тогда
?2 = 91 + <7з, С2 (U— и1) = С1и1-\-Сзи1-, отсюда
CCjC], 3	U (Ci+Cg)^
3 С1+ Cj-j-Ca’ С1 + С2+С3
3.56.	Заряд иа конденсаторе емкости Ст в положении ключа 1 qa = CiSi- Когда ключ переводят в положение 2, имеем £71 +
201
qi+qz=—q<>, т. e.
CrUi—C2U2 = — Cj^i; отсюда < t/i
$2^2 — S 1Q Ci+C2
3.57.	Емкость батареи конденсаторов, подключенных к конденсатору 1, равна 2С. Для напряжений на конденсаторах имеем l/4= Ui = 2U2 = 2U3 = 2Us = 2Ue. Окончательно
◄I qi = qi = UC/3, q3 = q3 = qi = q3^qil2=UCI0 >.
3.58.	Пусть конденсатор заряжен и разность потенциалов на нем равна U. Тогда при очередном замыкании ключа в положение 1 через резистор с сопротивлением Ri в начальный момент пойдет ток —	—U)/Ri. За время включения заряд конденса-
тора изменится на ^q—Iyt, где т—время включения. Когда ключ замкнут в положение 2, через резистор с сопротивлением Д2 идет ток /2 = (<^2-'Д)/Д2.
Так как U — установившееся напряжение, то Дт = —/2т, т. е-после двух переключений заряд на конденсаторе остается без изменения:
Sj-SL----отсюда У =
Л2	Ал 1 Л2
Ri
Искомый заряд
S2R1A- SiRz Rl +/?2

q = CU=C
3.59.	При зарядке конденсатора
Ио — IiRi~V Uс — /1Я1 + /2Я2» А? = (Л — 1%)
при его разрядке
Л<7 = -^(Т-т), (Л-/2)т—^(Т-т) = 0, Л2	^2
откуда Т	/	Т
Uc=l3Ri, Л = ^2“> U3—I3\Ri ——рЛ2
Окончательно
4 t/c=t/0/?2/?ir+7?2T
3.60.	По закону сохранения заряда имеем
91— 92 = 91 — <?2> Яг — <]з = Яг — qs-
Если провести вдоль цепи единичный заряд, вернувшись в первоначальную точку, то работа по перемещению этого заряда будет
202
равна нулю. Это дает tz V» С/ ◄ <7i = (l/3) (2</1—q2—q2), <?2 = (l/3) (2<?2—<7a—<7i)( q'i = 0 /3) (2<?3—qt — q2)
3.61.	В первом и втором случаях (рис. 0.3.61) конденсаторы оказываются соединенными так, что для разностей потенциалов на
конденсаторах получаем следующие соотношения:
112 _ ?02 — 112 '/is _ </аз—?13 Ci С2 ’01 Сз
где ^02 и qos — первоначальные заряды на конденсаторах 2 н 3. Для третьего случая из закона сохранения заряда следует, что
?2+?1=<7о2, —'Уз—?i=—'/оз-
На конденсаторе 1 после соединения с конденсаторами 2 и 3 разность потенциалов оказывается равной суммарной разности потенциалов на заряженных конденсаторах 2 и 3:
?2 । </з
С, С2 Сз
Заметим, что у соединенных между собой конденсаторов 2 и 3 суммарный заряд q3—?2 = ?оз—</о2 на внутренних обиладках отличен от нуля; отсюда
_?12 (Ci/Cg-j- 1)-|- q13 (Ci/C3-|-1) 14-Ci/Cs + Ci/Cj F’-
3.62.	Пусть расстояние между поршнями до и после зарядки равно /0 и I. Напряженность электрического поля, создаваемого одним поршнем, равна ^/2e0S, поэтому сила, действующая на заряд каждого поршня, равна <?2/2e0S. Условие равенства сил, действующих на поршни после их зарядки, дает
Ро$ -)-q2/2e9S = pS.
203
По закону Бойля — Мариотта имеем p0S/0 = pSl. Из этих уравнений находим
— 1 I q Ъ. I -1ф2еоро5’-
3.63.	Пусть установившиеся заряды на пластинах <уд, pg, qc-Поле, создаваемое одной пластиной, имеет напряженность Е = = Q/28oS. Разность потенциалов между В и С равна разности потенциалов между В н A: Ubc—UbA~U- С учетом суперпозиции
полей находим
и < ct
Рис. 0.3.63
Uвс— U — 2^	+<7в~?с)>
Uba —	(— 7л+ ?в + <7с)-
ZCQcJ
Закон сохранения заряда: <7д+<7в + <7с = 7- В итоге

Сведение задачи к схеме, изображенной на рис. 0.3.63, неверно, так как ведет к нарушению закона сохранения заряда: пропадает поле вне пластин.
А
4
Рис. 0.3.64
3.64.	Закон сохранения заряда: Qi— Q2 = Qi — Qa (рис. 0.3.64a). С учетом суперпозиции полей от пластин
• _ Qi р ___0.1 — 01 р _ Qi — Oi-УОг
1-80S’	2~	80S	’	3“	80S
Из-за замыкания проводником имеем
^14 = (£1 + £2 + £з) d = o, откуда Qi' = (2Qi—Q2)/3.
204
। Следовательно,
◄ £/23 = £2d = -
Oc-qO
Другое решение (рис. 0.3.646):
Qi=Qi + Q'>	Qa+ Q',	(1)
<714 = 0, t. e. Д(712 + Д(72з + д^з4 = 0- иначе
Qi/C-Q'/C+Q'2/C = 0.	(2)
Складывая равенства (1), имеем QiQa = Qi + Qa-f-SQ'; отсюда с учетом (2) получаем Qi4-Q2 = 3Q'. Окончательно
3.65.	Закон сохранения заряда: 1CU — Qi-4-Q2. Из равенства разностей потенциалов на обкладках конденсаторов имеем Q^/ZC = — Q2/C, т. е. Qi = 2Q2 = 4C(7/3. Наконец, закон сохранения энергии дает
о	О С(72	/ Q? , Q1 CU2
И'кон-^-з \2^с+'2С
В результате получаем
<4 v=VCU2/3M
3.66.	Заряд q0 переносится с металлической пластинкой с одной на другую пластину конденсатора: q0 = CU — e.0SU/(di — d2). Работа источника дйи идет на разгон пластины:
,, mv2 e„SU2	28^$
9о(/=_откуда « v=U у
Заметим, что сила F, действующая на отошедшую на расстояние х
2 /	о \
qa / , , 2х \	л
металлическую пластинку, равна	----т ), а ее работа
28()О \ uj — U2 /
4 =^(rfl_d2) = 9o(y.
3.67.	Количество теплоты, гвыделившееся на сопротивлении, равно разности установившихся значений энергии, запасенной в конденсаторах. Найдем эти значения для всех вариантов соединений в цепи (рис. 0.3.67).
а)	После первого размыкания ключа К заряды q = CU сохраняют свое значение. Таким образом, энергия £а (рнс. 0.3.67а) имеет
206
следующее значение:
Е -qi \ ?2 а 2с^2с/2
тгСи\
б)	После замыкаиня ключа К (рис. 0.3.676) заряды на конденсаторах приобретают новые значения: (ц н q2 соответственно. Из закона сохранения заряда имеем 91+92 = 29, нз равенства по-
8	г
Рис. 0.3.67
тенцналов на обкладках имеем q^C^q^iCji). Из этих соотношении получаем, что 91 = 49/3, q2—2q/3. Таким образом, энергия Eq в этом случае имеет вид
£б=4+А=:тс^-
в)	После второго размыкания ключа К (рис. О.3.67в) заряды на • конденсаторах сохранят свои значения: 91=49/3 н 92=29/3. Следовательно, энергия Ев здесь также легко вычисляется:
Е —У* I ______12 cf/2
£в 2С ‘ 2С~ 9 си 
г)	Наконец, в результате последнего замыкаиня ключа К приходим к первоначальной схеме с начальными значениями зарядов, напряжений и емкостей. Соответственно,
Е ——CU2 г~ 2(?-си .
Количество теплоты, выделившееся на сопротивлении после первого замыкания ключа,
Qi=£a-£6=yC£/2 -у С(72 = -1-С£/2,
206
после второго замыкания ключа —
Q2 - Ев - Ет = у С11г— CU*=±- CU*.
Таким образом, полное количество теплоты, выделившееся на сопротивлении, <2= <?1 + <?2= (5/18) CU* = t> Дж
3.68.	При установившемся напряжении конденсаторы имеют общий заряд	с параллельным подключением. Таким обра-
зом, выделившееся количество теплоты
. п _ C.U*________д2 = Са^ + Сз)^
V 2	г^ + Са + Сз)	2 (Ст + Са+G)
3.69.	После замыкания ключа получается эквивалентная схема, изображенная на рис. 0.3.69. Начальная н конечная энергий:
П7 *72 jw	Р ___eoS р e0S Cs_____d^
^нач^2С? ,Гков“2СГ+2С2 ’	h2~~d^’ G__’32'
Закон сохранения заряда: ?i + i72—<?. Из равенства разности потенциалов между пластинами А и В и пластинами С и В следует, что
Qi Q2	Q	Q
cTct’ откуда <7! = ^^,
Из закона сохранения энергии вычисляем выделившееся количество теплоты:
г™-г.„-е, 4 <г=г«.(Х+А)5
§ 3. Цепи постоянного и переменного тока
3.70.	Длина проволоки V/d2 = n[62— (&—d)2] L/d2, где L—длина отрезка трубки, V — объем металла. Сопротивление R — pl/S. Сопротивления проволоки н трубки
_pn[&2-(6-d)2]£	pL
п₽	d« ’ Kyt—n[b2-(b-d)2] ’
207
отсюда
pJ=n2-----------------=я (26/d-l)2 >.
/\тр	о
3.71.	Точки, обладающие равными потенциалами (их легко найти из соображений симметрии), можно соединить. Получающаяся эквивалентная схема проста. При этом RAB = 3R/2 — = 1,5 Ом
3.72.	Если ключ разомкнут, то RaB = 57?/8 = 0,625 Ом Если ключ замкнут, то точки F и D можно соединить нли сторону FD убрать. Тогда 7?^д = 7?/2 = 0,5Ом
3.73.	Соединив точки с равными потенциалами, получаем эквивалентную схему (рис. 0.3.73). По этой схеме находим искомое сопротивление:	7? = 77?/12 = 0,58 Ом ►,
3.74.	Соединив точки с равными потенциалами, получаем эквивалентные схемы (рис. 0.3.74). По схеме (рис. 0.3.746) находим искомое сопротивление: /?=3/?/4 = 0,75Ом
3.75.	Введем сопротивление единицы длины двухпроводной линии р. Тогда с учетом схем, приведенных на рис. 0.3.75, получаем
рх + /?0=«1, рх+/°Р^	=	р(/-х) + 7?0 = Яз.
г<О~ГР(‘—х)
Решая систему уравнений, находим
х-i
Ri + R3-2Ro ’
где R<S = [R3(R1-R2')]lli ►.
208
3.76.	Сила тока I = q/T, где Т = 2nr/v—период вращения; отсюда q — 2nrl/v
3.77.	Электроны движутся по стержню симметрично относительно точки А. Поэтому /д = 0. По мере удаления от точки А сила тока растет по линейному закону. На расстоянии I сила тока л=4т’где L=AB-
По закону Сма Uдв = IcpR/2- Так как зависимость тока от расстояния линейна, то среднее значение тока равно его среднему арифметическому: /ср = (/д + /д)/2 = 1/4. Окончательно 1/дв — = IR/8 ►.
Можно рассуждать иначе. Разобьем стержень АВ на N (причем Д' —> со) частей. В конце первого участка сила тока равна /j, в конце /г-го участка
1п=:п1±, Nli^I/l, откуда Ii = l/2N.
Сопротивление одного участка равно R/2N-, падение напряжения иа п-м участке
R I IR
2N 4Д2 "•
Падение напряжения на всем стержне АВ равно
IR V IR Д'(Д +1)	IR ..
4N2 2-i п~ 4№	2	~► 8 при Д —► оо,
п-1
3.78.	< Ux=U/3= 1 В >.
3.79.	<< ПЛв = /Р/15>..
3.80.	Элемент схемы, состоящий нз двух параллельно соединенных резисторов с сопротивлением 10 Ом и одного резистора с сопротивлением 5 0м, подсоединенного к ним последовательно, повторяется в схеме трижды. Этот элемент понижает напряжение, поданное на вход цепи, вдвое. Следовательно, искомое напряжение < UAB= 160:23 = 20В >.
3.81.	Разность потенциалов между точками В и В':
UbB’ —
разность потенциалов между точками А и А':
3 Rx+R
Знак модуля присутствует потому, что вольтметр указывает абсолютное значение напряжения. Учитывая оба знака разности R — Rx,
209
получаем
◄ Rx = R/2 при Rx < R, RX = 2R при Rx > R
3.82.	Падение напряжения на элементе U — IR2=lRt2—cl’lR2, т. e.
U = IRa2~-cIU, откуда U =	<
Закон Ома дает
U6 = 1R1 + U = IR1+~^T‘, отсюда, решая квадратное уравнение, находим
3.83.	Напряжение на приборе равно (7пр= U^R/lR + г), где R — сопротивление прибора, г—сопротивление проводов; отсюда r = R(Ut)/U—1). В первом случае сопротивление проводов г, = = R (100/90—1) = /?/9, во втором случае сопротивление г3 = = R (100/99— 1) = Р/99; отсюда ri/r2 = ^/9: Р/99= 11, т. е. надо увеличить сечение в И раз. Таким образом, 5Х= 1150= 11 мм2
3.84.	Так как ток /0, потребляемый лампой, по условию не зависит от напряжения, получаем, что падение напряжения на проводах с сопротивлением 2R зависит от номера лампы п, отсчитываемого от последней лампы: Un = 2RJ3n. Для N-к лампы
N
U = U0-2RI0^ п = Uo-2Rla^-N2+ П=0,9t/o;
п — 1
отсюда
◄ р---------Ул____
	10/eJV(W4-l)
3.85.	Показания вольтметра легко рассчитать:
it ________$________Rrv £гу ,, ,
г-4-Rry/(R-) Гу) P4-zy
отсюда	Гу = 7?2/г= 104 Ом
3.86.	Введем мощность Р1у выделяющуюся при параллельном соединении батарей, и Р2 — при последовательном соединении, а также ЭДС батареи Тогда имеем
Р - S2R р 9S2R р .р. '~(R[-r/3)2'	(Я+Зг)2 ’ ^2-4Г1’
отсюда	г= /?/5 = 2 0м
210
3 87 I — *^2—	II — zP _l Zr  +	+
3-87>У-ТТ+Тда- ◄	---------?2+/?—►.
Знак «+» при величине Irt в выражении для падения напряжения U объясняется тем, что ток I направлен противоположно действию ЭДС Si-
3.88,	Так как вольтметр высокоомный, током через него можно пренебречь. В итоге
у __ U oRj____U 0R3 у R1R4 — ^зКз fc.
Ri +Rz Rs-\-Rt 0 (^i + R-i) (Дз+Ri)
3.89.	Из соображений симметрии следует, что /дд = 0, Icd — —1вс=1<№- Количество выделившейся теплоты Q со PR. Запишем соотношения для токов, протекающих через ребра АВ и BE:
Iав'1he~R‘-2R, /дв +/де= Л>/2,	1вЕ=1й1^-
Таким образом,	.
' ◄ Свд:Свс:Ссй:^лв:Свв = 0:9:9:1:4 ►.
Если сопротивление ребра BD сделать равным нулю, а сопротивление ребра CD равным 2R, то соотношения для токов будут иметь вид
Ibc'-Icd — 1R'-R> t вс Л-Icd —I»’ Ibc = ^1o!^> Icd—Io!^ lAB'-IsE—R'-^Rt IAbA~ IBE— Io/2> IaB—Io/^i Ibe=I<№-
Таким образом, в этом случае
◄ Qbd'Qbc'Qcd'Qab'-Q.be— 0:16:8:1:4
3.90.	Потенциал точек А и С одинаков, нх можно соединить. Тогда эквивалентные схемы приобретают простой вид и
3.91.	Сопротивление утечки конденсатора R = pl/S, его емкость C=ee0S//, заряд
9=Ct/ = C(^-/r)=-^.	.
Так как С/? —ееор, то	9=£=^ееор/(/?-]-/•) ►.
3.92.	Заряд, протекший через резистор при вдвигании пластины,
д_,г ГУ!, ( Ы	eol4(£-IR)
Я - (Сг -Со) U	(S—IR) - d; .
q — It = Il/v,
еи
отсюда находим ток / и получаем (мощность на резисторе выделяется в виде теплоты)
I [ eoldv J
3.93.	Uc=Ufb, поэтому <4 9=£/сС={/0С/?2/(/?1 + /?2)
3.94.	Через конденсаторы постоянный ток не идет. Через резисторы с сопротивлениями Ri, R2, Rs идет ток
.	1= Uol(Ri-\-Rz-^-Rs)',
отсюда
a —U С Ri+Rz, ,, r Rz + Rz
◄ 91 “01 +	92“U,>CiR'+Rz + R3
3.95.	Постоянный ток через конденсаторы не идет. Таким образом,	,
- „ r> rr „ C^Ua	C3Uо
◄ ft-Cxl/o, <72=1+^,	=
3.96.	Постоянный ток через конденсатор не течет. Несложный расчет части цепи, состоящей из резисторов, дает значение потенциала левой обкладки конденсатора, равное 4(/0/5, потенциал правой обкладки равен нулю. Таким образом, q = 4U0C/5
3.97.	Через конденсаторы постоянный ток не идет. Потенциалы в точках схемы Л1, N и K:UM = Ua, U^--- Ui,!3, Uk=0. Сумма зарядов трех внутренних пластин конденсаторов равна нулю: ?i + Яг + <7з = 0. Пусть Uс — разность потенциалов на конденсаторе, обкладка которого заземлена. Тогда
C(U0-Uc) = qi, CUc = q2, C(Uc-Ual3) = qi-q2;
отсюда	q2 ——4U0С/9
3.98. 1) По закону Ома
. U qa — It__ q0 R~ RC Ri,C ’
◄ R = Re—£
2) По закону Ома
l/^^^ = /R,^ = /R, «<С = С0-4>-C	Cq	Л
3.99.	В интервал времени (половина периода), когда днод открыт и ток идет через резистор 3, эквивалентное сопротивление цепи равно 3R/2, а ток через этот резистор равен U/3R. Средняя за весь период мощность равна /2R/2, т. е. P = (72/18R
3.100.	Ток половину периода идет через один диод, выделяя на нем среднюю за этот полупернод мощность Pi = U^/Rf. Вторую половину периода ток идет через другой диод, выделяя на нем 212
среднюю за этот полупериод мощность Рг — U2/R2. Таким образом полная средняя мощность, выделяющаяся в цепи,
р _ W ~Т W ,,2 W~t~ W
2	~~и 2RiRi
3.101.	1) Тока через диод нет, пока приложенное к нему напряжение не достигнет 10 В. При более высоком напряжении диод не оказывает влияния на характер тока, протекающего в цепи. Пусть зависимость переменного напряжения от времени имеет вид U (t) = UQ cos (2n//T), где t/o = 22OB. Если учесть ЭДС батареи, то ясно, что диод открыт при
U (0^210 В, т. е. при cos (2л//Т) is 21/22.
В течение времени
т = (Т/л) arccos (21/22)
диод открыт. Искомая доля периода
т/Т= (1/л) arccos (21/22)
Максимальное падение напряжения на резисторе^ £7тах= 10 В Качественный график зависимости от времени падения напряжения на резисторе представлен на рис, 0.3.101.
2)	Зависимость тока через диод от напряжения расшифровывается просто: тока через диод нет, пока приложенное к нему напряжение не достигнет 10 В. При дальнейшем увеличении напряжения диод эквивалентен резистору с сопротивлением 10 Ом. Искомое максимальное падение напряжения на резисторе ^тах= =5 В
3.102.	Зависимость тока от времени имеет вид / = 70 (1 4-а sin at), где а = 0,1. Тогда количество теплоты
Q = R (/2)сР = Rio [ 1 + 2а (sin шОср+«2 (sin2 w/)cp] = Wo (1 + а2/2), так как
(sin ш/)ср = 0, (sin2 й>0ср = (1/2) —(1/2) (cos 2ш/)ср = 1/2.
Таким образом, среднее количество теплоты увеличится на (а2/2)Х Х100%, т. е. на 0,5 %
3.103.	По закону сохранения энергии в шаре в единицу времени выделяется количество теплоты Q*, равное разности энергии попавших на шар nt электронов и тепловой энергии l2R, выделившейся на резисторе с сопротивлением R в единицу времени:
Qt = nttnv2/2— I2R, где 7 = п<е;
213
отсюда
. п mv2 f. 2nte2R\ mv2 _ ,n ◄ Qt = «t—( 1---^2“) n₽H — >e’f = nieiR,
Qt = O nPH ^-<nte2R ►.
3.104.	Если поле в конденсаторе однородно, а вне конденсатора равно нулю, тогда будет изменяться лишь поперечная составляющая импульса у каждого из nt = lje электронов, пролетающих конденсатор в единицу времени:
Л (mv) у = mv., tg а.
Таким образом, на пучок действует сила
F = (//е) mvotga.
G учетом третьего закона Ньютона и закона Гука дополнительное удлинение пружины, вызванное воздействием электронов на пластину конденсатора,

Можно вычислить силу иначе, определив полный заряд, находящийся в любой момент времени внутри конденсатора и взаимодействующий с его полем.
Заметим, что начальное предположение о сохранении продольной составляющей импульса неверно! Действительно, при таком предположении получается, что электрон, пройдя конденсатор, увеличивает свою энергию. Если бы это было так, можно было бы получить сколько угодно энергии без подвода ее извне, заставив электрон, например, с помощью постоянного магнитного поля многократно проходить через конденсатор.
Электростатическое поле конденсатора потенциально, поэтому электрон, описав замкнутую траекторию, должен иметь опять начальную энергию: Я = ^((71 — Ui) = 0! Зцачит, при выходе из конденсатора на большом расстоянии от него энергия электрона не должна измениться. Поэтому сохраняется не продольная составляющая импульса, а полный импульс, что дает в выражении для импульса функции замену tga на sin а. Таким образом, окончательно без учета отклонения конденсатора от вертикали имеем
М ——г,— tnva iin a eMg	r
n ,ЛС , I	eU ,	eUl2
3.105.	/ = —, au——-j, h = —^r—=——=—
Vo v md 2 2m4d .	,	eUl2x .
214
3.106.	Напряженность поля внутри конденсатора Е = {£— lR)/d, где 1—ток, равный заряду, переносимому электронами, попавшими на положительно заряженную пластину в единицу времени. На эту пластину в общем случае попадает лишь часть пучка электронов, ограниченная теми из них, которые, двигаясь по параболической траектории, попадают на самый край пластины. Пусть у — расстояние до положительной пластины при взлете таких электронов, a t — время их полета. Тогда
at2__ еЕ I2
У ~~2~ ~~2пг
Число электронов, приходящих в единицу времени к положительной пластине, / = eyxnvvl). Таким образом,
1 = еухпу00 = (е2хпvl2/2mv0d) (£J — IR);
отсюда
R-\-2mvf)d/e2xnvl2
Если же весь пучок попадает на положительную пластину (тогда y = d), то	I — envdxv0 ►.
3.107.	Рассмотрим небольшой интервал времени А/. За это время к аноду подлетят Ал = IEt/е электронов (е—заряд электрона) и сообщат ему импульс, равный тиЛп. Скорость v электрона у анода можно найти из закона сохранения энергии: mv2j2=eU. Согласно второму закону Ньютона
ЕД/ = mvkn. — mvl А//г;
отсюда с учетом связи I=cU^2 получаем
F — (I/е) mv = cV~2т/е U2.
Таким образом, отношение сял F/E0 = (Z7/(70)2 = 4
§ 4. Электромагнитная индукция. Взаимодействие тока с магнитным полем
3,108.	(£ = —АФ/А/, l = S!R, -4 q = IД/ = BZ2/R >.
3.109.	Изменения магнитного потока и соответственно ЭДС индукции уменьшатся вдвое из-за площади; отсюда следует, что и амплитуда тока станет меньше в два раза.
3.110.	По закону Фарадея — АФ/А/. По закону Ома I = (§lR. Следовательно, искомый заряд
9=/д/==^£=в 1\
отсюда Вг/4р
пг2 — 2лг2/4 .
р-2яг ’
215
3.111.
o_(SB)a q--r-
_ (ла'2В)2 R
3.112.	Запишем работу вихревого электрического поля, вызвавшего изменение кинетической энергии протона:
„ АФ „ mv2 mva е©инд-вд^ — е&~~2	2~-
Ток в пучке протонов
nev
2nR ’
, nev0	Г fl
0=2nR' °™УДа ◄ /= ( Zo
г 2л2тЛ- у '
3.113.	При изменении магнитного поля возникает электрическое поле, раскручивающее кольцо. Магнитный поток
Ф = лВ (/2 + г2);
ЭДС индукции
<£инд = -ду-=л (^2+ Л )~дГ=£‘2лл
Сила, действующая на выделенный на кольце заряд А</, F = \qE.
Тангенциальное (направленное по касательной к траектории) ускорение
at = qE!m = const, v—atbt, \B = B, так как магнитное поле уменьшается до нуля. Окончательно
v = qB (Z2 + -'2)/2mr
3.114.	До удаления перемычки по закону Фарадея имеем
„	А®!	л Л2 Во _ q2 _ АФ2 _ nR2 В„ .
®1'" Cj М ~~ 2 Т ’ ®2 С2 М 2 Т ’ отсюда
л nRS R С п - nR2 R С
— Луу BoCi, q2-----27-0^2.
После удаления перемычки из закона сохранения заряда имеем
,	,	nR2
<71 + ?2 = qi +?2 = я? Во (Cj — С2).
216
Из равенства потенциалов на обкладках конденсаторов ^/Сх = q'i!C2 получаем
nR*B0Ci С1-Са ,	л/?2В0С2 Ci-C2
91 2Т	Ci + C/ 92 2Т Ci+C2^’
3.115.	Используя закон Фарадея, получаем
С^ИНД 1, 2 — Б1&1, 2> где Сэинд 1, 2 = ^1, 2^1, 2 Н* ^0^0-
Так как /0=Л— /2, то
]	^ИНД1^2--<^ИНД2^1 _ _____^1^2-^2^1____
^1^2~Ь RoRi + R0R2 RiR%~}~ RoRi~\~ R0R2
Окончательно^ /0 = 3,6 мА
3.116.	Используя закон Фарадея, получаем
<£ИНд = В^=Во/, гДе l = vt tga, vt0 = l0.
Сопротивление R = lRl = Rlvt tga. Тогда мощность, выделившаяся в цепи,
р=6ш1Д= z B2y3tga. А А/
Окончательно
* o-pt -fe-^ltga-^tga ► 2R ~ 2Rt g 2Rt g “
3.117.	По закону Фарадея | ^инд | = BAS/AZ = W2co/2. Мощность
Fv = Flm = l§-1НЯ IR-, отсюда В —В2/эсо/47?
3.118.	Запишем условие равенства моментов сил, действующих на рамку: (114+m2) gl2 cos a = HJ2 sin a. Окончательно
◄ z =
z/ii - m2
gctga
3.119.	За время т под действием силы F = I0Bl стержень приобретает скорость и = ат=/0В/т/пг. Из закона сохранения энергии имеем mu2/2 = /«g/i (1 — cos <ртах); отсюда получаем максимальный угол отклонения рамки:
. l0Bh	I0Bh .
◄ фтах — 2arosin	—_	при ----------<1
2т у gh	2т у gh.
/аВ1т .	,
пон —2 —  > 1 рамка будет вращаться. Н 2т / gh
217
3.120.	Из второго закона Ньютона в проекциях на горизонтальное и вертикальное направления имеем (рис. 0.3.120)
ml sin a.-4>i = tng tg a ± ^B/sina-o, mg = T cosa
(знак перед вторым слагаемым в правой части определяется направ-
лением вращения); отсюда
Если изменить направление вращения на протц воположное, то
1/ f В<?У । в &q
V \2mJ '/cosa 2m”
3.121.	Второй закон Ньютона с учетом силы действия магнитного поля на ток приводит к уравнению
пи _ Т	/-»Ас?ИНД ^Blkv
ma — tng — Bll, где /==-д^“С—— = С—-^-—CBla.
Таким образом, подставляя выражение тока 1 через ускорение а в первое уравнение, получаем
◄ «=
g
\+СВЧ*/т
3.122.	Так как скорость стержня постоянна, то сила действия магнитного поля на ток через стержень уравновешивается силой тяжести, действующей на него: lBl=mg. Законы Фарадея н Ома дают ,
<£иид = — Blv, 1R = £ — Blv; отсюда 4 и=-^—
3.123.	По закону Ома/ = £7/й, где R — yl/S. Выражения для сил имеют вид F = 1В1, mg=plSg. Окончательно
w t F BU tga=—=—г-mg yplg
3.124.	Пусть высота подъема ртути равна х, а ширина проводящей пластины Н. Тогда давление pgK = FIS=F[lH=lBlllH, так 218
как F — /BI. Ток
U _UHx '~ylltix~ yl '
Если х > /1, то I —UHhiyl. Таким образом,
. hUB UB ,	. UB .
Х =---=— при ---7~й- Х = о при -----7- < 1
pgly pgh	Pgty
При выбранных числовых значениях параметров х= 1,5 м
3.125.	По второму закону Ньютона имеем
ztw2b// = mg+ Т — qveB.
Из условия минимальности скорости vn в нижней точке траектории требуем, чтобы сила натяжения нити в верхней точке Т = 0. Закон сохранения энергии дает
Магнитное поле работы не совершает, так как сила Лоренца направлена перпендикулярно скорости. Знак минус у силы Лоренца выбран в соответствии с условиями задачи. Окончательно
Г_ , , <?2В2/2/,	. 4ffl2₽\lI/2
◄ Пн —g + 2m2 (j У +92B2//]	►•
3.126.	Так как через контур г, R2 переменный магнитный поток не проходит, то ЭДС в нем не индуцируется и сопротивления г и R2 оказываются включенными параллельно; отсюда
.	&N R"r 1 	&R*	ь.
Ri^2r/(^2~l“r) Rz-^r r R (Rtf R1R24*Rir)
3.127.	По закону Фарадея падение напряжения на катушке
Ul=№/M = LM/M.
Так как ток в цепи растет пропорционально временя, Ut со временем не изменяется и в любой момент UL—Ll/t. Следовательно, и напряжение на конденсаторе, равное напряжению на катушке, тоже остается постоянным:
Uc=— —qo~q С С„ С ’
где qQ— начальный заряд на конденсаторе, q—заряд, ушедший с обкладок конденсатора за время Z, С—емкость конденсатора в момент t. Из равенства LIЦ = q0/C0 находим ток:
j__ Qat
LCa 
219
Тогда ушедший с конденсатора заряд
I /2
9 = /ср«=2-/ = ?О2дс-о
Из условия постоянства напряжения на конденсаторе найдем емкость:
С —Со —---------'-о 1 -9,р	₽*
<7о	\	о /
Ответ справедлив при условии, что q < q0, т. е. при условии, что t-r2LC0 < 1.
3.128.	По закону сохранения энергии имеем
г U 0_ . 11 I, /2
CT-L1T+ 2Т”
Приравнивая магнитные потоки, проходящие через катушки индуктивности, получаем L1/1 = L2^2. Решая систему уравнений, на-
ходим
h = U о
L2 С \1/2
Li Lt - j- L2 J
12 = U 0
Lj С	У/2
1'1~\~1^2/
Глава IV. ОПТИКА
§ 1.	Отражение. Зеркала
4-1. ◄ fcmin- 1+L/Z
4.2.	Рассмотрим мнимый источник звука, находящийся в симметричной относительно стены точке А* (аналогично тому, как
проводится в оптике рассмотрение отражения света от плоского зеркала; рис. 0.4.2). Чтобы отраженный сигнал пришел сразу же 220
вслед идущему по прямой АВ, необходимо, чтобы
ci^lAC | + | CS|-Ze = | А*В |-/0 = [(Zi + /2)2+| BD |2]i/2-Zo, | BD p = ll-1 AD |2 = Z2 - (Z2- /1)2;
отсюда 4/ = [( Zo+4/^a)1/2 — Z0J/c
4.3.	При отражении звука угол падения равен углу отражения, т. е. задачу можно рассматривать аналогично оптической задаче на отражение света в плоском зеркале. В момент подачи сигнала изображение источника звука будет расположено симметрично относительно стены по другую сторону от нее на расстоянии I от нее. Вместо отражения звукового сигнала от стены можно рассматривать испускание звука из точки изображения источника. Если Z —время, через которое шофер услышит эхо, то за это время автомобиль пройдет путь vt, а звук — путь ct. Из геометрических построений получаем (рис. 0.4.3)
с2/2 = (2Z+о/ sin а)2+ (vt cos а)2.
Искомое расстояние x = vt, тогда
(с2/и2— 1) х2—4x1 sin а — 4Z2 = 0;
отсюда
◄ х=7~ —r[sina+K(с/0)2 — cos2aj (с/ у)2 — 1 L	г
Рис. 0.4.5
4.4.	Вследствие теплового расширения изменится фокусное расстояние F, равное половине радиуса зеркала. Пусть Ео— фокусное расстояние при О °C; тогда
Zi = Ei — F 0 (1 + at J, Z2— F2 — F0(l+aZ2), Fo = , ’7 1 „7"
1 *7~ Ctii
Таким образом,
,  1 + txZ2
12	1 + aZj
Zi ~ [ 1 + a (Z2 — Z1)] Zj
4.5.	Найдем среди лучей, падающих на зеркало, такой луч ЕА, который после отражения прошел бы через крайнюю точку В зеркала (рис. 0.4.5; О — центр зеркала, Д— точка падения луча). Тогда по закону отражения / ОАВ = £ ОАЕ — а. и / ОВА= £ 0ДВ = а, так как Л АОВ — равнобедренный (ДО и ОВ — радиусы сферы). Поэтому искомый угол расхождения лучей равен удвоенному углу ОСВ и равен 2(90°—а).
Проведем отрезок CD, перпендикулярный АО. Тогда ОС = = r/2cosa (причем АОС=£ ЕА0 = а как накрест лежащие;
221
ADa=D0 = r/2), OC/OB = tg a = 1/2 cos а, откуда sin a = 1/2, a = 30°. Искомый угол равен 2(90°—a)=120° Из чертежа можно увидеть сразу, что / ЕАВ-\-£ ЕВА — За. = ЗЗа\ отсюда а = 30°.
4.6.	По мере того как точка падения луча на зеркало перемещается к краю, угол отражения а и соответственно угол между
отраженными лучами 0 = 2 (л/2—2a) монотонно растут. Из рис. 0.4.6 следует, что
sin атах = 7?/2:£? = 1/2, т. е. атах = л/6,	0щах = я/3 = 60°
4.7.	Пусть в точке S' находится изображение источника света S в плоском зеркале. Пусть OS = d = 3r/4, OS'=f, ОО' = х, где О и О' — точки, в которых расположены вогнутое и плоское зеркала (рис. 0.4.7). Так как SO' = 0'S', то
г	J	1 (i ( Зг\
f—x—x—d, т. е. *=-2 м+т) '
По формуле зеркала
Фокусное расстояние вогнутого зеркала F = rl2, откуда f = 3rl2. Таким образом, х = 9г/8	Проведя решение в общем виде,
получаем
x = d2ii2d~г) при r/2<d<r.
4.8.	1) Изображение Si, создаваемое нижней половиной зеркала, расположено в той же точке, что и источник (r = 2F). Второе изображение находится по формуле линзы:
у+у==-у , f = y, откуда d = 2r, f = 2r/3, т. е. < S'S^ = 2r —2г/3 = 4г/3
2) Дополним половину зеркала до целого н построим изображение источника S, направив на зеркало два луча. Один луч, 222
пройдя через фокус и отразившись от зеркала, пойдет параллельно главной оптической оси; луч, распространявшийся вдоль прежней главной оптической оси, отразившись, пройдет через фокус. Так как источник был на двойном фокусном расстоянии от зеркала, то и изображение будет на расстоянии 2F, т. е. SS1 = 2ft.
Для второго изображения построение аиалогичио. Таким образом, SiS-2~4h
4.9.	Вычисляя долю нейтронов, прошедших последовательно через каждый миллиметр своего пути сквозь пластинку, получаем, что доля нейтронов, прошедших сквозь всю пластинку, равна <4 (0,85)8 х 0,27 (27 %)
4.10.	С учетом двукратного отражения Д = /0 (1—Z?)2, с учетом четырехкратного отражения /2 = Л>(1—R)2 R2. Полная интенсивность света, прошедшего через пластинку,
/ = /„(1-^(1 +/?Ч-*‘+ • • •) = 'о » т.е.<
§ 2.	Преломление
4.11.	Записывая последовательно закон преломления для каж. дой из пластин, в конце концов получаем угол падения, соответствующий полному внутреннему отражению. Тогда
sinanijn = n/feA'-i, т. е. amin = arcsin
4.12.	Рассматривая ход лучей на рис. 0.4.12, запишем
Дг= (Ло + Л) tg a—/г tg ₽—/iotg у, sin a	sin 6	п0	sin а
-г~а=п’ -т—-=— , т. е. —— = п0. sin р	sin у	п	sin у
. .	, Г,„ sin а 1 . , Г.	sin а 1
◄ Дг = й pg«-(Ba_8in^)1/aJ+fto [tg«-^sinaa)va|
4.13.	Доля 6 энергии, прошедшей сквозь шар, пропорциональна отношению площади кольца л(го—х2), через которое идет далее непоглощеиный свет, к площади сечения трубы лго- С учетом закона преломления имеем
x=rosin a = ronsin f} = гйпг/г„ — пг, sin ₽ = r/r0;
отсюда
6=1 —(nr/r0)2 при nr < r0, 6 = 0 при nr^r0 ►.
4.14.	Луч AB (рис. 0,4.14) образует наименьший угол падения («о на внешнюю поверхность. Условие невыхода света: угол Од больше предельного угла полного внутреннего отражения, т. е.
223
sin а0 3*1/л. Из Д ОАВ имеем
sina0 = (7?—d)IR, откуда R'StdnKn—1) = 4мм
4.15.	В соответствии с рис. 0.4.15 и с учетом малости углов а, р, у имеем
Н = R sin у « Ry, h— R sin a ss Ra, p « an, у = л —а —(л—2P) = 2P —a = (2n—l)a;
отсюда отношение радиусов пучков на выходе и на входе H/h «	= 2лг—1 = 1,6
Рис. 0.4.14
4.16.	Поверхность жидкости в струе образует с горизонтально направленным лучом света угол а, для которого имеем
, vuepr / г .	1	г
ctga=-------= 1/ —, sina=—; отсюда <4	>.
''ГОр • Tt	’*	*	*
4.17.	Проведем луч АВ из края круга (рис. 0.4.17) перпендикулярно боковой поверхности конуса. Тогда (так подобрано значе
ние угла a = 30°) после отражения от противоположной стороны конуса луч пойдет вертикально вверх, будет наблюдаться полное внутреннее отражение, так как п = 1,4 соответствует критическому углу падения, равному в условиях задачи 60°.
224
Проведай отрезок 'ВС, перпендикулярный ос. Наблюдатель В результате отражения вместо радиуса 04 будет видеть отрезок ВС. т. е. увидит круг радиуса ВС. £ ОАВ — £ 0ВЛ = а»=3(Л Поэтому ОВ=.ОА = га. Видимый радиус круга
ВС—ОВ sin а = го/2=0,1 м ►.
4.18.	Построения на рис. 0.4.18а стандартны и не требуют яояснений. Следует помнить о малости углов (sinа и tga «я).
По закону преломления staP/slfl«=:l/n, отсюда а «пр. Выражая АВ через £ АОВ и 2 ЛСВ, имеем rP = *(a—Р); отсюда х — гЦп— 1).
Рассмотрим распространение пучка света в обратном направлении (рис. 0.4.186). Проведем дополнительно еще отрезки ОА — г и DE. Из закона преломления sir»q>/sin0=n получаем ф « п0. Из условия преломления на плоскости имеем у/(ф —0)=л. Выразим 0D через £ OCD и отрезок ВЕ=АВ —АЕ, Учтем, что раз углы малы, то OD—x' tg у « х'у. Таким образом,
л'у=х'п (ф0)=Гф—г (ф—0),
Т. с,
г0=я'п(ф—0); отсюда 4g x'sefjh (n—!)=*/«
4.19.	Из соображений сийметрии и закона преломления уста-яавливается ход лучей, показанных на рис. 0.4.19, Для малыя углов
й=гф=х(п<р—ф); отсюда 4g х-гЦп— 1)
В то же время х равно фокусному расстоянию плосковыпуклой тонкой линзы (показатель преломления п,- радиус .кривизны г), отрезанной от шара,
8 Г. В. Меледцд	886
- 4.20. Из соображений симметрии и обратимости лучей света следует, что лучи проходят через центр цилиндра. Из закона преломления при прохождении света из стекла линзы в воздух имеем
Sin Ир___ т 1
sin Р ~ F sin Р ’
где пв—показатель преломления стекла, р—угол падения, а0 — угол преломления в воздухе, г—расстояние от точки падения
луча на границу раздела стекло—воздух до оси цилиндра. Для случая прохождения света из стекла в воду имеем
sin а__п0 ~ 2r 1
sin р п ~ х sin Р
(здесь а—угол преломления в воде, см. рис. 0.4.20); отсюда
sin a/sin а0= l/n = 2F/x, т. е. х = 2Гп
Рис. 0.4.22
4.21. Максимальное расхождение лучей, выходящих из шара вправо, определяется эффектом полного внутреннего отражения света (рис. 0.4.21):
nsina=l, n=2, sina=l/2, a=30°. Тогда
^оЛ+лв=ет+1йов’ <*='(2.+Гз)=О,37м>; 226
4.22. Только половина светового потока выходит вследствие эффекта полного внутреннего отражения нз шара: sina=l/n, где а—критический угол полного внутреннего отражения (рис. 0.4.22). Поэтому за пределы шара выйдут только лучи света, распространяющиеся внутри конических углов ASC и DSB. Поскольку А0= —ОВ—г, 08 = х—искомое расстояние, то
, а	sin а Г
rsina = xsinO, откуда х = г —
Сумма телесных углов ASC и DSB, по условию равная 2л, выражается через угол 0:
2л = 2• 4л sin* (9/2),
откуда получаем 0 = 60°. Окончательно
2 г 4	.
ЗЛ г
§ 3. Линзы
4.23.	x = dF2IFi=:$ см
4.24.	4 / = (/1 + /3 -2f — V(Zi -/2)2 + 4^)/2о = 3,5 с >.
4.25.	Без удлинительного кольца расстояние от линзы до изображения f = bF ЦЬ—-Г) =52,5 мм. Так как расстояние от линзы до пленки должно быть равным /4-/.(/4-/=60 мм), то расстояние до муравья
f р	p^Q 3 м
f+l~Fr bl-Fl+F* '
R
f-(Z/d+I)(Z/f_I) в-
4.27.	Используя формулу линзы, получаем
_L_+1 ’ L~f F
Учитывая, что по условию fa — fa=l,
4F=^=± 47
находим
у, fa = -3F.
4.28.	1) Линза—собирающая, т. е.
_£ f F	f —
l0^d	d—F' 11
Окончательно Z=|/2—fa | = 8F/3 Если закрыть центральную часть линзы, то изменится лишь освещенность изображения, так 8*	227
как кружок закроет лишь часть лучей, формирующих изображение.
2)	fld = FI(d-\-F) — 1/2, 1—IJ4 — FI4 Линза хотя и двояковыпуклая, но рассеивающая, так как по условию задачи ее оптическая плотность меньше оптической плотности окружающей среды.
4.29.	В воздухе по формуле линзы имеем
1,11	. FL
Т+Т=Т> откуда
За счет преломления луча иа границе при выходе из линзы в воду sin <i/sin Р = я, где а—угол в воздухе, а Р—угол в воде; отсюда для малых углов а я; Р«. Если изображение в воздухе находится иа расстоянии f от границы воздух — линза, а от границы вода — линза расстояние h, то
fa х /гр, т. е.	h — fn — FLnl(L — F)
4.30.	Полая двояковогнутая линза находится в оптически более плотной среде; следовательно, линза должна быть собирающей. Параллельный пучок, падающий на иее, должен был бы
собраться в фокусе F (рис. 0.4.30), ио, преломляясь на границе раздела АВ, он попадает в точку F1 (угол падения равен а, угол преломления равен Р):
sin а	1		АВ		AB
sin р	п ’	tga=	BF ’	tg Р =	BF' ’
BF	tga „	sin а	1		F-h
BF'	tg Р "	sin р	n ’	Dr —	Л
Искомое расстояние
DF'_»+Bf=£+» (1-1) ►.
4.31.	При повороте зеркала на угол Да отраженный луч повернется на угол 2Да, т. е. угловая скорость вращения отраженного луча равна 2а>. Мгновенная скорость светового пучка, сфокусированного в точке экрана, таким образом, равна v=2aF
228
4.32.	На экране получится светлое кольцо диаметра 2г — = 2ftg0«2f0 (рис. 0.4.32); 0 = f}—a, sin a/sin f}= l/n, f} w na, 0«a(n —1), t. e. ^2r = 2Fa(n — 1). При n=l,5, a = O,l рад, F-l м диаметр кольца 2r = 0,l м
4.33.	Направим луч вдоль поверхности конуса и посмотрим, как он пройдет сквозь лупу. Построение хода луча после лупы
показано на рис. 0.4.33. Изображение острия конуса будет мнимым. Пусть оно находится на расстоянии I от лупы. Тогда
4-=4---г,
г а I
отсюда искомый угол 20 = 2 arctg [(1 — djF) tg а] ►.
4.34.	Построим изображение тени, учитывая, что пленка в фотоаппарате расположена в фокальной плоскости. Из построения
(рис. 0.4.34) следует, что х = (1—г)/3 Можно найти область тени и другим способом, предварительно отыскав положение изображения линейки.
4.35.	Часть жука, линейный размер которой равен х, дает изображение размера х' (рис. 0.4.35). Освещенность изображения £ = ф/5, где Ф c/)x2Q—световой поток, исходящий от части жука,
229
размер которой равен х, и распространяющийся в телесный угол QooD’/d2, a Seo(«')*. Поскольку x/x'=d/f, получаем
„ f d\t D* Ds Di(d—F)*
E\T) f* ~	’
В последнем равенстве величина f выражена через расстояние d
и фокусное расстояние F по формуле линзы. Приравнивая освещенности для di~3F и d% = 5F и разных диаметров диафрагмы объектива, получаем
Di__di (di — F) 6
Ds-diidi-Fy- 5
4.36.	Из построений на рис. 0.4.36 следует, что
Р	т rj Г2_ Xj+1/i
2(F—Xi—yi) yi ’ и У!
D _га Гг _ уг 2(F—ya) Vi' rt х2—у2' отсюда
4.37.	Параллельный пучок дает изображение в фокальной плоскости: ^r = Ftga^. Если >у.е размер линзы таков, что r/F < tga, то минимальный размер пятна, равный диаметру d ►, получается, если прижать линзу вплотную к экрану.
4.38.	Отразившись от зеркала, свет вторично проходит через ту же линзу, так что посеребрение плоской поверхности линзы эквивалентно удвоению ее оптической силы, что уменьшает вдвое фокусное расстояние линзы: F' = F/2
4.39.	<< SS'==/4-F (1 -f-l/cos а) >>.
4.40.	Проведем луч из источника S так, чтобы, пройдя через первую линзу, он далее прошел через фокус второй линзы (рис. 0.4.40). Тогда за первой линзой этот луч пройдет параллельно ее главной оптической оси, а за второй линзой~-парал-230
' лельно главной оптической оси этой лиизы и пересечет фокальную плоскость в точке S'. Эта точка и есть изображение источника S
в системе данных лииз. Опустим перпендикуляр 5Л на прямую О1О2:
01Д = Гсоза, 0гВ = F sin a = DS', AD	OtOz + 02D — 3F F cos a.
Расстояние между источником S и его изображением S' по вертикали
H = SA-}-DS', DS’ = Ftga; отсюда H = F sin а (1 1/cos a).
Таким образом,
<4 SS' = (Я2 + ЛО2)1/2 = Г |(1 + 1/cos a)2 sin2 a + (3+cos a)2)1/a
4.41.	Рассмотрим два луча, идущих от источника света. Пусть первый луч идет, совпадая с оптической осью первой линзы. Эту линзу он пройдет без преломления. Вторая линза преломит луч так, что он пересечет ее оптическую ось в ее же фокусе. Пусть второй луч идет в точку пересечения плоскости первой лиизы с оптической осью второй линзы. Так как источник расположен на двойном фокусном расстоянии от линзы, этот луч пойдет после преломления симметрично падающему, а после второго преломления он пойдет параллельно оптической оси второй линзы (так как выходит из ее фокуса).
Изображение источника (точка пересечения двух рассмотренных лучей) находится иа расстоянии F/2 от преломляющей плоскости второй линзы и на расстоянии й/2 от главных оптических осей обеих линз. Искомое решение
<4 SS' = (1/2) V49F2-j-/i2
, При построении хода лучей вместо линзы конечных размеров можно рассматривать преломляющую плоскость.
4.42.	-4 D=.4D0/3 = 2,6 см >.
4.43.	Пусть dt — расстояние от изображения источника света S, которое дает линза без отражения света от зеркал, до линзы.
231
Тогда
l.Sf+di F ’ dl 3F'
После отражения света от зеркал изображение возникает на расстоянии 2F от ребра двугранного зеркала, т. е. на расстоянии F слева от линзы (одно изображение в двугранном зеркале—такое же, как в обычном плоском, но перевернутое по сравнению с ним. Два боковых изображения мы не рассматриваем). Таким образом, на линзу падает сходящийся пучок света, пересекающийся за ней, что можно рассматривать как мнимое изображение с расстоянием до линзы d2 =—F. Линза еще раз преломит этот пучок света. Используя формулу линзы, получаем
1	1 । 1	F
F	—F ' х	2 г
Изображение действительное (пересекаются в найденной точке сами лучи, а не их продолжения).
4.44.	Возникнут несколько изображений.
а)	Изображение точки S в коническом зеркале после однократного отражения от зеркала — кольцо радиуса R = F/2 на расстоянии ◄ fi = 2F ► от линзы.
б)	Точечное изображение создается пучком лучей, распространяющихся к зеркалу в небольшом телесном угле и испытавших по два отражения (после двух отражений от заданной конической поверхности луч меняет направление на противоположное). В результате двукратных отражений лучи идут, как от точечного источника, расположенного на оси зеркала позади его вершины на расстоянии 2F-{-F/2 от линзы. После прохождения этих лучей через линзу получается точечное изображение на расстоянии 4 fa = 5F/3 > от линзы.
в)	Наконец, еще одно изображение точки S в линзе, создаваемое пучком лучей, падающих на линзу непосредственно без отражения от зеркала,— точка на расстоянии fs = 3E ► от линзы.
4.45.	Чтобы изображение S' совпало с источником, надо, чтобы изображение источника в зеркале также совпало бы со своим источником. Есть два случая:
1)	Изображение совпадает с центром сферы, тогда
/i ’d~+R	7
(Zj, а—расстояния от S до лиизы), откуда
ПРИ d > н d < R
232
2)	£'+;f::=7r • откуда ◄ = ПРИ d > F ►• Здесь лучи, идущие от линзы, попадают в точки пересечения оптической оси с зеркалом, тогда они отразятся под тем же углом к оси, что и пришли.
Рис. 0.4.46
4.46.	При d < F решений нет. При F < d < J?-}-/7 одно решение: x = Fdl(d—F). При d > R-\-F — два решения (рис. 0.4.46):
1 , 1	1 с d 1 ,	1	_ 1	„
Xi+dT’ X1~~F d—F ’ х2 +d-R F’ 2 F d-R-F^’
§ 4. Фотометрия
4.47.	x = /iKn2/3 ’^T
4.48.	Вклад в освещенность экрана вносят сам источник света и три его мнимых изображения:
„	. /cos a, . cos а2 , cos а, . coscz.X
Е — J ( —s-H---5-44---5-24---)=!
\ /1	11	/3 ll J
L (УУ	3 । 1 । 1
У 4	10 КТО 10^10^18 2 /’
где Zj, Z2, Z3, Z4 — расстояния от источника света и его изображений до точки А экрана, а углы ai, а2, а3, а4 ясны из построения изображений. Окончательно
. Р . J 25/54-18
< I2 45 /10
4.49.	По закону преломления sinP = nsina. Освещенность изменится по двум причинам: во-первых, меняется угол падения на экран—вместо угла, равного 0° будет угол [3— а; во-вторых, изменяется освещенная площадь экрана за счет изменения ширины пучка — вместо ширины АВ получается ширина CD (рис. 0.4.49). Таким образом,
Е2 АВ а
Ey ~ GF C0S
233
Найдем отношение GFIAB. Поскольку GF_\_LC, ML || АВ, LM — AB, / MLF — a.,	LFG=[5, LF = LM/cos a —AB/cos a, to
Z'l? rr а ЛП COSP	£2 COS (6 —a)
GF*=LF cos р = лВ ----£; отсюда -===----—я—- cos a.
cos a ’	Ex cos p
С учетом закона преломления окончательно получаем
п sin2 a
V" 1—я2 sin2 a
круглый, значит, его форма определяется формой Солнца, и, следовательно, освещен-
4.50. Раз зайчик
не формой зеркала, а
ность в центре зайчика зависит от телесного угла, под которым из этой точки видно зеркало. Для точки В имеем Qi —(S//i) cos ец. Пусть освещенность на стене вне зайчика равна Ео. Освещенность, создаваемая в точке В зайчиком,
S
£я = 3£,0 —£,0 = 2£'в, 2£0 = А — cos «!, 11
где k—некоторый коэффициент пропорциональности, дающий, в частности, и правильную размерность. Для точ
ки С имеем Qa = (S//l) cos a2. Освещенность, создаваемая зайчиком в точке С,
Ес = k—г cos a2-cos р, где Zi = /2 cos Р, /а
А = 2£0
ll
S cos
В итоге
г- с- Г1 । л ( li V cos “а „ « I г- (, , о cos a2 , Ес — £л 1 + 2 I "т— }  1	 cos р I — jc о I 1 —1—2 ——— cos р | <
L \ 4 / cos <Zi J \ cos aj /
По условию задачи р = л/4, а1 = л/8, аа = л/4. Окончательно
<4 £с = £ор4-
2 cos (л/8) ] ~
Таким образом, изменение освещенности произошло по трем причинам: а) изменение расстояния от стены до зеркала' (множитель (Zi/Z2)2); б) изменение угла падения лучей в центре зайчика (множитель cos Р); в) отличив угла между направлением от зеркала к центру зайчика и плоскостью зеркала от 909 (множитель cos а2/cos at).
234
4.51.	Так как источник света находится в фокусе зеркала, то на пластинку справа падает параллельный пучок, отраженный, следовательно, с той же площади зеркала, что и площадь пластинки. Вследствие малости пластинки можно пренебречь изменением угла падения лучей, попадающих на нее. Тогда
Еа 2/(/г24-г2/4)3/г lr2	.	.
7Г= =------—-------- « -----------— при h < Г ►,
£п r(h2 + l2)3/2	4 (/i24-Z2)3/2
4.52.	Ес ~ —s-, Ел ~ -4---, где Ес и Ел—освещенности
1с ‘с- 2л12
поверхности Земли, создаваемые Солнцем и Луной, 1с—расстояние от Солнца до Луны или Земли. В итоге
-4 п=^-	«0,9.10-»
Ес 2 \ I j	г
Глава V. ЗАДАЧИ-ОЦЕНКИ
5.1.	При броске без разбега максимальная дальность полета гранаты I ~ oo/g. Полагая, что за счет разбега гранате сообщается дополнительная горизонтальная скорость о и что вертикальная составляющая скорости о0 sin а0 практически не меняется, а значит, не меняются и высота н время полета
, 2vo ~ О v0
I '1 sin ссл=—-------
g	g
(для а0 = 45° с учетом, что о0 ~ У 1g), получаем увеличение дальности полета tv ~ У 2.1/g v ~ 30 м при о ж 10 м/с, I и 50 м.
5.2.	Бег—последовательность фаз полета после толчков ног. Если считать в грубой модели, что характер толчка сохранится, то дальность полета между толчками s ~ (2n0/g) sin a cos а (здесь и0 — начальная скорость в фазе полета, а—угол, который она образует с горизонтальной плоскостью, g—ускорение свободного падения), время полета т ~ (2n0/g) sin а; отсюда горизонтальная скорость v=s/x ~ va cos а не зависит от ускорения свободного падения, т. е. скорость бега по порядку величины должна быть близкой к скорости на Земле. Результат можно увидеть сразу, так как горизонтальная составляющая скорости, если не учитывать особенности толчков, не должна зависеть от ускорения свободного падения.
5.3.	Пусть объем воды в ваине равен SH, а время вытекания воды равно I, тогда SH ~ Sovcpt. Полагая среднюю скорость вытекания воды ос₽ ~ У2§Н и взяв высоту уровня воды Н к 0,5 м,
235
площадь отверстия в ванне So « 10“® м2, площадь ванны S « 1 м2, получаем
, SH
t ~ —р=— и 3 мин.
К 2gHS0
5.4.	Угол а = Z ВО А и AB/R = V (R^h.)2 —R2/R х Vih/R'. Обозначения ясны из рис. 0.5.4. Время поворота Земли на угол а. равно
,	, Т A/'2h Т Г~ Т
/ = и/ш = а—«]/——«]/_ _^15мин
при 1? = 6,4-103 км, /i=10 км, 7=1 сут. Заменив R на R cos <р, можно учесть широту <р. Близкий к полученной оценке порядок дает учет рефракции.
5.5.	За время перемещения t на длину кузова I центр тяжести машины наберет вертикальную скорость ац т ~ gl/v и угловую скорость со ~ г)ц.т-2//~ 2g/ti. Тогда число оборотов автомобиля п ~ /пад<о/2л ~ yigH/nv х 1,5 при горизонтальной скорости ц = 30 м/с.
5.6.	Приняв скорость самолета v х 3,3-102 м/с, имеем х х х v У 2h/а х v У2ft/10g = 4,5-102 м. Здесь предполагается, что летчик выдерживает кратковременную (около 1 с) перегрузку, равную 11g.
5.7.	Отклонение к западу обусловлено вращением Земли с угловой скоростью со: х ~ [со (/?з -j- /г)—соРз ] i, где высота полета снаряда h = vl/2g, время полета снаряда t^v^/g, R3—радиус Земли, со=2л/Т — угловая скорость вращения Земли с суточным периодом Т, Таким образом, отклонение снаряда к западу
х ~ "Т"^2" ~10 м при у° ~8,102 м/с‘
5,8.	Приравнивая выталкивающую силу силе тяжести, действующей на сферу радиуса г, и пренебрегая плотностью гелия по сравнению с плотностью воздуха р ~ 1 кг/м3, получаем г ~ mjbp х 102 м.
5.9.	По условию равновесия воздух действует на вертолет снизу вверх с силой, равной силе тяжести, действующей на вертолет. Слой воздуха между вертолетом и водой при небольшой высоте с такой же силой действует на воду, создавая в ней яму. Таким образом, вертолет вытесняет такую же массу воды, какую имеет сам (воздух между вертолетом и водой при этом играет роль передающего силу элемента и из рассмотрения может быть исключен). Выбрав массу вертолета m х 104 кг, длину лопастей I х 5 м, полу чаем глубину ямы й ~ m/рл/2 ж 0,1 м.
236
в.10. Средняя глубина океана Н » 4 км. Тогда Др~ (2/3) «3-107 Па. При этом учтено, что океан занимает две трети поверхности Земли. Допрлнительное давление пара воды Др эквивалентно давлению слоя воды, который появился бы, если бы всю воду океана распределить слоем одинаковой толщины по всей поверхности Земли.
5.11.	Изменение плотности воздуха Др~р, а изменение давления воздуха в шахте Др ~ (р + Др) gH ~ 2pg// » 2-10? Па, т. е. Р =	~ 3> 105 Па.
5.12.	На водных лыжах лыжник не тонет, когда сила тяжести mg, действующая на него, уравновешена вертикальной
Рис. 0.5.4	Рис. 0.5.15
составляющей силы действия воды на лыжи, которая пропорциональна p0u2S cos a-sln а,-где 5—площадь лыж, a— угол наклона нх к горизонту. Если пренебречь выталкивающей силой, то при т = 80 кг, 5 = 0,4 м2 имеем
p0u2S cos a-sln а ~ mg, v ~ Y2mg/p6Sx 2 м/с.
5.13.	Сравниваем моменты сил: Fr ~ tng-2r, где г —радиус зубчатого колеса, 2г—расстояние от осн этого колеса до педали, mg—сила тяжести, действующая на человека, равная максимальной силе давления на педаль. При т и 70 кг получаем F ~ 2mg « и 1,4-103 Н.
5.14.	Работа против сил F на пути 5 (размер обуха) равна FS х mv2/2, где т — масса топора, р—его скорость. Топор, как и руки, движется примерно с той же скоростью, что и ноги при беге. Таким образом, F ~ mv2/2S х 5-103 Н при mxi кг, 5 к 10~2 м, v х 10 м/с.
5.15.	Используем обозначения, введенные на рис. 0.5.15, где Т—сила натяжения веревки, т— масса белья, h—провисание веревки, а I—расстояние между ее концами. Условие равновесия 2Tsina = mg для малых углов а переходит в более простое выра
237
жение 2Ta — mg, причем тогда а ~ Л//; отсюда Т ~ mgl/2h » 103 Н при т = 10 кг, Z = 10 м, ft = 0,5 м, g=10 м/с1.
5.16.	v~ (1/2л) Уg/l х 0,5 Гц при I и Гм.
5.17.	F ~ 5mg (см. также задачу 1.138). При оценке предполагается, что вся масса гимнаста сосредоточена в его центре масс. Для m « 70 кг получаем F х 3,5-103 Н.
5.18.	В треугольнике АВС (рис. 0.5.18) сторона Лб равна mg, сторона АС равна тш2г, АВС — а — малый угол отклонения отвеса от направления на центр Земли, <р — широта, г = /?зсо8ф. Тогда по теореме синусов имеем
та2г___ sin а	а
mg sin [л — (фф-а)] ~ sin <р
(с учетом малости угла а), т. е. ma>2r~amg/sin <р; отсюда
[Дз®2 .	®2Дз . о
а х —— sin q> cos <р = —sin 2<р.
Таким образом, хиЮкмпри /?з =6,4- 10е м, <р = 45°, Т = 8,6-104 с, g = 10 м/с2.
5.19.	В отличие от полюса на экваторе наблюдаются два эффекта: во-первых, уменьшение веса тела за счет центростремительного ускорения пц. c = ®2f3 и, во-вторых, увеличение расстояния от поверхности Земли до ее центра.
 Если использовать гидростатическую модель распределения давления внутри Земли, то получим, что давление в центре Земли, создаваемое «столбами вещества» от полюса до центра Земли, имеющими высоту г3—х, и от экватора до центра Земли, имеющими высоту rg, согласно закону Паскаля, должны быть равны. Учитывая, что ускорение свободного падения на полюсе равно g, а на 2
экваторе g—согз, получаем
g .	. g~O2r3	ГЗ®2 гз4л»
р-J (г3—х) ~ р —g----гз; отсюда х----— — — —
где 7=1 сут. Окончательно х х 20 км.
5.20.	Средняя плотность вещества Солнца
Pc“A/c/l/c=3'wc/4^c-
Из второго закона Ньютона находим массу Солнца:
gT2 ’
4 л2 гс т3Мс	;24ла3
тЗ—^-=в -уг- 1 отсюда рс~ -у^-
1 г/см3,
где а = £>с/г3с ж 0,01—угловой размер Солнца (Dc—диаметр Солнца, гс — расстояние от Земли до Солнца), Т х п-10’ с=1 год, G — гравитационная постоянная,
.238
5.21.	За время Д/ на парашют площади S налетает со скоростью v воздух массы Дт — p»S Д/ и передает парашюту импульс Дтг>. Таким образом, сила сопротивления воздуха Е = = brnv/kt ~ рц25.. Эта сила при установившемся движении уравновешивает силу тяжести, действующую на парашютиста:
pv2S и mg;
отсюда v ~ Vmg/pnr2 и 5 м/с при т и 102 кг, г х 3 м.
5.22.	Введем размеры автобуса: высоту 2ft, ширину 2d, длину I. Если плотность воздуха р, а скорость ветра v, то давление ветра создает силу, действующую на боковую поверхность автобуса; F ~ Z-2/ipa2.
Рассмотрим моменты сил отрицательно горизонтальной оси, проходящей через точки соприкосновения переднего и заднего
Рис. 0.5.26
т — масса автобуса; отсюда
- 104 кг, d=h ~ 1 м, I а 8 м,
колес с дорогой: Fh ~ tngd, где ~ У mgd/2plh2 х 70 м/с при т р » 1,3 кг/м3.
5.23.	Принимая высоту подъема центра тяжести Д/i при переворачивании равной примерно 0,5 м, имеем v ~ У 2g Д/г аг 3 м/с.
5.24.	При небольших деформациях шины х сила пропорциональна деформации: Е = хх. Если F = mg, х = \ (где Д — размер сжатия шины при нормальной езде по ровной дороге), то л-=Р!х — = mg/&. По закону сохранения энергии имеем
mv2 хх2 ~ mg d2 ~2~'~Т *~~2’
где в соответствии с задачей положено x = d (здесь d—диаметр сечения шины); отсюда v х d Уg/\ х 2 м/с при Д ж 10~2 м, d и » 5> 10~2 м, g= 10 м/с2.
5.25.	F = та ~ mv2/2l к 7* 103- Н при массе человека т х 60 кг.
5.26.	Пусть центр тяжести О грузовика находится на расстоянии h от земли, /—длина грузовика, F — сила трения, &F=F2—Ff— разница сил, действуюших на передние и задние колеса (рис, 0.5.26).
239
Пусть при этом масса грузовика равна т, ускорение равно а, ско« рость равна V. Из-за отсутствия вращения относительно оси, про» ходящей через точку О, имеем Fh » &FI/2, откуда ДГ ~ 2Fh/l. Из второго закона Ньютона следует,- что ma^F, Причем если ввести путь торможения L, то о2	, „ тиъ h
aK_iemf!ibF„Tr
Выбирая ft й 1 м, I й 5 м, m s 10* кг, L = 10 м, о=10 м/с, полу» наем &F к 10* Н. Прн использовании выражения для силы трения Ft&kmg имеем ДГ я 2kmgh/l» 10* Н .при Л = 0,5.
6.27. Грузовик массы т, имея в начале торможения скорость о, обладает кинетической энергией пи>*12. Вся эта энергия переходит в теплоту за время торможения t где /—путь торможения. Средняя мощность, таким образом,
Р ~ mo2/2:2//t>=mv®/4/,.
Приискорости о» 60 км/ч=16 м/с, т к 10* кг, I « 10 м .получаем fslO® Вт, т, е, в виде теплоты выделяется мощность порядка мегаватта!
5,28. Пусть скорость велосипедиста на финише равна о, а сила сопротивления воздуха F ~ рв25, где р—плотность воздуха, aS — эффективная площадь велосипедиста, которая встречает набегающий лоток воздуха. Тогда мощность Р к Fv я po3S. Полагая р«1 кг/м3, V « 60 км/ч » 16 м/с, S » 0,5 м2, получаем Р « 2 кВт.
6,29, Последние метры ft капля пролетает с постоянной, ско» ростью, так как сила тяжести mg уравновешивается силой сопротивления воздуха F:mg=F. Работа силы сопротивлений ^=FCOnpft= f=>mgh. Масса капли
/п=р«4лг3/3л,р«4г3, т. е. Д»4рг3§Л,
При р = 103 кг/м3, гя1 мм=10~?м, g —10 м/с2, ft = l м имеем Л~4*10~6 Дж.
*5.30. За время t кузнечик, распрямляя ногу длины I, набирает скорость V. Средняя мощиЪсть кузнечика
где Я—высота полета кузнечика; отсюда
Полагая /»3*10-2м, Я«1м,§» 10 м/с?, имеем P[tn~(2gH}W[41tt й0,8 кВт/кг и 1 кВт/кг,
€48
5.31.	Средняя сила при выдергивании гвоздя равна приблизительно средней силе F при его забивании. Из закона сохранения энергии получаем F ~ 5mv2/2l. При массе молотка т я 1 кг, скорости его ня 5м/с и длине /я 10 см сила, необходимая для выдергивания гвоздя, F я 103 Н.
5.32.	Потенциальная энергия mgh переходит в работу силы реакции опоры, действующей на ноги человека. Человек гасит свою скорость, приседая на расстояние I, равное примерно половине его роста: F ~ mgh/l я 3,5-103 Н при т я 70 кг, hjl ~5.
5.33.	Работа силы равна
т. е. Г~^я8-102 И,
при массе ядра т я 8 кг, длине полета ядра L я 20 м, пути, проходимом ядром вместе с рукой при разгоне, I я 1 м.
5.34.	Грубая оценка по порядку величины из закона сохранения энергии дает
mv;, „ ц. т
^-Fh, F~^g,
где Л4чел — масса человека, f—сила давления одной ноги стоящего человека. Конечно, неточно £,Кин = ”1Уц т /2 (здесь оц. т—-скорость центра тяжести, т — масса ручки), но в грубой модели такая точность достаточна:
так как ц = сог, т. е.
vx ~ 2 j/" h — g ® 15 м/с
при М/т я 70, h я 0,1 м, I я 1,5м, §=10м/с2.
5.35.	p = F/S ~ F/d^, где F я 1 Н, d я 0,2 мм — ширина следа, оставляемого шариком на бумаге, ряЗ-407 Па.
5.36.	Разгоняется вода, а вместе с ней и снаряд, до скорости о g/i ~ 102 м/с при h я 1 км.
5.37.	Струя движется только за счет давления столба воды p = pgh. Из закона сохранения энергии для единицы объема при плотности р и скорости v имеем
---pgh, откуда v я V2gh,
При h = 100м, g=10M/c2 скорость v я 45м/с,
9 Г. В. Мел един	241
5.38.	Работа снл давления пороховых газов идет в основном на разгон пули: pcpSZ ~	, где Зя 1 см2= 10-4 ма, длина ствола
I ж 0,5 м, масса пули m ~ 10г= I0-2 кг, v х 8.102м/с, рсо~ ~ mo2/2S/ « 6-Ю7Па.
5.39.	Мощность
р ~ mv2/2 ~ т ’
где т—время сплющивания пули, т — ее масса, v—скорость. Если ввести длину пули I, являющуюся приблизительно расстоянием, на котором гасится скорость при расплющивании, то
~ 1 -21
'	Т ~ р/2 v ’
Таким образом, Р х mv3/4l х 2-10вВт = 2МВт при т«3<10_?кг, I х 10~2 м, v х 3-102 м/с.
5.40.	Изменение импульса капли при ударе, равное mv, определяется произведением средней силы Ecp^PcpS, действующей на каплю, на время взаимодействия капли со стеной t~r[v, где г — радиус капли, a v—ее скорость. Таким образом, mv~Ft, или р (4/3)	~ рсрлг2-г/п,	VрСр/р ~ 30 м/с.
5.41.	Используем гидростатическую модель распределения давления внутри Земли. Высота столба вещества равна радиусу Земли, gcp = g/2. Давление p~pgcpr3~p (g/2) r3, р = М 3/V3~3g/4nGr3, т. е. p~3g2/2itG ~ 1011Па.
5.42.	Из второго закона Ньютона для небольшого объема воды получим р = pg -|-р (gh-\-alo) ~ Ро+Р (v2/2Z) /0» 1,4.105 Па при высоте цистерны Ля 1м, длине цистерны /оя5м (учтено, что ускорение машины а~о2/2/).
5.43.	Работа пороховых газов A~Fl, где I—длина части пули внутри гильзы. Так как
то скорость пули
v=—=— 1Л2Л—г-^-п  . та т„ Г тг-\-та
В ружье работа пороховых газов A0~FL, где L—длина разгона пули в стволе. Скорость пули при стрельбе оо~(1/тп)УГ2Л0/ип без учета отдачи ружья. Таким образом,
А/Ао
242
Полагая силу давлении пороховых газов F приблизительно постоянной и одинаковой в этих двух случаях, имеем
о—'Оо у	~ 40 м/с
при 1/L и 102, тп/тг яа 3.
5.44.	Введем ряд параметров: давление воздуха в аудитории р, ее объем V, массу воздуха М, температуру Т, молярную массу воздуха ц. Тогда по закону Клапейрона—Менделеева имеем pV = (М/ц) RT. Число молекул воздуха N выражается через постоянную Авогадро следующим образом:
N~Na (л ~Na rt‘
Кинетическая энергия молекулы воды равна
mv2__3 ЬТ_ 3 RT
2 ” 2	~ 2 Na'
Запас энергии в воздухе
ч ч E=±kTN=±pV.
Вводя удельную теплоемкость с воды и записывая уравнение теплового баланса, для массы т воды, нагревающейся на Л/, имеем
Е = ^-pV = cm&t, откуда m =	~ 102 я 103 кг
2	2сАг
при ри Ю^Па, V=1,8-103m3, AZ = 80K, с=4,2-103Дж/(кг-К).
5.45.	Кинетическая энергия осколков и воды при разлете примерно одинакова (давление внутри бутылки после превращения воды в идеальный газ мгновенно возрастет больше чем в тысячу раз, так как из уравнения Клапейрона — Менделеева имеем p = pRT/p) и в сумме будет равна начальной энергии воды:
mv2~pV~mRT/p- отсюда RT/p.
Полагая ц = 0,018 кг/моль, 7? = 8,31 Дж/(моль>К), ТяаЗООК, получаем v«400m/c.
5.46.	Закон сохранения импульса: mv~Mu, где т и v—масса и скорость молекул, М н и — масса и скорость мяча, h~u2)2g~ ~m2v2/M22g, масса молекул m~pVp/RT г при объеме мяча F = (4/3) лг3 = 4-103 см3 (если г»10см). Полагая давление внутри мяча ра:1,5р0, [1 = 0,029кг/моль, Л4?в4-102г, скорость молекул оа;4-102м/с и g~ 10 м/с2, получаем /1»2м.
5.47.	Масса кислорода в атмосфере Земли М = (1/5) 4nr^p0/g. Число молей взаимодействующих углерода и кислорода одинаковы,
9*
243
так Как при сгорании угля идет химическая реакция С+Оа = С©а: тс/Рс==/По2/Но,- т- е- Wo, = OTcf‘o,/lic-
Таким образом,
~ЛГ	4яг|РоИс ~ 5 10 ’
так как г3«6,4-106м, ро~1О6Па, рс = 0,012 кг/моль, Ро2 = == 0,032 кг/моль.
5.48.	Средняя плотность вещества человека близка к плотности воды (человек, почти полностью погрузившись в воду, может неподвижно лежать, находясь в состоянии равновесия). Плотность воды ро известна. Объем человека можно оценить по формуле V—m/f'o, rjsfi in — его масса. Выталкивающая сила F равна плотности воздуха р, умноженной на объем человека V и иа ускорение свободного падения g: F ~ tngplpa № 1Н при т ж 75 кг.
5.49.	С учетом сохранения объема аудитории и давления в ней по закону Клапейрона—Менделеева получаем
Ат ~ тАТ/Т = pVAT/T,
где р — плотность воздуха, а V — объем аудитории. При V « 2-103 м8, Т « 300 К, р « 1,3кг/м8 получаем Д/n я 80 кг.
5.50.	Так как объем и масса газа в баллоне сохраняются, то по закону Шарля Ро1Т = р11\, где р0 и То — атмосферное давление и температура. Давление внутри выключенной лампочки выбира. ется таким, чтобы после нагрева поднявшееся давление р нагретого до температуры Т газа сравнялось с внешним р0. Тогда р<=раТй/Т. Полагая То « 300 К, 7 « 400 К, получаем р = Зр0/4 « « 0,7-106Па.
5.51.	Сила F — ApS = (рв — р) S, где S — площадь, а Дд—перепад давлений, который возникает из-за охлаждения воздуха в стакане и из-за изменения объема, так как вода, получившаяся из льда, занимает меньший объем. Из обобщенного газового закона имеем
Ро (Ур Ки)_Р (Уо Ув)
То - Т ’
где р0—атмосферное давление, р—новое значение давления, Ул — объем льда, Кв —объем воды, Уо — объем стакана. В итоге имеем
f=""s(^+¥^)-30H
при У, = 20см8, 70 = 273К, Д7=Т—Тв=20 К, S=20cm2, Кд=110см8.
5.52.	Давление в остывающей медицинской банке падает. В результате возникает сила F, прижимающая байку к спиие.
244
Пусть температура и давление газа в горячей банке равняются Т\ и pi, а в остывшей — Та и р2, тогда F(рх—p2)S. Масса и объем воздуха в банке постоянны, поэтому Pi,T1 = Рг!Тг\ отсюда
Р2 Р1 у, ,	‘ ^Р1 ( I Т )	Т
Пусть ДТаЛООК, Т^аМООК, Sx 10 см2, рх = 105 Па. Тогда Р?»25Н.
5.53.	Пусть d—средний диаметр стволов и/ — среднее расстояние между деревьями. Мысленно сместим деревья от наблюдателя так, чтобы стволы образовали сплошной круговой забор, радиус которого равен искомому расстоянию х. За таким забором человека не будет видно в любом направлении. В заборе длины 2лх окажется примерно 2nx/d деревьев, которые были сдвинуты с площади лх2. Если на площадь S~/2 приходится в среднем одно дерево, то на /
площади лх2 их будет лх2//2. Таким образом, — о Г % 2лх/с/й:лх2//2, откуда, полагая/~3м, йя0,2м, Г* * И получаем x»2/2/d« 102 м.	\	/
5.54.	Возможна оценка, как при ударе мя- \. J' ча о стенку. Вводя обозначения (рис. 0.5.54):
R— радиус мяча, г—радиус площади соприкос- Рис- 0.5.54 новения мяча с плоскостью, Ар — превышение
давления внутри мяча над внешним, F—сила удара, m— масса мяча, получаем
——, х < R; отсюда r2»x-2R.
Из второго закона Ньютона имеем
/ла = Г = Дрлг2, fa:Apn-2Rx,
т. е. сила линейно зависит от деформации (как в законе Гука); отсюда, рассматривая коэффициент при х как жесткость и, получаем и = 2л7?Др. Вводя массу мяча т, получаем для частоты колебаний
2л/Т — &—Уи1т — У 2nRA.pl т, откуда
Д/= 7'/2 = л/<» = YnmftRAp и 10-2с.
5.55.	Выберем размеры матраца: длина /я2м, ширина hxO,5 м, толщина d«O,l м. Работа А силы F на пути (л/2) Z/2 при сгибании матраца приблизительно равна pAV-, где р — избыточное давление в матраце (р»104Па), а ДГ-—изменение объема матраца,' возникшее из-за изменения его формы в области сгиба (ДИ^й2/!),
245
Таким образом, г л I	с 4pd*h
~2~2— Р^1’ 0ТКУДа Р ~ • л~ ~ 30 И.
5.56.	Полагая объем парной V ss 100 м3, из уравнения Клапейрона—Менделеева для водяного пара (т — масса пара, Др —изменение давления, р, — молярная масса воды, Т — температура пара) имеем
. т	. т RT
ДрУ = — RT, т. е. Др ~-----тг.
Полагая /п=1кг, Тк350К, р. = 0,018 кг/моль, У«102м3, R = 8,31 ДжДмоль-К), получаем Др«1,6-103 Па~0,02 атм. Возможно решение и через сравнение масс воздуха и пара в парной.
5.57.	Сила, действующая на шток насоса, примерно равна весу человека: £о«5ОО Н. Площадь поршня Sk3-10~3 м2, избыточное давление !±ркР01$к2-105Па. Сила, действующая на застрявшую дробинку, лг2» 1 Н при гк 1,5* 10~3м. Ускорение дробинки
а ~ — = ,Л)С\—3 ~ 104 м/с2, т р (4/3) лг3
где рйЛ04кг/м3; отсюда отах» У^а/^УО м/с, где /и0,5 м—длина выходной трубки.
5.58.	При оценке скорости воздуха можно рассуждать так. Объем легких V~Svt, где S —сечение трубки, о —скорость воздуха, t—время выхода. При 5^0,5 см2, У«5л, /а:5 с получаем г~У/5/я;20 м/с. Другой подход дает несколько иную оценку. Человек ртом может создать перепад давления Дра;104Па. Но Арлгри2; отсюда о^р^р/р ~ 30м/с.
5.59.	Объем газа, выходящего через отверстие площади S со скоростью v в течение времени /, равен гл$/~У/2. Скорость выхода газа о можно оценить или по его температуре: тц2/2~ЗИ72 = ~3RT/2Nj, или просто приравняв скорость газа v скорости звука сй:3-10гм/с. Пусть объем спутника Уй;1м3 и S=10-4m2 при о»3-102м/с, тогда t~V]2vSKi0c..
5.60.	Число ходов насоса n = mM/mll~VKpl,,l/Vup0 (при каждом ходе насоса в камеру мяча поступает одно и то же количество воздуха: m^p^S^/RT), пока мяч не будет надут и масса воздуха в нем не станет равной тм = р (4/3) лг3|л/(/?7'). Выбирая радиус мяча гяЮсм, давление рк!,5 р0, рабочий объем насоса VH = /S« rj2-102cms, получаем пи30.
5.61.	Дополнительным давлением, создаваемым резиновой оболочкой, можно пренебречь. Выталкивающая сила, действующая иа шарик, равна mog = pK|xg//?7’e, где Тв — температура окру-246
кающего воздуха. Масса нагретого до температуры Т воздуха m = pVp,/RT. Тогда, если М — масса резиновой оболочки, получаем условие подъема шарика
mag^(M+m)g-, отсюда Т >
При Л4»5г, диаметре шара 2г»35см, 7^и300 К и р»106Па получаем, что критическая температура 7’кр»500К»200°С.
5.62.	Энергия подводного взрыва пгк идет в основном на работу против сил гидростатического давления pV. Тогда
5.63.	Минимальный угловой размер, еще различимый глазом, a=/i/L»10~s (до таблицы, на которой различаются миллиметровые детали размера h, расстояние L равно нескольким метрам). Выбрав фокусное расстояние глаза F«2cm (размер глазного яблока), имеем a, = h/L~x/F, x~Fa»2-10~6м.
Если исходить из дифракционных ограничений на разрешающую способность глаза, то аЛИф~Х//Э, где Х»0,5 мкм—длина световой волны, a Dm2—3 мм—диаметр зрачка. Тогда x~/-7,/D~10~5m,
5.64.	Пусть ДЯ—размер иа экране минимальной детали, различимой глазом зрителя с расстояния I, Н—размер экрана, Д/г—
размер кристаллика («зерна» на пленке), h—размер кадра, (рис. 0.5.64). Чтобы на экране ие было линейных искажений деталей, должно строго выдерживаться подобие: &,H]H=&hlh.
Угловое разрешение ДО глаза (отношение мельчайшей детали, еще различимой глазом, к расстоянию до нее) определяется, например, из воспоминаний о посещении глазного кабинета: с расстояния нескольких метров на таблице для проверки зрения видны детали букв, не меньшие нескольких миллиметров, т. е. ДО» 10“3. Тогда AHailAQ, где I—расстояние от зрителя до экрана. Таким образом, Д/г/~/гДЯ/Я~-/г/Д0/Яя0,04 мм при / = 20м, /г» 1см, Д0»10-3, Я = 5м.
Й47
5.65.	Человек видит две близкие точки, как одну, т. е. слив,’ шимися, с расстояния нескольких метров I, если расстояние d между этими точками не превышает нескольких миллиметров (вспомните проверку зрения у врача-окулиста). Для большего расстояния h между двумя точками (расстояние между рельсами) они сольются иа большем расстоянии L. Имеем d/1л 10~3ash/L‘, отсюда LaM/das\ км при has 1,5 м.
5.66.	Из геометрических построений хода лучей через зрачок глаза h!H~d/l (рис. 0.5.66), откуда hasHd/l. При росте человека Ди2м и размере глазного яблока а~2см имеем для расстояния до человека /~10м: hasHd/laA-10-3 м = 4 мм.
5.67.	Свет, падающий на линзу диаметра О0^.5см, собирается в фокальной плоскости, формируя пятно — изображение Солнца — диаметра DasaF — DqF/гс, где а — угловой размер Солнца, равный отношению диаметра Солнца Ос к расстоянию гс от Земли до Солнца. Угловой размер аж0,01. Отношение |3 мощности излучения, падающего на линзу, к мощности излучения, падающего на экран, обратно пропорционально отношению площадей линзы и изображения Солнца на экране: |3 = O3/O2 = D0/aF2«4-102 при Г = 25см (оптическая сила линзы равна 4 дптр).
5.68.	При скорости иглы v и периоде колебаний Т характер, ного звука соответствующий размер неоднородности 1~иТ~иЦ, где f—частота звука. Выбрав радиус звуковой дорожки га: 10 см и учитывая, что частота вращения долгоиграющей пластинки v=33 MHH_1a:0,5 с-1, имеем v = cor = 2nvrx:30 см/с. Полагая характерную частоту звука /»103Гц, получаем l~2mR/fasQ,3 мм.
Глава VI. ЗАДАЧИ-ДЕМОНСТРАЦИИ
6.1.	Гвозди намагничиваются. Возникают магниты, одноименные полюсы которых расположены рядом. Но одноименные магнитные полюсы отталкиваются. В точках подвеса отталкиванию препятствует трение, а внизу концы гвоздей, висящие свободно, расходятся в соответствии с действием силы магнитного отталкивания и силы тяжести.
6.2.	Предположим, что гвоздь вытаскивают, не расшатывая его (при расшатывании просто расширяется канал, в котором находится гвоздь). Основной причиной облегчения выдергивания гвоздя при его проворачивании является изменение направления силы треиия, которая при проскальзывании направлена против скорости. Согнутый гвоздь проворачивать нетрудно (большое плечо силы— большой момент силы при малой самой силе), а сила вытягивания резко уменьшается,- так как надо компенсировать не всю силу 248
трения, направленную против полной скорости гвоздя, а лишь ее составляющую, направленную вдоль гвоздя.
6.3.	Наибольшая дальность отскока получится, если составляющая переданного шарику импульса, направленная вдоль нити, будет минимальна (т. е. если в момент удара линия, соединяющая центры шарика и препятствия, перпендикулярна нити, то составляющая импульса вдоль нити равна нулю и дальность отскока получится наибольшей из всех возможных). При смещении по горизонтали круглого препятствия составляющая импульса, переданного шарику, направленная после удара вдоль нити, увеличивается, и, значит, уменьшается доля энергии, идущая на движение шарика по горизонтали. Импульс, направленный вдоль нити, и связанная с ним кинетическая энергия оказываются потерянными из-за неупругости нити.
6.4.	Из законов сохранения энергии и импульса следует, что при упругом ударе двух одинаковых шариков происходит обмен их импульсами. Поэтому в первом случае скатившийся с горки шарик 1 передаст свой импульс ближнему шарику 2, а сам остановится. Тот в свою очередь передаст этот импульс шарику 3, а сам останется неподвижным. Таким образом, будет двигаться лишь крайний шарик 3.
Если между скатившимся с горки шариком 1 и шариком 2 помещена толстая резина, то ее роль сводится к продлению времени передачи импульса шарику 2.
Рассмотрим по этапам процесс передачи импульса, который на самом деле носит плавный характер. Резина передает импульс шарику 2, а тот—шарику 3. Передаваемый резиной шарику 2 импульс в течение некоторого времени возрастает, и, следовательно, возрастает и импульс, передаваемый шариком 2 шарику 3. Шарик 2 будет как бы достигать в течение времени передачи резиной импульса шарика 3 и добавлять ему импульс. Это будет продолжаться до тех пор, пока шарик 2не оторвется от резиновой прокладки. После этого оба шарика 2 и 3 покатятся с неизменяю-щимися и почти равными скоростями.
6.5.	В первом случае в отсутствие препятствия сила трения преодолевается при натягивании нити по горизонтали. Когда нить отпущена, горизонтальная сила трения покоя держит подставку.
Во втором случае до столкновения маятника с препятствием подставка неподвижна, как и в первом случае. А после столкновения импульс шарика mv за укороченное (из-за уменьшения длины маятника) время Л/ ~ л l/g передается подставке, и, следовательно, та же сила трения преодолевается большей горизонтальной силой.
Можно подойти к объяснению и иначе. Горизонтальная обставляющая силы йатяжёйия легко выражается через углы
249
отклонения нити от вертикали:
7'rop = /ng sin а (3 cos а—cos а0).
Чем больше максимальный угол отклонения маятника «0, тем больше горизонтальная сила. После удара нити о препятствие максимальный угол отклонения увеличивается. Возрастают и средняя горизонтальная составляющая силы, преодолевающая силу трения покоя и ее максимальное значение. Это и приводит к смещению подставки в направлении от руки к препятствию.
6.6.	В отсутствие качаний сила натяжения нити
Т = mg -С	(k cos а — sin а).
При качаниях, когда маятник проходит вблизи положения равновесия,
Т	1 mv2	( 1 I 2/1 \
T = mg-|—— xmg{ l+y 1, т. е. сила натяжения увеличивается и, нарушая равновесие, рывками сдергивает брусок вниз.
6.7.	Когда брусок кладут на пару роликов, сила трения, действующая на него со стороны заклиненного ролика В, направлена
влево (рис. 0.6.7) и стремится повернуть брусок по часовой стрелке. В результате сила давления на правый (нижний) ролик будет больше, чем на левый (верхний). Поэтому в первом случае максимально возможное значение силы трения покоя F-ip — kN^ (рис. 0.6.7а) будет меньше, чем во втором (рис. 0.6.76). Если брусок начал соскальзывать, то благодаря тому, что центр тяжести смещается вправо, сила давления на левый ролик будет уменьшаться, а на правый—увеличиваться. Поэтому в первом случае брусок, начав съезжать, не сможет остановиться, а во втором — остановится.	:
6.8.	Во втором случае гильзе и стержню передаются равные импульсы, но стержень закреплен, его скорость остается равной нулю. Поэтому почти вся энергия идет на разгон гильзы. В первом же случае заметная доля энергии передается стержню, и высота полета гильзы должна быть заметно ниже. Несложно сделать 250
оценки (m—масса гильзы, М — масса стержня):
= Ал. А	= i/Ж^Й = 2У\М .
1 2m'2M’P1 V т+М ' 1 V т(т-{-М) ’
Высота взлета h~v~l2g, Поэтому
hi о?____ М
V2 т М
Если массы гильзы и стержня сравнимы (т « М), то высота й2 не меньше чем вдвое превышает hi. Если же различие масс велико: т > М, то h2/hi « т/М > 1, т. е. и различие в высотах велико.
6.9.	При закрывании одного из отверстий жидкость, вытекающая из него, тормозится. При этом создается избыточное давление, которое ускоряет жидкость, вытекающую через оставшееся отверстие. Поэтому высота фонтанчика сначала увеличивается. После установления стационарного режима высота оставшегося фонтанчика оказалась бы иа прежнем уровне (при пренебрежении изменением уровня в сосуде), если бы не сказывалось трение. На деле эта высота несколько больше первоначальной, так как потери иа треиие в шланге уменьшаются из-за уменьшения скорости текущей в нем жидкости.
6.10.	Воздух при движении поршня сравнительно легко меняет свой объем и плотность, а вода — нет.
6.11.	Игрушка «водолаз» должна быть эластичной (например, резиновой), чтобы, сдавливая ее, можно было заставить ее тонуть (при уменьшении объема выталкивающая сила, направленная вверх, уменьшается и перестает компенсировать неизменную силу тяжести, действующую иа игрушку).
Существует критическая глубина, начиная с которой под влиянием только гидростатического и атмосферного давления игрушка сжата уже настолько, что выталкивающая сила меньше силы тяжести, действующей на нее. Если к моменту устранения дополнительного давления, вызванного поддувом воздуха, игрушка достигла критической глубины, то оиа начнет тонуть. Ее объем будет продолжать уменьшаться, и, следовательно, условия всплытия будут нарушаться все сильнее. «Водолаз» уже не всплывет. Если же «водолаз» не успел дойти до критической глубины, то он начнет стремительнее всплывать, увеличивая при этом свой объем.
6.12.	При нагреве воздуха его плотность уменьшается (р = = рц/RT) и начинается конвекция: меиее плотный нагретый воздух, обедненный кислородом, всплывает. На его место поступает более холодный воздух, обеспечивая тем самым приток к пламени необходимого для горения кислорода. При этом эти встречные по
251
токи тормозят друг друга и частично перемешиваются. При нехватке кислорода пламя коптит (содержит большое количество нё-сгоревшего, иеокислившегося углерода—сажи).
В трубке создается заметный поток воздуха^ обусловленный разностью давлений иа ее концах. При этом взаимодействие со встречным потоком отсутствует, поэтому скорость потока холодного воздуха велика (воздух в восходящую струю попадает лишь снизу, где давление максимально) и продукты сгорания быстро удаля. ются из области горения. Когда же трубка закрыта, нагретый воздух уже не может выйти вверх, начинает выходить снизу и ощутимо ограничивает доступ свежему воздуху. Пламя может погаснуть.
6.13.	В бутылке без трубочки вода при вытекании образует разреженную область вверху, которая препятствует вытеканию. Оно продолжится только после того, как через воду «пробулькнет» пузырь воздуха, выравнивающий давление с атмосферным. В результате вода вытекает медленнее, чем во втором случае, где трубочка обеспечивает постоянное нормальное давление над водой, несмотря на то что трубочка несколько уменьшает площадь, через которую вытекает вода, проходя через горлышко бутылки.
6.14.	В начальный момент манометр показывает резкое падение давления. Оно обусловлено в основном увеличением объема воздуха под поршнем. Затем давление увеличивается, превышая первоначальное значение. Это связано с тем, что ацетон интенсивно испаряется, при этом давление его пара, а значит и суммарное давление пара и воздуха, увеличивается. Росту давления способствует и рост температуры смеси до температуры окружающей среды.
6.15.	При взбалтывании за счет существенного увеличения поверхности соприкосновения горячей воды с воздухом происходит быстрый разогрев воздуха, увеличивающий давление в сосуде. Кроме того, резко возрастает испарение. К давлению воздуха добавляется заметное давление водяных паров. Шарик раздувается. Потом воздух остывает, пар конденсируется, давление спадает. Далее все повторяете^ но прн меньшей температуре, поэтому явление выражено слабее.
6.16.	В отличие от сухого сосуда, в мокром вначале внешнее давление компенсируется как давлением воздуха, так и давлением паров воды. Пары при охлаждении конденсируются. Это и приводит к тому, что уровень воды в мокром сосуде в итоге оказывается выше, чем в сухом.
6.17.	При внесении в колбу холодного стержня на ием происходит конденсация пара. Масса и температура пара уменьшается, что приводит к уменьшению Давления в закрытой колбе. При этом температура кипения воды также понижается. В результате уже слегка остывшая Рода, ейбва начинает кипеть, '
j 6.18. Вначале быстро разогреваются тонкие стеики металличе-ского сосуда и объем его увеличивается, поэтому уровень жидкости
; понижается. Затем прогревается сама жидкость, ее объем растет и ^уровень в трубке повышается.
6.19.	После надевания каркаса разность потенциалов между, сеткой и вертушкой становится равной нулю. Заряды перестают сте-, кать с вертушки. Момент сил, ранее заставлявший вертушку вращаться, обращается в нуль.
6.20.	Под действием поля внутри конденсатора на шарике индуцируются заряды. Они распределяются по поверхности шарика так, чтобы заряды, одноименные с зарядом одной пластины конденсаторов, расположились дальше от нее, а разноименные—ближе к ней. Это приводит к возникновению силы, притягивающей шарик к ближайшей из пластин. При соприкосновении шарика с пластиной на него переходит заряд, одноименный с зарядом пластины, возникает отталкивание от этой пластины и притяжение к другой. Далее все повторяется. Шарик переносит на себе заряды с одной обкладки конденсатора иа другую.
6.21.	Под действием поля .заряженного шара 1 на изолированном шаре 2 перераспределяются заряды (рис. 0.6.21). На заземленном шаре 3 индуцируется заряд, знак -которого противоположен знаку заряда на шаре 1. Когда шар 2 находится далеко от
Рис. 0.6.21
шара /, сила притяжения, действующая со стороны шара 3, больше, чем со стороны шара 1. На некотором расстоянии (его всегда можно подобрать) сила притяжения, действующая со стороны шара 1, становится больше, и шар 2 отклонится теперь уже к нему.
6.22.	Взаимодействие радиального тока и перпендикулярного плоскости колеса магнитного поля создает момент силы Ампера, направленной так, что колесо будет вращаться. При смене полюса направления силы, момента силы и вращения сменяются на противоположные.
6.23.	В сплошном кольце индуцируется кольцевой вихревой ток такого направления, что его взаимодействие с магнитным полем катушки приводит к отталкиванию кольца (вдоль стержня) от катушки. Когда кольцо разрезано, такой кольцевой ток возникнуть не может, и выталкивания кольца не наблюдается, *
253
6.24.	В пространстве между катушками с перемейным током возникает переменный магнитный поток. В металлическом листе наводятся вихревые токи так, чтобы создаваемое ими магнитное поле компенсировало изменение потока через металлический лиСт. Катушки отталкиваются от медного листа, а следовательно, и друг от друга. Железный лист—ферромагнетик. Пря его намагничивании появляется притяжение между листами и катушками, которое оказывается сильнее отталкивания, возникающего за счет вихревых токов.
6.25.	При разомкнутых концах взаимодействие медной проволоки с магнитным полем практически отсутствует и колебания почти не затухают.
При замкнутых концах в витках катушки при ее попадании в магнитное поле наводится индукционный ток, причем, по правилу Ленца, такого направления, чтобы воспрепятствовать изменению магнитного потока через катушку. В результате на переднюю часть витка, вошедшего в сильное поле Blt действует сила Ff, направленная против движения. На другую часть витка, находящуюся в периферийной области поля, где оно значительно слабее (В2 < Bi, поле неоднородно), действует сила F2 противоположного направления. Но Ft<^.Fi, поэтому результирующая сила тормозит движение.
При выходе из поля возникает результирующая сила, препятствующая выходу; Индукционные токи приводят к тепловым потерям энергии в проводе катушки, в результате чего колебания быстро затухают.
6.26.	Частоты колебаний ®х и ®2=®1+Л® отличаются на небольшую величину Дер. При сложении возникают биения:
sin «>xt+Sin (®i< + Дай) —
= 2 cos (Aco//2)'Sin (®x<+Дсо//2) » 2 cos (Ди#/2) sin a>it.
Амплитуда биений 2 cos (Дой/2) медленно периодически увеличивается н уменьшается. Это и соответствует усилению и ослаблению звука.
' 6.27. Возбуждаемые в струне стоячие волны образуют систему узлов и пучностей. В пучности взаимодействие магнитного поля магнита с током в струне максимально н приводит к увеличению амплитуды и, следовательно, громкости звука. Для узла изменение минимально.
6.28.	Волна, идущая от динамика, интерферирует с волйбй, отражённой от поршня, и образует стоячую волну.
На поршне всегда узел, т. е. минимум амплитуды колебаний. Громкость'на втором-динамике будет максимальной, когда микрофон находится в Пучности давления, совпадающей с узлом ампли-264
дуды стоячей волны (рис, Q.6.28). Если поршень сдвинуть иа расстояние Д/»Х/2, то микрофон опять будет находиться в пуч» ности давления Волны. Измеряя Д/ и зная частоту волны v, мбж*1 но вычислить скорость звука c = %v = 2vA/.
6.29.	Если иа динамик подается напряжение Ux = 1)л cos art, то с микрофона снимается запаздывающее по фазе напряжение
иу=ик cos (at + q) — U„ cos [co (t — f0)], где сдвиг по фазе обусловлен, в частности, временем ta прохождв» иия звуком расстояния от динамика до микрофона. Световое пят-
Рис. 0.6.30
ио на экране, таким образом, участвует в двух взаимно перпендикулярных колебаниях одинаковой частоты, ио разных амплитуд, имеющих сдвиг фазы.
В общем случае иа экране виден эллипс. Если сдвиг фазы <р = лй, где k — 0, 1, 2, ..., то эллипс вырождается в отрезок « углом наклона к горизонтальной оси
а± = arctg (у/х) = arctg (UUIU^.
Сдвиг фазы <р = й>/о = со//с, где с—скорость звука. В случав вырождения эллипса в отрезок по величинам /*, со, используя связь лй = а>//с, можно определить скорость звука:
2п = ((о/с)|/*— lk± J, c = |/fe— lk± i|/T.
6.30.	Прн вдвигании жё*лезиого сердечника индуктивность катушки и, следовательно, ее индуктивное сопротивление возрастают. Свечение лампочки прн увеличении индуктивности первой катушки будет возрастать, если эта катушка включена в цепь параллельно лампочке. Аналогично, свечение лампочки при увеличении индуктивности второй катушки будет уменьшаться, если катушка включена в цепь последовательно с лампочкой. Таким образом, приходим к схеме, изображенной иа рис. 0.6.30.
6.31.	Очевидно, что одни из источников дает постоянное напряжение, а второй —переменное. При переменном токе дуга существенно слабее, чем при постоянном (проявляется большее реактивное сопротивление катушки индуктивности).
2S5
6.32.	Поскольку реактивные сопротивления и 1/свС имеют разные знаки (колебания напряжения иа них противоположны по фазе), то при замыкании ключа 2, если coL ~ 1/аС, происходит их компенсация и переменный ток через лампочку возрастает по сравнению со случаем отключенной емкости.
6.33.	Три белые полосы постоянно видны лишь в том случае, если за время между двумя вспышками полоса, совершив любое число оборотов, сдвинется на угол 2л/3 или 4л/3. Полный угол поворота полосы (pi = ink -|- 2л/3 или ф2 = 2лй-|-4л/3, где k—любое целое число. Таким образом,
„ ЗЛ-f-l „ ЗЛ4-2	2л
ф! = 2л——, ф2 = 2л—J— , т. е. ф=-—п, ООО
где п — любое число, не кратное 3. Угловая скорость вращения диска, таким образом, равна v-2nn/3.
6.34.	На рис. 0.6.34 изображена схема эксперимента. Здесь S —лампочка, S' и S"«-ee первые изображения в'зеркалах (в действительности изображений бесконечно много, но для объяснения явления достаточно рассмотреть лишь два изображения). На уча-
сток CD экрана падает лишь прямой свет от лампочки, а на участки ВС и DE падает как прямой свет, так и свет, отраженный от зеркала. Вот почему участки ВС и DE выглядят светлее участка CD. Далее картина повторяется по обе стороны от центральной полосы.	*
6.35.	При наличии воды стеклянные пластины практически не препятствуют распространению света, так как показатели преломления стекла и воды близки: среда оказывается оптически почти однородной (вспомните—погруженное в воду стекло становится едва видимым). В отсутствие воды возникает большое число отражений-от пластин и световой поток ослабляется: изображение исчезает.
6.36.	Когда на пути параллельного пучка встает стеклянная пластина, фототок уменьшается, так как часть светового потока стекло отражает. Такой же эффект потери части света, конечно, существует и для лампочки накаливания. Однако лучи света, па-256
дающие под углом на плоскопараллельную пластинку, смещаются, пройдя ее. Таким образом, часть потока, прежде ие попадавшая на фотоэлемент, теперь дает дополнительный вклад, который при соответствующем подборе условий превысит эффект потерь.
6.37.	Если экран помещен за двойным фокусным расстоянием линзы, возникает характерное распределение света, объясняемое с помощью рис. 0.6.37. В области АВ и EF проходит свет прямого пучка, в области CD распределяется свет, попавший на линзу, но после прохождения фокуса распределившийся по большей, чем у линзы, площади размера BE, в результате чего кружок CD освещен слабее, чем область прямого пучка. Наконец, участки ВС и
ED освещаются светом как прямого пучка, так и пучка, отклоненного линзой. Отсюда —наблюдаемая на экране кольцевая структура.
6.38.	Как видно из построений рис. 0.6.38, полоска разбивает световой поток на две части, каждая из которых формирует размытое изображение предмета. Штриховой линией указано положение экрана, при котором этн изображения сливаются в одно резкое.
6.39.	Наличие преломляющей толстой пластины-кюветы эквивалентно смещению источника света на некоторое расстояние. Когда кювету убирают, это эквивалентно смещению лампочки на некоторое расстояние вдоль оси линзы, что и приводит к расфокусировке изображения, так как резкое изображение спирали должно теперь получиться при другом положении экрана.
6.40.	Вследствие охлаждения банки на ее поверхности конденсируется водяной пар, что приводит к помутнению отражающей поверхности (появляются капельки росы или даже иней, если температура охлажденной поверхности ниже нуля). Вместо отражения в определенном направлении свет начинает рассеиваться в довольно широком диапазоне углов. В итоге освещенность экрана в области первоначального зайчика уменьшается,
257
6.41.	Вследствие многократных отражений от обеих поверхностей толстой пластины (рис. 0.6.41) наблюдается несколько вы. шедших по обе стороны пластины пучков, последовательно ослаб, ленных, так как каждый раз при встрече с поверхностью пластины пучок частично отражается поверхностью, а частично преломляется. Наиболее яркими должны быть самые крайние пятна.
6.42.	Светлая точка О—место нгГ закрашенной стороне 1 пластины, куда попадает лазерный луч (рис. 0.6.42). Свет, рассеянный из этой точки, в результате полного внутреннего отражения от противоположной границы 2 вновь попадает на закрашенную сторону 1, начиная с расстояния 7? = 2d/tg<p0 от светлой точки О (d—толщина пластины, <р0—угол полного внутреннего отражения: sin<p0=l/n, где п — показатель преломления вещества пластины). До этого на закрашенную часть попадает лишь не полностью отраженный свет.
6.43.	Число рассеивающих и поглощающих свет частиц вдоль оси не меняется при разбавлении, поскольку площадь сечения сосуда неизменна, полное число частиц сохраняется и, значит, число частиц, встреченных светом на пути вдоль оси, сохраняется (во сколько раз увеличивается путь, во, столько же раз уменьшается концентрация молока, воду считаем абсолютно прозрачной).
При прохождении света поперек оси нз-за уменьшения числа частиц в единице объема при разбавлении уменьшается и число частиц, встреченных светом на этом пути (концентрация уменьшилась, а .путь остался прежним). Таким образом, в последнем случае больше света пройдет при разбавлении, а в первом разбавление практически не отражается на интенсивности прошедшего света.
СПИСОК РЕКОМЕНДУЕМОЙ ЛИТЕРАТУРЫ
1.	Элементарный учебник физики: Учебное пособие. В 3-х т./Под ред. Г. С. Ландсберга.— 10-е изд., перераб.— М.: Наука, 1985— 1986.
2.	Бутиков Е. И., Быков А. А., Кондратьев А. С. Физика в примерах и задачах.— 3-е изд.— М.: Наука, 1989.
3.	БрукЮ. М., Стасенко А. Л. Как физики делают оценки — метод размерностей и порядки физических величин//О современной физике — учителю.— М.: Знание, 1975.
4.	Перельман Д. И. Занимательная физика. В 2-х кн.— 21-е изд.— М.: Наука, 1986.
5.	Маковецкий П. В. Смотри в корень.— 5-е изд.— М.: Наука, 1984.
6.	Уокер Дж. Физический фейерверк: Вопросы и ответы по физике: Пер. с англ./Под ред. И. Ш. Слободецкого.— М.: Мир, 1979.
7.	Коган Б. 10. Сто задач по физике.— 2-е изд.— М.: Наука, 1986; Размерность физической величины.— М.: Наука, 1968.
8.	Ланге В. Н. Физические парадоксы, софизмы.— М.: Просвещение, 1967.
9-	. Енохович А. С. Справочник.по физике и технике.— М.: Просвещение, 1976.
10.	Гольдфарб Н. И. Сборник вопросов и задач по физике.— М.: Просвещение, 1982.
11.	Сборник задач по элементарной физике/Б. Б. Буховцев, Г. Я. Мякишев, В. Д. Кривченков, И. М. Сараева.— 5-е изд.— М.: Наука, 1987.
12.	Балаш В. А. Задачи по физике и методы их решения.— М.: Просвещение, 1983.
13.	Тульчинский М. Е. Занимательные задачи-парадоксы и софизмы по физике.— М.: Просвещение, 1971.
14.	Ащеулов С. В., Барышев В. А. Задачи по элементарной физике.— Л.: Изд-во Ленинград, ун-та, 1974.
15.	Задачи по физике/И. И. Воробьев, П. И. Зубков, Г. А. Кутузова и др.; Под ред. О. Я. Савченко.— 2-е изд.— М.: Наука, 1988.
16.	Слободецкий И. Ш., Асламазов Л. Г. Задачи по физике.— М.: Наука, 1980.— (Б-чка «Квант»; Вып. 5).
17.	Слободецкий И. Ш., Орлов В. А. Всесоюзные олимпиады по физике.— М.: Просвещение, 1982.
18.	Сборник задач по физике/Л. П. Баканина, В. Е. Белонучкин, С. М. Козел, И. П. Мазанько; Под ред. С. М. Козела, — М.: Нау-' ка, 1983.
19.	Задачи московских физических олимпиад/Под ред. С. С. Кротова,— М,: Наука, 1988,— (Б-чка «Квант»; Вып. 60),
Приложение
ЕДИНИЦЫ И РАЗМЕРНОСТИ ФИЗИЧЕСКИХ ВЕЛИЧИН В СИ
Определения единиц физических величин приведены для основных и дополнительных единиц СИ. Внесистемные единицы, допустимые к применению наравне с единицами СИ, отмечены кружком
Величина		Единица		Связь с основными единицами СИ
Длина	L	метр “астрономическая единица длины “световой год “парсек	м а. е. св. год ПК	Основная единица Метр представляет собой расстояние, проходимое в вакууме плоской электромагнитной волной за 1/299792458 долю секунды 1 а. е. = 1,49598-1011 м 1 св. год = 9,4605.1018 м 1 пк=3,0857-101в м
Площадь	L2	квадратный метр °гектар	м2 га	1 га = 104 м3
Объем	L3	кубический метр “литр	«•5 S 4 			1 л = 10~3 м3
Плоский угол	——	радиан	рад	Дополнительная единица Радиан равен углу между двумя радиусами окружности, длина дуги между которыми равна радиусу
Величина		Единица	
		“градус “минута “секунда	
Телесный угол	—	стерадиан	
Время	т	секунда °минута °час °сутки	
Скорость	LT-1	метр в секунду	
Ускорение го 2 		LT-*	метр на секунду в квадрате	
Продолжение
	Связь с основными единицами СИ
О ! Н i <	1° = (л/180) рад 1' = (rt/10800) рад 1" = (л/648000) рад
ср	Дополнительная единица Стерадиан равен телесному углу с вершиной в центре сферы, вырезающему на поверхности сферы площадь, равную площади квадрата со стороной, равной радиусу сферы
с мин ч сут	Основная единица Секунда равна 9 192 631 770 периодов излучения, соответствующего переходу между двумя сверхтонкими уровнями основного состояния атома цезия-133 1 мнн = 60 с 1 ч = 3600 с 1 сут = 86400 с
м/с	
м/с2	
Продолжение
Величина		Единица		Связь с основными единицами СИ
Угловая скорость	7-1	радиан в секунду	рад/с	1 рад/с=1 с-1
Угловое ускорение	7-2	радиан на секунду в квадрате	рад/с2	1 рад/с2 = 1 с'2
Частота периодического процесса	7-1	герц	Гц	1 Гц= 1 с~х
Частота вращения	7-1	секунда в минус первой степени	с-1	
Масса	М	килограмм °тоина °атомная единица массы	КГ т а. е. м.	Основная единица Килограмм равен массе международного прототипа килограмма 1 т= 103 кг 1 а. е. м. = 1,6605655-10-27 кг
Плотность	L~3M	килограмм на кубический метр	кг/м3	
Удельный объем	L3M~i	кубический метр на килограмм	м3/кг	
Продолжение
Величина		Единица		Связь с основными единицами СИ
Массовый расход	МТ-1	килограмм в секунду	кг/с	
Объемный расход	L3T-1	кубический метр в секунду	м3/с	
Сила	LMT~2	ньютон	н	1 Н = 1 кг-м-с-2
Давление	L-WT-1	паскаль	Па	1 Па— 1 Н/м2=1 м_1-кг-с~2
Жесткость	мт-2	ныотон на метр	Н/м	1 Н/м=1 кг-с-2
Нап-ряжение	L~'MT-2	паскаль	Па	1 Па = 1 Н/м2=1 м_1-кг-с~2
Поверхностное натяжение	МТ~2	ньютон на метр	Н/м	1 Н/м = 1 кг-с~2
Импульс (количество движения)	LMT-1	килограмм-метр в секунду	кг-м/с	
Момент силы	L2MT~2	ньютон-метр	Н-м	1 Н -м = 1 м2-кг-с~2
Момент импульса (момент количества движения)	L41T-1	килограмм-метр в квадрате в секунду	кг-м2/с	
Величина		Единица	
Момент инерции	(ЛИ	килоггамм-метр в квадрате	
Работа, энергия	L2MT~2	джоуль	
Мощность (поток энергии)	L2MT~3	ватт	
Температура (термодинамическая)	0	кельвин °градус Цельсия	
Температурный коэффициент	0-i	кельвин в минус первой степени	
Количество вещества	N	моль	
Молярная масса	MN-1	килограмм на моль	
Продолжение
	Связь с основными единицами СИ
кг - м2	
Дж	1 Дж—1 Н-м=1 м2-кг-с~2
Вт	1 Вт=1 Дж/с = 1 м2-кг-с~3
К	Основная единица Кельвин равен 1/273,16 части термодинамической температуры тройной точки воды
°C	//°С = 77К —273,15
к-1	
моль	Основная единица Моль равен количеству вещества системы, содержащей столько же структурных элементов, сколько содержится атомов в углероде-12 массой 0,012 кг
кг/моль	
Продолжение
Величина		Единица		Связь с основными единицами СИ
Молярный объем	LW-1	кубический метр на моль	м3/моль	
Количество теплоты (теплота)	L2MT~2	джоуль	Дж	1 Дж = 1 Н-м = 1 м2-кг-С-2
Удельная теплота	L2T~2 F	джоуль * на килограмм	Дж/кг	1 Дж/кг = 1 М2-С“2
Молярная теплота	L2MT~2N~1	джоуль на моль	Дж/моль	1 Дж/моль = 1 М2-КГ-С_2-МОЛЬ-1
Теплоемкость	L2MT~2@~l	джоуль на кельвин	Лж/К	1 Дж/К=1 м2-КГ . С-2-К-1
Удельная теплоемкость	L2T-2Q-1	джоуль на килограмм-кельвин	Дж/(кг-К)	1 Дж/(кг-К) = 1 м2-с-2-К_х
Молярная теплоемкость	L,2MT~2Q~1N-1	джоуль на моль-кельвин	Дж/(моль-К)	1 Дж/(моль • К) = = 1 М2.КГ-С-2-К~1-МОЛЬ-1
Тепловой поток	L2MT~3	ватт	Вт	1 Вт=1 Дж/с=1 м2-кг-с-3
Плотность теплового потока	MT-3	ватт на квадратный метр	В т/м2	1 Вт/м2 = кг-с-3
Концентрация (плотность ЧИС-ла частиц) сл		L~3	метр в минус третьей степени	м~3	
Величина		Единица	
Коэффициент диффузии	£27-1	квадратный метр в секунду	
Сила электрического тока	/	ампер	
Плотность электрического тока	L~2I	ампер на квадратный метр	
Количество электричества (электрический заряд)	TI	кулон	
Поверхностная плотность электрического заряда	L~*TI	кулон иа квадратный метр	
Продолжение
	Связь с основными единицами СИ
м«/с	
А	Основная единица Ампер равен силе неизменяющегося тока, который при прохождения по двум параллельным проводникам бесконечной длины и ничтожно малой площади кругового поперечного сечения, расположенным в вакууме иа расстоянии 1 м один от другого, вызвал бы на каждом участке проводника длиной 1 м силу взаимодействия, равную 2-10“7 Н
А/м2	
Кл	1 Кл=1 с-А
Кл/м2	1 Кл/м2 = 1 м-2-с-А
Продолжение
14Z
Величина		Единица		Связь с основными единицами СИ
Пространственная плотность электрического заряда	L~ST1	кулон на кубический метр	Кл/м3	1 Кл/м3—1 м~з.с.А
Электрическое напряжение, электрический потенциал, разность электрических потенциалов, электродвижущая сила	L2MT-з/-1	вольт	В	1 В = 1 Вт/А —1 м2-кг-с~3-А-1
Напряженность электрического поля	LMT-3!-1	вольт на метр	В/м	1 В/м = 1 Вт/(А-м)— 1 м-кг-с~3-А-1
Электрическое сопротивление	L2MT~3/~2	ом	Ом	1 Ом = I B/A--I м2-кг-с~з.А~2
Удельное электрическое сопротивление	L3MT~3I~3	ом-метр	Ом -м	1 Ом-м = 1 м3-кг-с-3-А-2
Электрическая проводимость	L-2M~'-T312	сименс	См	1 См — 1 Ом-1 —1 м-2.кг-1'С3-А2
'Величина		Едини
Удельная электрическая проводимость	£-3^-173/2	сименс на метр
Электрическая емкость		фарад
Электрическая постоянная	/.-ад/-174/2	фарад на метр
Поток электриче- ского смещения	Т1	кулон
Электрическое смещение	L~2TI	кулон на квадратный метр
Магнитный поток	l2mt~2/~i	вебер
Магнитная индукция	мт^/-1	тесла
Индуктивность	L2MT~21~2	генри
Магнитная постоянная	LMT~21~2	генри на метр
П родолжение
на	Связь с основными единицами СИ
См/м	I См/м= 1 Ом_1-м-1= 1 м~3-кг~1-с3-А2
Ф	1 ф=1 Кл/В = 1 м-2-кг-1.с1-А2
Ф/м	1 Ф/м = 1 м-3-кг_1-с4-А2
Кл	I Кл = 1 с-А
Кл/м2	1 Кл/м2=1 м~2-с-А
Вб	1 Вб=1 В-с= 1 Тл-м2= 1 м2-кг-с-2-А-1
Тл	1 Тл= 1 В-с/м2= 1 Вб/м2 = 1 кг-с-2-А-1
Гн	1 Гн = 1 м2-кг-с-2.А-2
Ги/м	1 Гн/м = 1 м-кг-с-2-А-2
Продолжение
Величина		Единица		Связь с основными единицами СИ
Напряженность магнитного поля	L-17	ампер на метр	А/м	
Энергия излучения	L2MT~2	джоуль	Дж	1 Дж = 1 М2-КГ-С“2
Мощность излучения (поток излучения)	L2MT-3	ватт	Вт	1 Вт=1 Дж/с=1 м2-кг-с-3
Интенсивность излучения (плотность потока излучения)	МТ~3	ватт иа квадратный метр	Вт/м2	1 Вт/м2=1 кг-с-3
Поток частиц	Т-1	секунда в минус первой степени	с-1	
Плотность потока частиц	L-2T-1	секунда в минус первой степени-метр в минус второй степени	с-1-м~2	
о
Продолжение
Величина		Единица		Связь с основными единицами СИ
Сила света	J	кандела	кд	Основная единица Кандела равна силе света в заданном направлении источника, испускающего монохроматическое излучение частотой 540-1012 Гц, энергетическая сила света которого в этом направлении составляет 1/683 Вт/ср
Световой поток	J	люмен	лм	1 лм = 1 кд-ср.
Световая энергия	TJ	люмен-секунда	лм*с	1 лм-с=1 с-кд-ср
Светимость	L-4	люмен на квадратный метр	лм/м2	1 лм/м2 = 1 м~2-кд-ср
Освещенность	L-zj	люкс	лк	1 лк = 1 лм/м2=1 м_2-кд-ср
Яркость	L-4	кандела на квадратный метр	кд/м2	
Оптическая сила	L~1	°диоптрия	дптр	1 дптр = 1 м-х